SlideShare a Scribd company logo
1 of 148
Pediatric Board ReviewPediatric Board Review
2006-20072006-2007
Session One: July 24, 2006Session One: July 24, 2006
Session Two: August 24, 2006Session Two: August 24, 2006
Session Three: September 26, 2006Session Three: September 26, 2006
www.indiandentalacademy.comwww.indiandentalacademy.com
INDIAN DENTAL ACADEMY
Leader in continuing dental education
www.indiandentalacademy.com
BibliographyBibliography
 Blueprints in Pediatrics 1998Blueprints in Pediatrics 1998
 Nelson’s Pediatrics 17Nelson’s Pediatrics 17thth
EditionEdition
 PREP Questions 1999-2006PREP Questions 1999-2006
 Up To DateUp To Date
www.indiandentalacademy.comwww.indiandentalacademy.com
Scrabble QuizScrabble Quiz
AEINRSTAEINRST
 This is the most common seven letterThis is the most common seven letter
combination you can have on your rack. Therecombination you can have on your rack. There
are 8 different seven letter words “bingos” thatare 8 different seven letter words “bingos” that
can be spelled from this. Can you name one?can be spelled from this. Can you name one?
– Anestri
– Nastier
– Ratines
– Retains
– Retinas
– Retsina
– Stainer
– Stearin
www.indiandentalacademy.comwww.indiandentalacademy.com
Scrabble QuizScrabble Quiz
 There are 101 acceptable two letter wordThere are 101 acceptable two letter word
plays in Scrabble, 16 of which start withplays in Scrabble, 16 of which start with
what letter?what letter?
 AA
 AA, AB, AD, AE, AG, AH, AI, AL, AM, AN,AA, AB, AD, AE, AG, AH, AI, AL, AM, AN,
AR, AS, AT, AW, AX, AYAR, AS, AT, AW, AX, AY
www.indiandentalacademy.comwww.indiandentalacademy.com
Scrabble QuizScrabble Quiz
 The word ROOST has 4 anagrams, nameThe word ROOST has 4 anagrams, name
them.them.
– ROOTSROOTS
– ROTOSROTOS
– TOROSTOROS
– TORSOTORSO
www.indiandentalacademy.comwww.indiandentalacademy.com
Question 1.2000Question 1.2000
 A TRUE statement about the epidemiology ofA TRUE statement about the epidemiology of
measles is thatmeasles is that
– A. In countries with no immunization programs, theA. In countries with no immunization programs, the
peak age of infection is infancypeak age of infection is infancy
– B. In US immunization programs have reduced theB. In US immunization programs have reduced the
incidence of infection by 80%incidence of infection by 80%
– C. Usually spread by direct contact with infectiousC. Usually spread by direct contact with infectious
dropletsdroplets
– D. Patients become contagious when the rashD. Patients become contagious when the rash
appearsappears
– E. Incubation period is 4-5 days from exposure toE. Incubation period is 4-5 days from exposure to
onset of symptomsonset of symptoms
 Answer CAnswer C
www.indiandentalacademy.comwww.indiandentalacademy.com
Teaching Points 1.2000Teaching Points 1.2000
 MeaslesMeasles
– Cough, coryza (inflammation of nasalCough, coryza (inflammation of nasal
mucosal membranes), fever, conjunctivitis,mucosal membranes), fever, conjunctivitis,
exanthem of red macules and papules andexanthem of red macules and papules and
Koplik spotsKoplik spots
– Young children have OM, pneumonia, croupYoung children have OM, pneumonia, croup
and diarrheaand diarrhea
– Acute encephalitis (1:1000) -->PermanentAcute encephalitis (1:1000) -->Permanent
brain injurybrain injury
– In US death 1-3/1000 due to respiratory ofIn US death 1-3/1000 due to respiratory of
neurologic complicationsneurologic complications
www.indiandentalacademy.comwww.indiandentalacademy.com
Teaching Points 1.2000Teaching Points 1.2000
 Transmitted by direct contact with infectiousTransmitted by direct contact with infectious
droplets ordroplets or LESS COMMONLYLESS COMMONLY airborne spreadairborne spread
 Since vaccine use in 1963 there has been aSince vaccine use in 1963 there has been a
99% reduction99% reduction in incidence in USin incidence in US
 Children are contagiousChildren are contagious 4-5 days BEFORE rash4-5 days BEFORE rash
appears to 4 days after appearance of rashappears to 4 days after appearance of rash
 Incubation period is 8-12 days from exposure toIncubation period is 8-12 days from exposure to
onset of symptomsonset of symptoms
www.indiandentalacademy.comwww.indiandentalacademy.com
www.indiandentalacademy.comwww.indiandentalacademy.com
Koplik SpotsKoplik Spots
Henry Koplik (U.S. Physician, 1858 to 1927)Henry Koplik (U.S. Physician, 1858 to 1927)
www.indiandentalacademy.comwww.indiandentalacademy.com
Question 2.2000Question 2.2000
 A previously healthy 2 year old is brought to EDA previously healthy 2 year old is brought to ED
because her mom is unable to awaken her forbecause her mom is unable to awaken her for
45 minutes. She has not been ill. PE reveals an45 minutes. She has not been ill. PE reveals an
afebrile, hypotonic child who withdraws her handafebrile, hypotonic child who withdraws her hand
from painful stimuli but does not spontaneouslyfrom painful stimuli but does not spontaneously
open her eyes. RR is 36/min, BP is 92/64. Whatopen her eyes. RR is 36/min, BP is 92/64. What
is the next best thing to obtain?is the next best thing to obtain?
– A. BUNA. BUN
– B. CXRB. CXR
– C. EKGC. EKG
– D. EEGD. EEG
– E. Toxicology screenE. Toxicology screen
 Answer EAnswer E
www.indiandentalacademy.comwww.indiandentalacademy.com
Teaching Points 2.2000Teaching Points 2.2000
 Pneumonic for Causes of AlteredPneumonic for Causes of Altered
Consciousness: “Tips from the Vowels”Consciousness: “Tips from the Vowels”
 T-I-P-S-A-E-I-O-UT-I-P-S-A-E-I-O-U
– Trauma/TumorTrauma/Tumor
– Infection/Inborn Errors/Insulin/hypoglycemiaInfection/Inborn Errors/Insulin/hypoglycemia
– PoisonsPoisons
– ShockShock
– Alcohol/AbuseAlcohol/Abuse
– Epilepsy/EncephalopathyEpilepsy/Encephalopathy
– IntussusceptionIntussusception
– OpiatesOpiates
– UremiaUremia
www.indiandentalacademy.comwww.indiandentalacademy.com
Question 3.2000Question 3.2000
 You have intubated an 8 month old with sepsis and apneaYou have intubated an 8 month old with sepsis and apnea
with a 4.0 endotracheal tube. Proper placement is confirmedwith a 4.0 endotracheal tube. Proper placement is confirmed
by observing chest rise and auscultating symmetric breathby observing chest rise and auscultating symmetric breath
sounds after bag and mask ventilation. Perfusion is <3sounds after bag and mask ventilation. Perfusion is <3
seconds, and heart rate is 120/minute. Five minutes laterseconds, and heart rate is 120/minute. Five minutes later
the RT tells you the oxygen saturation is 83% and the bloodthe RT tells you the oxygen saturation is 83% and the blood
pressure and pulse are dropping. Breath sounds are absentpressure and pulse are dropping. Breath sounds are absent
on the right and the right chest is hypertympanitic. What ison the right and the right chest is hypertympanitic. What is
the most likely diagnosis?the most likely diagnosis?
– A. Esophageal intubationA. Esophageal intubation
– B. Incorrect ET sizeB. Incorrect ET size
– C. Obstructed ET tubeC. Obstructed ET tube
– D. Right main stem bronchus intubationD. Right main stem bronchus intubation
– E. Right pneumothoraxE. Right pneumothorax
 Answer EAnswer E
www.indiandentalacademy.comwww.indiandentalacademy.com
Teaching Points 3.2000Teaching Points 3.2000
 ET Tube Size (mm) = …ET Tube Size (mm) = …
– [16 + Age(yr)]/4[16 + Age(yr)]/4
– Preterm 2.5-3.0Preterm 2.5-3.0
– Term 3.0-3.5Term 3.0-3.5
– Infant 3.5-4.0Infant 3.5-4.0
– 1 year 4.0-4.51 year 4.0-4.5
– 3 year 4.5-5.03 year 4.5-5.0
– *An uncuffed tube should be used in patients…*An uncuffed tube should be used in patients…
 <8 years of age<8 years of age
 Approximate distance of insertion measured at teethApproximate distance of insertion measured at teeth
or lips in cm = …or lips in cm = …
– Internal Diameter X 3Internal Diameter X 3
 Straight blades preferred for neonates and infantsStraight blades preferred for neonates and infants
www.indiandentalacademy.comwww.indiandentalacademy.com
Teaching Points 3.2000Teaching Points 3.2000
 Drugs that can be given by ETT after dilution withDrugs that can be given by ETT after dilution with
normal saline to 3-5 ml followed by positivenormal saline to 3-5 ml followed by positive
pressure ventilation…pressure ventilation…
 LANELANE
– LidocaineLidocaine
– AtropineAtropine
 0.02mg/kg IV,IO; 0.02-0.06mg/kg ET0.02mg/kg IV,IO; 0.02-0.06mg/kg ET
 Minimum dose is 1ml (0.1 mg) as it comes 0.1 mg/mlMinimum dose is 1ml (0.1 mg) as it comes 0.1 mg/ml
– Naloxone (Narcan)Naloxone (Narcan)
 0.1 mg/kg/dose IM/ET/IV/IO to maximum 2mg/dose. May repeat0.1 mg/kg/dose IM/ET/IV/IO to maximum 2mg/dose. May repeat
every 2-3 minutesevery 2-3 minutes
– EpinephrineEpinephrine
 0.1ml/kg of 1:1000 (0.1mg/kg) IV, IO, ET in non-neonates0.1ml/kg of 1:1000 (0.1mg/kg) IV, IO, ET in non-neonates
 Use 1:10,000 (0.1-0.3ml/kg for all doses and routes) epinephrineUse 1:10,000 (0.1-0.3ml/kg for all doses and routes) epinephrine
via ETT for neonates only. Use high dose 1:1,000 epinephrine forvia ETT for neonates only. Use high dose 1:1,000 epinephrine for
ETT beyond neonatal periodETT beyond neonatal period
www.indiandentalacademy.comwww.indiandentalacademy.com
PneumothoraxPneumothorax
www.indiandentalacademy.comwww.indiandentalacademy.com
PneumothoraxPneumothorax
www.indiandentalacademy.comwww.indiandentalacademy.com
Teaching Points 3.2000Teaching Points 3.2000
 Esophageal intubation essentially ruled outEsophageal intubation essentially ruled out
because of previous presence of rise and fall ofbecause of previous presence of rise and fall of
chest and presence of breath sounds unilaterallychest and presence of breath sounds unilaterally
 However, ET tube could have moved if theyHowever, ET tube could have moved if they
repositioned patient; check depth marker atrepositioned patient; check depth marker at
teeth/lipsteeth/lips
 If ETT tube is too small, air leaking around tubeIf ETT tube is too small, air leaking around tube
can impair efforts to ventilate but an audible aircan impair efforts to ventilate but an audible air
leak generally is heard with each positiveleak generally is heard with each positive
pressure breathpressure breath
www.indiandentalacademy.comwww.indiandentalacademy.com
Teaching Points 3.2000Teaching Points 3.2000
 Treatment for tension pneumothoraxTreatment for tension pneumothorax
– Immediate needle decompressionImmediate needle decompression
– Patient supine with head of bed at 30 degreePatient supine with head of bed at 30 degree
– 18-20 gauge over the needle catheter (angiocatheter)18-20 gauge over the needle catheter (angiocatheter)
inserted into the…inserted into the…
 22ndnd
intercostal space just “over the top” (Hey dude, this procedureintercostal space just “over the top” (Hey dude, this procedure
is over the top , man!) of the 3is over the top , man!) of the 3rdrd
rib at the midclavicular linerib at the midclavicular line
– 5-10 ml syringe attached to angiocatheter and aspirated5-10 ml syringe attached to angiocatheter and aspirated
gently as needle is advancedgently as needle is advanced
– Loss of resistance or rush of air apparent as soon asLoss of resistance or rush of air apparent as soon as
pleural space is enteredpleural space is entered
– If pneumothorax is confirmed a one-way drainage deviceIf pneumothorax is confirmed a one-way drainage device
should be attachedshould be attached
– A chest tube should be placed after successful needleA chest tube should be placed after successful needle
decompressiondecompression
www.indiandentalacademy.comwww.indiandentalacademy.com
Needle DecompressionNeedle Decompression
www.indiandentalacademy.comwww.indiandentalacademy.com
Question 4.2000Question 4.2000
 A 6 year old boy with severe factor VIII deficiencyA 6 year old boy with severe factor VIII deficiency
hemophilia develops increased swelling of the righthemophilia develops increased swelling of the right
distal forearm. There is not history of trauma. Thedistal forearm. There is not history of trauma. The
peripheral circulation is normal and there is no jointperipheral circulation is normal and there is no joint
involvement. The MOST important complication ofinvolvement. The MOST important complication of
bleeding in this location is:bleeding in this location is:
– A. Blood lossA. Blood loss
– B. Muscular DamageB. Muscular Damage
– C. Neurologic ImpairmentC. Neurologic Impairment
– D. Tendon ShorteningD. Tendon Shortening
– E. Vascular DamageE. Vascular Damage
 Answer CAnswer C
www.indiandentalacademy.comwww.indiandentalacademy.com
Teaching Points 4.2000Teaching Points 4.2000
 Children with hemophilia can develop severeChildren with hemophilia can develop severe
peripheral neurologic deficits when hematomasperipheral neurologic deficits when hematomas
compress nerves via compartment syndromecompress nerves via compartment syndrome
 In older children increasing pain out of proportionIn older children increasing pain out of proportion
to size of hematoma, numbness and paresis areto size of hematoma, numbness and paresis are
critical signscritical signs
 Often there is no history of trauma inOften there is no history of trauma in
hemophiliacshemophiliacs
 Significant blood loss seen in hematomas of…Significant blood loss seen in hematomas of…
– Thigh or retroperitoneumThigh or retroperitoneum
www.indiandentalacademy.comwww.indiandentalacademy.com
Teaching Points 4.2000Teaching Points 4.2000
 Routine venipuncture is safe as long as it isRoutine venipuncture is safe as long as it is
followed by…followed by…
– 5 minutes of firm finger pressure5 minutes of firm finger pressure
 Femoral or jugular venipuncture or arterialFemoral or jugular venipuncture or arterial
puncture should not be undertaken WITHOUT…puncture should not be undertaken WITHOUT…
– PRIOR FACTOR REPLACEMENTPRIOR FACTOR REPLACEMENT
 What about IM immunizations…What about IM immunizations…
– Ok as long as followed by 5 minutes of firm fingerOk as long as followed by 5 minutes of firm finger
pressurepressure
– Large IM injections should be avoided (Decadron,Large IM injections should be avoided (Decadron,
Rocephin IM etc)Rocephin IM etc)
www.indiandentalacademy.comwww.indiandentalacademy.com
Teaching Points 4.2000Teaching Points 4.2000
 Tendon shortening only occurs with…Tendon shortening only occurs with…
– Chronic, sever hemarthrosesChronic, sever hemarthroses
– Limited mobilityLimited mobility
– Nerve damageNerve damage
 Muscular damage is not frequent amongMuscular damage is not frequent among
children with hemophiliachildren with hemophilia
www.indiandentalacademy.comwww.indiandentalacademy.com
Prepatellar HematomaPrepatellar Hematoma
www.indiandentalacademy.comwww.indiandentalacademy.com
Hematoma after IM injectionHematoma after IM injection
www.indiandentalacademy.comwww.indiandentalacademy.com
Question 5.2000Question 5.2000
 An ambulance team brings a 6 year old girl in anAn ambulance team brings a 6 year old girl in an
MVA to the ED. She had been unrestrained in theMVA to the ED. She had been unrestrained in the
back seat. The girl is unresponsive on arrival and isback seat. The girl is unresponsive on arrival and is
bleeding profusely from a scalp wound. Herbleeding profusely from a scalp wound. Her
Glasgow Coma Scale is 3. What is the BEST initialGlasgow Coma Scale is 3. What is the BEST initial
step in evaluation and management?step in evaluation and management?
– A. Control profuse scalp bleedingA. Control profuse scalp bleeding
– B. Establish IV accessB. Establish IV access
– C. Order portable cervical spine radiographsC. Order portable cervical spine radiographs
– D. Remove all clothingD. Remove all clothing
– E. Secure an adequate airwayE. Secure an adequate airway
 Answer E.Answer E.
www.indiandentalacademy.comwww.indiandentalacademy.com
Teaching Points 5.2000Teaching Points 5.2000
 Priority during primary surveyPriority during primary survey
– AirwayAirway
 Failure to provideFailure to provide
oxygenated blood to theoxygenated blood to the
brain and other vital organsbrain and other vital organs
is the quickest killer of ais the quickest killer of a
trauma victimtrauma victim
– BreathingBreathing
– CirculationCirculation
– DisabilityDisability
– ExposureExposure
 Any victim of head trauma with aAny victim of head trauma with a
GCS of ___ or less should beGCS of ___ or less should be
intubated immediatelyintubated immediately
– 88
 X-rays for cervical fractures…X-rays for cervical fractures…
– AP, Lateral and odontoid viewsAP, Lateral and odontoid views
Dens of Axis C-2Atlas C-1
Odontoid Viewwww.indiandentalacademy.comwww.indiandentalacademy.com
Teaching Points 5.2000Teaching Points 5.2000
Glasgow Coma ScalesGlasgow Coma Scales
INFANTS & TODDLERS CHILDREN’S & ADULTS
EYE
OPENING
Spontaneous 4 EYE
OPENING
Spontaneous 4
To Voice 3 To Voice 3
To Pain 2 To Pain 2
None 1 None 1
BEST
VERBAL
RESPONSE
Smiles, Interacts 5 BEST
VERBAL
RESPONSE
Oriented 5
Consolable 4 Confused 4
Cries to Pain 3 Inappropriate Words 3
Moans to Pain 2 Incomprehensible Words 2
None 1 None 1
BEST
MOTOR
RESPONSE
Normal spontaneous movement 6 BEST
MOTOR
RESPONSE
Obeys Commands 6
Localizes Pain 5 Localizes Pain 5
Withdraws to Pain 4 Withdraws (Pain) 4
Abnormal Flexion 3 Flexion (Pain) 3
Abnormal Extension 2 Extension (Pain) 2
None 1 None 1www.indiandentalacademy.comwww.indiandentalacademy.com
Question 6.2000Question 6.2000
 A 2 week old presents with tachypnea, poorA 2 week old presents with tachypnea, poor
perfusion, gallop rhythm, diminished pulses, andperfusion, gallop rhythm, diminished pulses, and
hepatomegaly. ABG shows metabolic acidosis.hepatomegaly. ABG shows metabolic acidosis.
Echocardiography reveals critical AorticEchocardiography reveals critical Aortic
Stenosis. What intervention is most likely toStenosis. What intervention is most likely to
stabilize the infant’s condition?stabilize the infant’s condition?
– A. DobutamineA. Dobutamine
– B. EpinephrineB. Epinephrine
– C. Nitric OxideC. Nitric Oxide
– D. 100% oxygenD. 100% oxygen
– E. Prostaglandin E-1E. Prostaglandin E-1
 Answer E.Answer E.
www.indiandentalacademy.comwww.indiandentalacademy.com
Teaching Points 6.2000Teaching Points 6.2000
 Severe CHF and cardiogenic shock inSevere CHF and cardiogenic shock in
neonate, think LEFT SIDED LESIONSneonate, think LEFT SIDED LESIONS
– Hypoplastic left heartHypoplastic left heart
– Critical congenital aortic stenosisCritical congenital aortic stenosis
– Critical neonatal coarctation of aortaCritical neonatal coarctation of aorta
 As ductus arteriosus closes, earlyAs ductus arteriosus closes, early
compensatory RV contribution tocompensatory RV contribution to
systemic blood flow and perfusion tosystemic blood flow and perfusion to
kidneys and other organs is lostkidneys and other organs is lost
rapidly developing severe metabolicrapidly developing severe metabolic
acidosisacidosis  myocardial and organmyocardial and organ
dysfunctiondysfunction
 Therefore, need to keep duct OPENTherefore, need to keep duct OPEN
with PGEwith PGE11 (alprostadil) EVEN TO AN(alprostadil) EVEN TO AN
INFANT 1-2INFANT 1-2 MONTHSMONTHS OF AGEOF AGE
www.indiandentalacademy.comwww.indiandentalacademy.com
Teaching Points 6.2000Teaching Points 6.2000
 Inotropic agents such as dobutamine orInotropic agents such as dobutamine or
epinephrine may provide supportive therapy butepinephrine may provide supportive therapy but
will not be lifesavingwill not be lifesaving
 Nitric oxide is a pulmonary dilator used in PPHNitric oxide is a pulmonary dilator used in PPH
of newborn but is contraindicated in obstructiveof newborn but is contraindicated in obstructive
left sided heart lesionsleft sided heart lesions
 Serious hypoxia is NOT a problem for infantsSerious hypoxia is NOT a problem for infants
with obstructive left sided lesionswith obstructive left sided lesions
 Arterial pO2 may be normal initially in a sickArterial pO2 may be normal initially in a sick
neonateneonate
 As PDA is opened the O2 will drop due to rightAs PDA is opened the O2 will drop due to right
ventricular flow to the bodyventricular flow to the body
www.indiandentalacademy.comwww.indiandentalacademy.com
Teaching Points 6.2000Teaching Points 6.2000
 Attempts to keep O2 HIGH ARE DETRIMENTAL asAttempts to keep O2 HIGH ARE DETRIMENTAL as
high inspired oxygen levels causes powerfulhigh inspired oxygen levels causes powerful
PULMONARY ARTERIAL DILATION which “steals”PULMONARY ARTERIAL DILATION which “steals”
RV output to the pulmonary arteries and away fromRV output to the pulmonary arteries and away from
the bodythe body
 This “steal” factor NOT o2 induced ductal closure isThis “steal” factor NOT o2 induced ductal closure is
the primary risk associated with use ofthe primary risk associated with use of
supplemental O2 when PGE1 is used to keep ductsupplemental O2 when PGE1 is used to keep duct
openopen
 Some degree of pulmonary hypertension (i.e. small,Some degree of pulmonary hypertension (i.e. small,
tight, non-dilated arteries) is essential to promotetight, non-dilated arteries) is essential to promote
systemic flow to the peripheral circulation via thesystemic flow to the peripheral circulation via the
ductusductus
www.indiandentalacademy.comwww.indiandentalacademy.com
Teaching Points 6.2000Teaching Points 6.2000
 Hypoplastic Left HeartHypoplastic Left Heart
– 22ndnd
most common congenitalmost common congenital
cardiac lesion presenting in 1cardiac lesion presenting in 1stst
week of lifeweek of life
– Most common cause of deathMost common cause of death
from CHD in 1from CHD in 1stst
month of lifemonth of life
 What are characteristics?What are characteristics?
– Hypoplasia of left ventricleHypoplasia of left ventricle
– Hypoplasia of aortic rootHypoplasia of aortic root
– Aortic valve atresiaAortic valve atresia
– Critical mitral valve stenosis or atresiaCritical mitral valve stenosis or atresia
 Result is a reduction or eliminationResult is a reduction or elimination
of blood flow through left side ofof blood flow through left side of
heartheart
www.indiandentalacademy.comwww.indiandentalacademy.com
Hypoplastic Left HeartHypoplastic Left Heart
 Ineffective left side of heartIneffective left side of heart 
obligatory left to right shunt where?obligatory left to right shunt where?
– Atrial levelAtrial level
 And right to left shunting where?And right to left shunting where?
– Ductus arteriosusDuctus arteriosus
 Systemic flow isSystemic flow is completely ductalcompletely ductal
dependentdependent
 How do coronary arteries get fed?How do coronary arteries get fed?
– Retrograde coronary perfusionRetrograde coronary perfusion
 As PDA closes neonates becomeAs PDA closes neonates become
critically illcritically ill  CHF, cyanotic,CHF, cyanotic,
tachycardic, tachypneic, ralestachycardic, tachypneic, rales
(crackles) from pulmonary edema,(crackles) from pulmonary edema,
and hepatomegaly; poor peripheraland hepatomegaly; poor peripheral
pulses with vasoconstrictedpulses with vasoconstricted
extremities.extremities.
www.indiandentalacademy.comwww.indiandentalacademy.com
Hypoplastic Left HeartHypoplastic Left Heart
 Cardiac ExamCardiac Exam
– Loud S2 (Marks beginningLoud S2 (Marks beginning
of diastole and is due toof diastole and is due to
closure of semilunarclosure of semilunar
valves; primarily pulmonicvalves; primarily pulmonic
valve slamming againstvalve slamming against
increased blood volumeincreased blood volume
and pressure)and pressure)
– S3 (Early diastoleS3 (Early diastole
corresponds with end ofcorresponds with end of
first phase of rapidfirst phase of rapid
ventricular filling from atriaventricular filling from atria
and can be normal inand can be normal in
children and young adults)children and young adults)
www.indiandentalacademy.comwww.indiandentalacademy.com
Hypoplastic Left HeartHypoplastic Left Heart
 Treatment:Treatment:
– PGEPGE11 (alprostadil) immediately to maintain ductal(alprostadil) immediately to maintain ductal
dependent systemic blood flowdependent systemic blood flow
– Cardiac transplantation in the newborn period isCardiac transplantation in the newborn period is
primary treatment for HLHSprimary treatment for HLHS
 Scarcity of newborn organs available for transplantationScarcity of newborn organs available for transplantation
 Life-long need for anti-rejection therapyLife-long need for anti-rejection therapy
 Average life span of the transplanted heart is limitedAverage life span of the transplanted heart is limited
(currently less than 15 years).(currently less than 15 years).
– Most common treatment for HLHS is palliativeMost common treatment for HLHS is palliative
"staged reconstruction""staged reconstruction"
www.indiandentalacademy.comwww.indiandentalacademy.com
Hypoplastic Left HeartHypoplastic Left Heart
– 1. Norwood operation: performed 11. Norwood operation: performed 1stst
weekweek
 RV becomes the systemic or main ventricleRV becomes the systemic or main ventricle
pumping to the body.pumping to the body.
 ““Neoaorta” is made from part of the pulmonaryNeoaorta” is made from part of the pulmonary
artery and the original, tiny aorta, is enlarged toartery and the original, tiny aorta, is enlarged to
provide blood flow to the body.provide blood flow to the body.
 To provide blood flow to lungs, a small tube graft isTo provide blood flow to lungs, a small tube graft is
placed either from the left subclavian or leftplaced either from the left subclavian or left
innominate artery to the lung vessels (modifiedinnominate artery to the lung vessels (modified
Blalock-Taussig shunt: or from the RV to the lungBlalock-Taussig shunt: or from the RV to the lung
vessels (Sano modification).vessels (Sano modification).
www.indiandentalacademy.comwww.indiandentalacademy.com
Hypoplastic Left HeartHypoplastic Left Heart
– 2. Bi-directional Glenn procedure2. Bi-directional Glenn procedure
 3 to 6 months of age3 to 6 months of age
 SVC is taken off the heart and sewn directly to theSVC is taken off the heart and sewn directly to the
pulmonary arterypulmonary artery
– 3. Fontan operation3. Fontan operation
 2 or 3 years of age2 or 3 years of age
 IVC is connected directly the blood from theIVC is connected directly the blood from the
pulmonary arteriespulmonary arteries
www.indiandentalacademy.comwww.indiandentalacademy.com
Question 7.2000Question 7.2000
 During a routine visit, a 14 year old overweightDuring a routine visit, a 14 year old overweight
patient tells you he watches more than 25 hours ofpatient tells you he watches more than 25 hours of
TV per week. What is the most appropriate advice?TV per week. What is the most appropriate advice?
– A. Parents should limit TV to no more than 2 hours/dayA. Parents should limit TV to no more than 2 hours/day
– B. TV commercials have little impact on selection of toysB. TV commercials have little impact on selection of toys
and foodand food
– C. Average adolescent spends 15 hours/week watchingC. Average adolescent spends 15 hours/week watching
TVTV
– D. Unclear relationship between TV viewing and risk ofD. Unclear relationship between TV viewing and risk of
obesityobesity
– E. No relationship between TV viewing and snackingE. No relationship between TV viewing and snacking
 ANSWER AANSWER A
www.indiandentalacademy.comwww.indiandentalacademy.com
Teaching Points 7.2000Teaching Points 7.2000
 Children 2-11 22 h/weekChildren 2-11 22 h/week
 Adolescents 12-17 20 h/weekAdolescents 12-17 20 h/week
 Age 18, adolescent will have seen 200,000 violentAge 18, adolescent will have seen 200,000 violent
acts and 18,000 murders on TVacts and 18,000 murders on TV
 Encourage parents to “co-view” programs withEncourage parents to “co-view” programs with
childrenchildren
 Saturday morning commercials; 50% food, 33%Saturday morning commercials; 50% food, 33%
toystoys
 20 best selling toys are from TV shows based on20 best selling toys are from TV shows based on
the toysthe toys
 Increased TV viewing significant factor leading toIncreased TV viewing significant factor leading to
obesityobesity
 Increased TV viewing = increased snackingIncreased TV viewing = increased snacking
www.indiandentalacademy.comwww.indiandentalacademy.com
Deep Thoughts from RyanDeep Thoughts from Ryan
 ““Dad, how long doesDad, how long does
it take a booger toit take a booger to
form?”form?”
 Booger: Dried nasalBooger: Dried nasal
mucus; bogie inmucus; bogie in
EnglandEngland
 Medical Term:Medical Term:
– RhinolithsRhinoliths
www.indiandentalacademy.comwww.indiandentalacademy.com
Question 8.2000Question 8.2000
 A male infant born at 36 weeks gestation had a leftA male infant born at 36 weeks gestation had a left
testicle palpable in the inguinal canal. At 12 monthstesticle palpable in the inguinal canal. At 12 months
of age, the left testicle has failed to descend into theof age, the left testicle has failed to descend into the
scrotum. What is most appropriate care for thisscrotum. What is most appropriate care for this
infant?infant?
– A. Observation until 2 years of ageA. Observation until 2 years of age
– B. OrchiopexyB. Orchiopexy
– C. Radionuclide scan of left testicleC. Radionuclide scan of left testicle
– D. Treatment with human chorionic gonadotropinD. Treatment with human chorionic gonadotropin
– E. Treatment with testosteroneE. Treatment with testosterone
 ANSWER BANSWER B
www.indiandentalacademy.comwww.indiandentalacademy.com
Teaching Points 8.2000Teaching Points 8.2000
 What percent of preterm male infants have anWhat percent of preterm male infants have an
undescended testisundescended testis
– 30%30%
 Spontaneous descent into scrotum occurs…Spontaneous descent into scrotum occurs…
– over next 3-6 monthsover next 3-6 months
 What is the most reliable method to localize theWhat is the most reliable method to localize the
undescended testis?undescended testis?
– LaparoscopyLaparoscopy
 Orchiopexy, a surgery which places the testis intoOrchiopexy, a surgery which places the testis into
the scrotum is indicated for a testis that fails tothe scrotum is indicated for a testis that fails to
descend by…descend by…
– 6 months of age and is performed at 9-12 months of age6 months of age and is performed at 9-12 months of age
www.indiandentalacademy.comwww.indiandentalacademy.com
Teaching Points 8.2000Teaching Points 8.2000
 At 6-12 months of age histological changesAt 6-12 months of age histological changes
representing degeneration of the seminiferousrepresenting degeneration of the seminiferous
tubules occurstubules occurs decrease in quality and quantitydecrease in quality and quantity
of spermatogenesisof spermatogenesis which is progressivewhich is progressive
 The longer the testis remains in its improperThe longer the testis remains in its improper
location the greater the fertility impairmentlocation the greater the fertility impairment
 What percent of all testicular tumors occur inWhat percent of all testicular tumors occur in
patients with an undescended testiclepatients with an undescended testicle
– 10%10%
 Orchiopexy improves fertility but DOES NOTOrchiopexy improves fertility but DOES NOT
change malignancy riskchange malignancy risk
www.indiandentalacademy.comwww.indiandentalacademy.com
Teaching Points 8.2000Teaching Points 8.2000
 Relocated testis has aRelocated testis has a 35-48 times35-48 times greatergreater
risk of malignancy than normal testisrisk of malignancy than normal testis
 Does the normal testis also have anDoes the normal testis also have an
increased risk?increased risk?
– YES, just not as highYES, just not as high
 Counsel parents to:Counsel parents to:
– Seek immediate attention for acute testicularSeek immediate attention for acute testicular
pain due to risk of torsionpain due to risk of torsion
– Perform monthly examination of both testesPerform monthly examination of both testes
www.indiandentalacademy.comwww.indiandentalacademy.com
Teaching Points 8.2000Teaching Points 8.2000
 Retractile testiclesRetractile testicles
– Bilateral mostlyBilateral mostly
– Found in children >1 year of age, 5-6 yearFound in children >1 year of age, 5-6 year
olds strong reflexolds strong reflex
– Due to strong cremasteric reflexDue to strong cremasteric reflex
– ““Milk them” into scrotumMilk them” into scrotum
– Warm room, frog leg position can makeWarm room, frog leg position can make
examination easierexamination easier
www.indiandentalacademy.comwww.indiandentalacademy.com
Question 9.2000Question 9.2000
 A newborn female has an open neural tubeA newborn female has an open neural tube
defect, low set ears, VSD, and rib anddefect, low set ears, VSD, and rib and
vertebral column malformations. Which ofvertebral column malformations. Which of
the following MATERNAL conditions wasthe following MATERNAL conditions was
most likely present during pregnancy?most likely present during pregnancy?
 A. AlcoholismA. Alcoholism
 B. Diabetes mellitusB. Diabetes mellitus
 C. HypothyroidismC. Hypothyroidism
 D. Iodine deficiencyD. Iodine deficiency
 E. SyphilisE. Syphilis
– ANSWER: BANSWER: Bwww.indiandentalacademy.comwww.indiandentalacademy.com
Teaching Points 9.2000Teaching Points 9.2000
 Early prenatal deficits: malformations (3 foldEarly prenatal deficits: malformations (3 fold
increase), growth deficiency, stillbirthincrease), growth deficiency, stillbirth
 The worse the diabetic control the moreThe worse the diabetic control the more
severe the defectssevere the defects
 Cardiac: VSD, TGA, dextrocardiaCardiac: VSD, TGA, dextrocardia
 CNS: Anencephaly,CNS: Anencephaly, holoprosencephalyholoprosencephaly ,,
spina bifida, hydrocephalus,spina bifida, hydrocephalus, caudalcaudal
regression syndromeregression syndrome
 Rib defectsRib defects
www.indiandentalacademy.comwww.indiandentalacademy.com
Teaching Points 9.2000Teaching Points 9.2000
 Infants present with macrosomia due toInfants present with macrosomia due to
hyperinsulinemia and excessive glucosehyperinsulinemia and excessive glucose
availabilityavailability
 Both linear growth and weight are affectedBoth linear growth and weight are affected
 If mom has severe vascular disease, canIf mom has severe vascular disease, can
result in IUGRresult in IUGR
 Additional complications: Hyperbilirubinemia,Additional complications: Hyperbilirubinemia,
hypoglycemia, vascular thromboses,hypoglycemia, vascular thromboses,
respiratory distress, birth injury due torespiratory distress, birth injury due to
macrosomiamacrosomia
www.indiandentalacademy.comwww.indiandentalacademy.com
HoloprosencephalyHoloprosencephaly
 Caused by a failure of theCaused by a failure of the
embryo's forebrain to divide toembryo's forebrain to divide to
form bilateral cerebralform bilateral cerebral
hemispheres causing defects inhemispheres causing defects in
face development face and inface development face and in
brain structure and function.brain structure and function.
 The "alobar" form ofThe "alobar" form of
holoprosencephaly is shown hereholoprosencephaly is shown here
in which there is a single largein which there is a single large
ventricle, because there is noventricle, because there is no
attempt to form separate cerebralattempt to form separate cerebral
hemispheres.hemispheres.
 May be associated with trisomy 13May be associated with trisomy 13
and rarely in association withand rarely in association with
maternal diabetes mellitus.maternal diabetes mellitus.
www.indiandentalacademy.comwww.indiandentalacademy.com
Spina BifidaSpina Bifida
www.indiandentalacademy.comwww.indiandentalacademy.com
Spina BifidaSpina Bifida
www.indiandentalacademy.comwww.indiandentalacademy.com
Teaching Points 9.2000Teaching Points 9.2000
 Fetal alcohol syndromeFetal alcohol syndrome
– What is the most common teratogen to which fetusesWhat is the most common teratogen to which fetuses
are exposed?are exposed?
 AlcoholAlcohol
– Which U.S. population has the highest incidence ofWhich U.S. population has the highest incidence of
children with FAS?children with FAS?
 Native AmericansNative Americans
– What is the incidence in U.S. (excluding NativeWhat is the incidence in U.S. (excluding Native
Americans)Americans)
 1:10001:1000
– Affects 40% of children in women who drink moreAffects 40% of children in women who drink more
than 4-6 drinks per daythan 4-6 drinks per day
www.indiandentalacademy.comwww.indiandentalacademy.com
www.indiandentalacademy.comwww.indiandentalacademy.com
Fetal Alcohol SyndromeFetal Alcohol Syndrome
www.indiandentalacademy.comwww.indiandentalacademy.com
Fetal Alcohol SyndromeFetal Alcohol Syndrome
www.indiandentalacademy.comwww.indiandentalacademy.com
Fetal Alcohol SyndromeFetal Alcohol Syndrome
www.indiandentalacademy.comwww.indiandentalacademy.com
Epicanthal FoldsEpicanthal Folds
www.indiandentalacademy.comwww.indiandentalacademy.com
Teaching Points 9.2000Teaching Points 9.2000
 MaternalMaternal
hypothyroidism hashypothyroidism has
little effect on fetuslittle effect on fetus
which produces itswhich produces its
own thyroid hormoneown thyroid hormone
 Women withWomen with
untreateduntreated
hypothyroidism alsohypothyroidism also
give birth togive birth to
NORMAL babiesNORMAL babies
www.indiandentalacademy.comwww.indiandentalacademy.com
Teaching Points 9.2000Teaching Points 9.2000
 Maternal iodine deficiency (rare in developedMaternal iodine deficiency (rare in developed
countries)countries)  fetal deficiencyfetal deficiency
– GoiterGoiter
– Mental retardationMental retardation
– Slightly increased head size due to…Slightly increased head size due to…
 Myxedema [“hard” edema due to increased mucinsMyxedema [“hard” edema due to increased mucins
(proteoglycans) in the fluid] of subcutaneous tissues of(proteoglycans) in the fluid] of subcutaneous tissues of
the brain}the brain}
– Hyperbilirubinemia due to…Hyperbilirubinemia due to…
 Delayed maturation of glucuronide conjugationDelayed maturation of glucuronide conjugation
– Feeding difficulties (choking spells, lack ofFeeding difficulties (choking spells, lack of
interest, somnolence, sluggishness)interest, somnolence, sluggishness)www.indiandentalacademy.comwww.indiandentalacademy.com
Teaching Points 9.2000Teaching Points 9.2000
– Respiratory difficulties due to…Respiratory difficulties due to…
 Large tongue, apneic episodes, noisy respirations, nasalLarge tongue, apneic episodes, noisy respirations, nasal
obstructionobstruction
– Retarded bone growthRetarded bone growth
– ConstipationConstipation
– Umbilical hernia with large abdomenUmbilical hernia with large abdomen
– Hypothermia and cold and mottled skinHypothermia and cold and mottled skin
– Slow pulseSlow pulse
– Genital and extremity edemaGenital and extremity edema
– Pericardial effusion, murmur, cardiomegalyPericardial effusion, murmur, cardiomegaly
 Prompt treatment with iodine necessary toPrompt treatment with iodine necessary to
prevent mental retardationprevent mental retardation
www.indiandentalacademy.comwww.indiandentalacademy.com
Teaching Points 9.2000Teaching Points 9.2000
 Maternal syphilisMaternal syphilis
– Fetus affected by…Fetus affected by…
 Transplacental transmissionTransplacental transmission Treponema pallidumTreponema pallidum
– Syphilis in untreated women can be transmitted toSyphilis in untreated women can be transmitted to
fetus at any time, fetal transfer most common duringfetus at any time, fetal transfer most common during
11stst
year of maternal infectionyear of maternal infection
– 2/3 of live-born neonates with congenital syphilis are2/3 of live-born neonates with congenital syphilis are
asymptomatic at birth.asymptomatic at birth.
– Overt infection can manifest in the fetus, the newborn,Overt infection can manifest in the fetus, the newborn,
or later in childhood.or later in childhood.
– Clinical manifestations after birth are dividedClinical manifestations after birth are divided
arbitrarily into early ( 2 years of age) and late (>2arbitrarily into early ( 2 years of age) and late (>2
years of age)years of age)
www.indiandentalacademy.comwww.indiandentalacademy.com
Teaching Points 9.2000Teaching Points 9.2000
 Fetal manifestations of SyphilisFetal manifestations of Syphilis
– StillbirthStillbirth
– Neonatal deathNeonatal death
– Overt infection at birth, such as hydropsOvert infection at birth, such as hydrops
fetalis (abnormal accumulation serous fluid infetalis (abnormal accumulation serous fluid in
fetal tissues)fetal tissues)
– Intrauterine death in 25% of affected infants,Intrauterine death in 25% of affected infants,
with perinatal mortality in an additional 25-with perinatal mortality in an additional 25-
30%, if untreated30%, if untreated
www.indiandentalacademy.comwww.indiandentalacademy.com
Teaching Points 9.2000Teaching Points 9.2000
 Early congenital manifestations ofEarly congenital manifestations of
SyphilisSyphilis
– Quite variable, appear within the first 5 weeks of lifeQuite variable, appear within the first 5 weeks of life
– Cutaneous lesions frequently occur on the palms andCutaneous lesions frequently occur on the palms and
soles; if ulcerative in nature, they are highlysoles; if ulcerative in nature, they are highly
contagiouscontagious
– HepatosplenomegalyHepatosplenomegaly
– JaundiceJaundice
– AnemiaAnemia
– Snuffles (obstructed nasal respiration in newborn)Snuffles (obstructed nasal respiration in newborn)
– Metaphyseal dystrophy and periostitis often are notedMetaphyseal dystrophy and periostitis often are noted
on radiographs at birthon radiographs at birth
www.indiandentalacademy.comwww.indiandentalacademy.com
Teaching Points 9.2000Teaching Points 9.2000
 Late congenital manifestationsLate congenital manifestations
– Develop from scarring related to earlyDevelop from scarring related to early
infection but can be prevented by treatment ofinfection but can be prevented by treatment of
the infant within the first three months of birththe infant within the first three months of birth
– Late findings include frontal bossing, shortLate findings include frontal bossing, short
maxilla, high palatal arch, Hutchinson triadmaxilla, high palatal arch, Hutchinson triad
(Hutchinson teeth [blunted upper incisors],(Hutchinson teeth [blunted upper incisors],
interstitial keratitis, and eighth nerveinterstitial keratitis, and eighth nerve
deafness), saddle nose, and perioral fissuresdeafness), saddle nose, and perioral fissures
(rhagades)(rhagades)
www.indiandentalacademy.comwww.indiandentalacademy.com
Saddle Nose
deformity in
congenital syphilis
www.indiandentalacademy.comwww.indiandentalacademy.com
Hutchinson TeethHutchinson Teeth
www.indiandentalacademy.comwww.indiandentalacademy.com
Question 10.2000Question 10.2000
 A 5 year old male is hospitalized in January withA 5 year old male is hospitalized in January with
fever and seizures. LP reveals clear CSF with 47fever and seizures. LP reveals clear CSF with 47
WBCs/mmWBCs/mm33
all of which are lymphocytes. On PEall of which are lymphocytes. On PE
he appears obtunded but arouses with painfulhe appears obtunded but arouses with painful
stimuli. Neurologic exam reveals no focal findings.stimuli. Neurologic exam reveals no focal findings.
Which diagnostic test is most likely to reveal thisWhich diagnostic test is most likely to reveal this
child’s illness?child’s illness?
– A. Bacterial culture of CSFA. Bacterial culture of CSF
– B. PCR test of CSF for HSVB. PCR test of CSF for HSV
– C.C. Streptococcus pneumoniaeStreptococcus pneumoniae bacterial antigen test ofbacterial antigen test of
CSFCSF
– D. Viral culture of CSFD. Viral culture of CSF
– E. Viral culture of nasopharyngeal and rectal swabsE. Viral culture of nasopharyngeal and rectal swabs
 ANSWER BANSWER B
www.indiandentalacademy.comwww.indiandentalacademy.com
Learning Points 10.2000Learning Points 10.2000
 Exam findings are consistent with encephalitisExam findings are consistent with encephalitis
 CSF findings are consistent with a viral etiologyCSF findings are consistent with a viral etiology
 Most likely cause is a sporadic case of herpesMost likely cause is a sporadic case of herpes
simplex virussimplex virus
 Viral cultures of CSF for HSV are RARELYViral cultures of CSF for HSV are RARELY
positive beyond neonatal periodpositive beyond neonatal period
 HSV is not found in cultures of sites outside theHSV is not found in cultures of sites outside the
CNSCNS
www.indiandentalacademy.comwww.indiandentalacademy.com
Learning Points 10.2000Learning Points 10.2000
 What is the treatment of choice for herpesWhat is the treatment of choice for herpes
encephalitis in a child?encephalitis in a child?
– IV Acyclovir 10 mg/kg/dose q8 for 14-21 daysIV Acyclovir 10 mg/kg/dose q8 for 14-21 days
 What is the treatment of choice forWhat is the treatment of choice for
neonatal herpes encephalitis?neonatal herpes encephalitis?
– IV Acyclovir 20 mg/kg/dose q8 for 14-21 daysIV Acyclovir 20 mg/kg/dose q8 for 14-21 days
www.indiandentalacademy.comwww.indiandentalacademy.com
Question 11.2000Question 11.2000
 An 18 year old girl presents for a RV. ExamAn 18 year old girl presents for a RV. Exam
reveals tanner stage 5 breasts with 2x2 cmreveals tanner stage 5 breasts with 2x2 cm
nontender, smooth, mobile mass in left breast. Itnontender, smooth, mobile mass in left breast. It
is located at the upper outer quadrant. Theis located at the upper outer quadrant. The
overlying skin is normal and there is no historyoverlying skin is normal and there is no history
of nipple discharge. Family history is negative forof nipple discharge. Family history is negative for
breast cancer. What is the most appropriate nextbreast cancer. What is the most appropriate next
step in evaluation of this breast mass?step in evaluation of this breast mass?
– A. Excisional biopsyA. Excisional biopsy
– B. Fine-needle biopsyB. Fine-needle biopsy
– C. MammographyC. Mammography
– D. Reassurance and reevaluation in 4-6 monthsD. Reassurance and reevaluation in 4-6 months
– E. Referral for surgical consultationE. Referral for surgical consultation
 ANSWER DANSWER D
www.indiandentalacademy.comwww.indiandentalacademy.com
Teaching Points 11.2000Teaching Points 11.2000
 What is the diagnosis?What is the diagnosis?
– FibroadenomaFibroadenoma
 Fibroadenomas are most common breastFibroadenomas are most common breast
lesion in an adolescent femalelesion in an adolescent female
 Differential diagnosis breast masses:Differential diagnosis breast masses:
– Fibrocystic changesFibrocystic changes
– CystsCysts
– AbscessesAbscesses
– Rarely, malignancyRarely, malignancy
www.indiandentalacademy.comwww.indiandentalacademy.com
Teaching Points 11.2000Teaching Points 11.2000
 Without family history of breast cancer in a 1Without family history of breast cancer in a 1stst
or 2or 2ndnd
degreedegree
relative, primary breast cancer is extremely rare inrelative, primary breast cancer is extremely rare in
adolescentsadolescents
 Fewer than 1/1,000 of adolescent breast masses areFewer than 1/1,000 of adolescent breast masses are
malignantmalignant
 Metastatic lesions from…Metastatic lesions from…
– Rhabdomyosarcoma, neuroblastoma, and lymphoma can occur, butRhabdomyosarcoma, neuroblastoma, and lymphoma can occur, but
rare.rare.
 If anxiety high can begin with what test…If anxiety high can begin with what test…
– Ultrasound to differentiate between cystic and solid massesUltrasound to differentiate between cystic and solid masses
 FNA or biopsy can be done in adolescent with unusually largeFNA or biopsy can be done in adolescent with unusually large
or atypical mass or who is highly anxious about malignantor atypical mass or who is highly anxious about malignant
potentialpotential
 Mammography never is indicated fir evaluation of a breastMammography never is indicated fir evaluation of a breast
mass in teenagers because the dense breast tissue makesmass in teenagers because the dense breast tissue makes
study difficult to interpretstudy difficult to interpret
www.indiandentalacademy.comwww.indiandentalacademy.com
Question 12.2000Question 12.2000
 A 15 year old girl complains of dysuria andA 15 year old girl complains of dysuria and
abdominal pain for 2 days. She denies nausea,abdominal pain for 2 days. She denies nausea,
vomiting, flank pain and vaginal discharge.vomiting, flank pain and vaginal discharge.
Menarche occurred to years ago and mensesMenarche occurred to years ago and menses
have been irregular. PE reveals SMR stage 4have been irregular. PE reveals SMR stage 4
genitalia, mild suprapubic tenderness, andgenitalia, mild suprapubic tenderness, and
otherwise normal findings. What is the mostotherwise normal findings. What is the most
likely diagnosis?likely diagnosis?
– A. Bacterial vaginosisA. Bacterial vaginosis
– B. Candidal vulvovaginitisB. Candidal vulvovaginitis
– C.C. ChlamydiaChlamydia urethritisurethritis
– D. Pelvic inflammatory diseaseD. Pelvic inflammatory disease
– E. Urinary tract infectionE. Urinary tract infection
 ANSWER E.ANSWER E.
www.indiandentalacademy.comwww.indiandentalacademy.com
Teaching Points 12.2000Teaching Points 12.2000
 Bacterial vaginosisBacterial vaginosis
– Grayish discharge with pH >4.5Grayish discharge with pH >4.5
– Saline wet mount reveals >20% clue cells and an absenceSaline wet mount reveals >20% clue cells and an absence
of lactobacilliof lactobacilli
– An isolatedAn isolated Gardenerella vaginalisGardenerella vaginalis infection does notinfection does not
cause dysuria and lower abdominal paincause dysuria and lower abdominal pain
 Candidal vaginosisCandidal vaginosis
– Whitish dischargeWhitish discharge
 Chlamydia trachomatisChlamydia trachomatis
– Causes dysuria if urethra involvedCauses dysuria if urethra involved
– No abdominal pain UNLESS PID presentNo abdominal pain UNLESS PID present
– PID symptoms: lower abdominal pain, cervical motionPID symptoms: lower abdominal pain, cervical motion
tenderness, adnexal tenderness; fever, vomiting, diarrhea,tenderness, adnexal tenderness; fever, vomiting, diarrhea,
irregular vaginal bleeding, increased vaginal dischargeirregular vaginal bleeding, increased vaginal discharge
www.indiandentalacademy.comwww.indiandentalacademy.com
Clue CellsClue Cells
 Vaginal epithelial cells that appear
fuzzy without distinct edges under
a microscope due to being coated
with G. vaginalis
 Clue cells flake off of the walls of
the vagina and are found in
vaginal smear.
 Chemicals released by the
bacteria that cause bacterial
vaginosis (BV) may damage
vaginal wall cells, causing them to
flake off in greater numbers than
usual.
 When bacterial vaginosis is
present, more than 20% of the
sample vaginal epithelial cells are
clue cells.
www.indiandentalacademy.comwww.indiandentalacademy.com
Clue CellsClue Cells
www.indiandentalacademy.comwww.indiandentalacademy.com
Question 13.2000Question 13.2000
 A 3 year old girl comes to the ER withA 3 year old girl comes to the ER with
temperature of 103.1temperature of 103.1oo
F and acute onsetF and acute onset
diarrhea. Stool is guaiac positive withdiarrhea. Stool is guaiac positive with
leukocytes. There is no history of foreign travelleukocytes. There is no history of foreign travel
and the child has not received antibioticsand the child has not received antibiotics
recently. What is the most likely organism?recently. What is the most likely organism?
– A.A. Clostridium difficileClostridium difficile
– B.B. Giardia lambliaGiardia lamblia
– C. RotavirusC. Rotavirus
– D.D. Salmonella enteritidisSalmonella enteritidis
– E.E. Vibrio choleraeVibrio cholerae
 ANSWER D.ANSWER D.
www.indiandentalacademy.comwww.indiandentalacademy.com
Teaching Points 13.2000Teaching Points 13.2000
 Viral diarrheaViral diarrhea
– Low-grade fever, vomiting, large, loose wateryLow-grade fever, vomiting, large, loose watery
stoolsstools
– Most common cause…Most common cause…
 RotavirusRotavirus
– Season predominance…Season predominance…
 Winter in United StatesWinter in United States
www.indiandentalacademy.comwww.indiandentalacademy.com
Teaching Points 13.2000Teaching Points 13.2000
 Bacterial DiarrheaBacterial Diarrhea
– High fevers, small frequent stools with mucous orHigh fevers, small frequent stools with mucous or
bloodblood
– What can happen if you treatWhat can happen if you treat SalmonellaSalmonella infectioninfection
with antibiotics?with antibiotics?
 Prolong the carrier stateProlong the carrier state
– Which organism is seen after antibiotics?Which organism is seen after antibiotics?
 Clostridium difficileClostridium difficile
– What organism is seen after ingestion of seafood orWhat organism is seen after ingestion of seafood or
water?water?
 VibrioVibrio cholerae
– Giardia lambliaGiardia lamblia results in chronic diarrhea withresults in chronic diarrhea with
malabsorptionmalabsorption
www.indiandentalacademy.comwww.indiandentalacademy.com
Question 14.1000Question 14.1000
 You are evaluating a 4 week old boy for tearing ofYou are evaluating a 4 week old boy for tearing of
the right eye that has worsened over the past week.the right eye that has worsened over the past week.
Physical exam reveals slight tearing but noPhysical exam reveals slight tearing but no
evidence of purulent exudate or conjunctivalevidence of purulent exudate or conjunctival
erythema. All other findings are normal. The MOSTerythema. All other findings are normal. The MOST
appropriate initial management is…appropriate initial management is…
– A. Administration of amoxicillinA. Administration of amoxicillin
– B. Endoscopic dacrocystorhinostomyB. Endoscopic dacrocystorhinostomy
– C. Instillation of silver nitrate in the eyesC. Instillation of silver nitrate in the eyes
– D. Observation with intermittent massage of the ductD. Observation with intermittent massage of the duct
– E. Surgical dilation of the nasolacrimal ductE. Surgical dilation of the nasolacrimal duct
 ANSWER DANSWER D
www.indiandentalacademy.comwww.indiandentalacademy.com
Teaching Points 14.2000Teaching Points 14.2000
 Congenital nasolacrimal duct obstruction is mostCongenital nasolacrimal duct obstruction is most
common abnormality of infant lacrimal system, 5%common abnormality of infant lacrimal system, 5%
affectedaffected
 What percent of those affected have bilateralWhat percent of those affected have bilateral
obstruction?obstruction?
– 30%30%
 Obstruction is usually found where?Obstruction is usually found where?
– Distal endDistal end
 Pertinent negatives on examPertinent negatives on exam
– Conjunctival inflammation, photophobia, blepharospasm,Conjunctival inflammation, photophobia, blepharospasm,
corneal cloudingcorneal clouding
www.indiandentalacademy.comwww.indiandentalacademy.com
Question 68.1999Question 68.1999
A previously health 1-year-old infant who weighs 10 kg presentsA previously health 1-year-old infant who weighs 10 kg presents
to your office with a fever of 39° C (102.2° F). Her mother isto your office with a fever of 39° C (102.2° F). Her mother is
very concerned about the child’s intake and asks for guidancevery concerned about the child’s intake and asks for guidance
regarding caloric requirements during this illness.regarding caloric requirements during this illness.
Of the following, the best estimate of the child’s caloricOf the following, the best estimate of the child’s caloric
requirements at this time isrequirements at this time is
A.A.500 kcal/d plus 500 kcal due to the fever500 kcal/d plus 500 kcal due to the fever
B.B.1,000 kcal/d plus 250 kcal due to the fever1,000 kcal/d plus 250 kcal due to the fever
C.C.1,500 kcal/d1,500 kcal/d
D.D.1,500 kcal/d plus 250 kcal due to the fever1,500 kcal/d plus 250 kcal due to the fever
E.E.2,000 kcal/d2,000 kcal/d
Answer: BAnswer: Bwww.indiandentalacademy.comwww.indiandentalacademy.com
Question (68.1999)Question (68.1999)
 What is the most effective indicator of whether aWhat is the most effective indicator of whether a
child is getting enough calories?child is getting enough calories?
– Growth ChartGrowth Chart
 Name 5 factors that affect a child’s energy (calorie)Name 5 factors that affect a child’s energy (calorie)
requirements.requirements.
– Basal metabolism calories (Maintenance at rest andBasal metabolism calories (Maintenance at rest and
fasting)fasting)
– Growing caloriesGrowing calories
– Exercise caloriesExercise calories
– Eating caloriesEating calories
– Hypermetabolic states et al (See Table)Hypermetabolic states et al (See Table)
www.indiandentalacademy.comwww.indiandentalacademy.com
Calculation of CaloricCalculation of Caloric
Requirements Based on BodyRequirements Based on Body
Weight (68.1999)Weight (68.1999)
Body WeightBody Weight Caloric RequirementCaloric Requirement
≤≤ 10 kg10 kg 100 kcal/kg100 kcal/kg
11 to 20 kg11 to 20 kg 1,000 kcal + 501,000 kcal + 50
kcal/kg for each kgkcal/kg for each kg
above 10 kgabove 10 kg
≥≥ 20 kg20 kg 1,500 kcal + 201,500 kcal + 20
kcal/kg for each kgkcal/kg for each kg
above 20 kgabove 20 kgwww.indiandentalacademy.comwww.indiandentalacademy.com
Calculating Energy Lost in FeverCalculating Energy Lost in Fever
(68.1999)(68.1999)
 FormulaFormula
– For each degree above 37For each degree above 37oo
C, multiply by 12%C, multiply by 12%
of maintenance requirements per degree. Thenof maintenance requirements per degree. Then
add to original maintenance requirements foradd to original maintenance requirements for
total energy needs.total energy needs.
– Example: 10 kg infant with 39Example: 10 kg infant with 39oo
C temperature.C temperature.
Calculate total caloric requirements.Calculate total caloric requirements.
 10 kg X 100kcal/kg= 1,000 kcal maintenance needs10 kg X 100kcal/kg= 1,000 kcal maintenance needs
 22oo
X 0.12(1,000kcal) = 240 kcalX 0.12(1,000kcal) = 240 kcal
 Total Needs = 1,240 kcalTotal Needs = 1,240 kcal
www.indiandentalacademy.comwww.indiandentalacademy.com
Question 69Question 69
A 5-year-old boy is brought to the emergency department afterA 5-year-old boy is brought to the emergency department after
having been struck by an automobile. Physical examination revealshaving been struck by an automobile. Physical examination reveals
facial abrasions, abdominal tenderness, and gross blood at thefacial abrasions, abdominal tenderness, and gross blood at the
urethral meatus. Pelvic radiography reveals a left-sided fracture ofurethral meatus. Pelvic radiography reveals a left-sided fracture of
the superior pubic rami.the superior pubic rami.
Of the following, the best procedure for INITIAL evaluation of theOf the following, the best procedure for INITIAL evaluation of the
urinary tract in this patient isurinary tract in this patient is
A.A.Bladder catheterization via the urethraBladder catheterization via the urethra
B.B.Computed tomography of the abdomenComputed tomography of the abdomen
C.C.Intravenous pyelographyIntravenous pyelography
D.D.Renal untrasonographyRenal untrasonography
E.E.Retrograde urethrographyRetrograde urethrography
Answer: EAnswer: Ewww.indiandentalacademy.comwww.indiandentalacademy.com
Additional Caloric Requirements in SelectedAdditional Caloric Requirements in Selected
Medical and Surgical Conditions (68.1999)Medical and Surgical Conditions (68.1999)
ConditionCondition Maximum Amount ofMaximum Amount of
Additional CaloriesAdditional Calories
RequiredRequired
Hypermetabolic stateHypermetabolic state
due to recent surgerydue to recent surgery
20%20%
Multiple traumaMultiple trauma 25%25%
Severe infectionSevere infection 50%50%
Third-degree burnsThird-degree burns
(<20% of body)(<20% of body)
100%100%
UncomplicatedUncomplicated
starvationstarvation
50%50%
www.indiandentalacademy.comwww.indiandentalacademy.com
Question (69.1999)Question (69.1999)
 In children who sustain multiple injuries in aIn children who sustain multiple injuries in a
vehicular crash what are the top two “systems”vehicular crash what are the top two “systems”
that are involved?that are involved?
– Central Nervous System #1Central Nervous System #1
– Genitourinary System #2Genitourinary System #2
 Blood at tip of penis suggests urethral injury.Blood at tip of penis suggests urethral injury.
 Injury to the prostatomembranous portion ofInjury to the prostatomembranous portion of
urethra associated with pelvic fracture is mosturethra associated with pelvic fracture is most
common.common.
 Isolated urethral injury in female isIsolated urethral injury in female is
UNCOMMON.UNCOMMON.
www.indiandentalacademy.comwww.indiandentalacademy.com
Answer Explanations (69.1999)Answer Explanations (69.1999)
 Retrograde urethrography (E.) -CatheterRetrograde urethrography (E.) -Catheter
placed just inside urethral meatus and dyeplaced just inside urethral meatus and dye
insertedinserted
 Routine catheterization (A.) isRoutine catheterization (A.) is
contraindicated with blood at urethralcontraindicated with blood at urethral
meatus because…meatus because…
– Procedure might convert a PARTIAL TEAR ofProcedure might convert a PARTIAL TEAR of
meatus into a COMPLETE TRANSECTIONmeatus into a COMPLETE TRANSECTION
www.indiandentalacademy.comwww.indiandentalacademy.com
Answer Explanations (69.1999)Answer Explanations (69.1999)
 Abdominal CT would be helpful for…Abdominal CT would be helpful for…
– Evaluating intrabdominal injuriesEvaluating intrabdominal injuries
 Splenic injurySplenic injury
 Liver injuryLiver injury
 Renal injuryRenal injury
 Intravenous pyelography and Renal USIntravenous pyelography and Renal US
will evaluate renal pathology but notwill evaluate renal pathology but not
evaluate suspected urethral injuriesevaluate suspected urethral injuries
www.indiandentalacademy.comwww.indiandentalacademy.com
Apgar ScoreApgar Score
(Virginia Apgar 1953)(Virginia Apgar 1953)
Component ofComponent of
Apgar ScoreApgar Score
Score (Check at 1 and 5 minutes)Score (Check at 1 and 5 minutes)
00 11 22
Heart RateHeart Rate NoneNone <100<100
beats/minbeats/min
>100>100
beats/minbeats/min
RespiratoryRespiratory
EffortEffort
NoneNone Weak cryWeak cry Lusty cryLusty cry
Muscle ToneMuscle Tone LimpLimp Some flexionSome flexion Well flexedWell flexed
ReflexReflex
IrratibilityIrratibility
No responseNo response Some motionSome motion VigorousVigorous
responseresponse
ColorColor Uniformly BlueUniformly Blue AcrocyanosisAcrocyanosis CompletelyCompletely
pinkpink
www.indiandentalacademy.comwww.indiandentalacademy.com
Question 70Question 70
You are attending the emergency delivery by cesarean section of aYou are attending the emergency delivery by cesarean section of a
primiparous woman. The gestation was complicated by pregnancy-primiparous woman. The gestation was complicated by pregnancy-
induced hypertension. Deep variable fetal heart rate decelerationsinduced hypertension. Deep variable fetal heart rate decelerations
were noted during labor. At delivery, the infant is acrocyanotic withwere noted during labor. At delivery, the infant is acrocyanotic with
poor tone; spontaneous movement and minimal respiratory effort arepoor tone; spontaneous movement and minimal respiratory effort are
present.present.
Of the following, your INITIAL management is toOf the following, your INITIAL management is to
A.A.Ascertain the heart rate and assign a 1-minute Apgar scoreAscertain the heart rate and assign a 1-minute Apgar score
B.B.Begin tactile stimulation and provide blow-blow oxygenBegin tactile stimulation and provide blow-blow oxygen
supplementationsupplementation
C.C.Dry all skin surfaces and clear the oropharynxDry all skin surfaces and clear the oropharynx
D.D.Initiate bag-mask ventilationInitiate bag-mask ventilation
E.E.Insert an umbilical catheter and administer naloxoneInsert an umbilical catheter and administer naloxone
Answer: CAnswer: Cwww.indiandentalacademy.comwww.indiandentalacademy.com
Learning Points (70.1999)Learning Points (70.1999)
 The body and head of an infant areThe body and head of an infant are
immediately dried with a prewarmed towelimmediately dried with a prewarmed towel
to remove…to remove…
– Amniotic fluidAmniotic fluid
 And to prevent…And to prevent…
– Evaporative heat loss which could lead toEvaporative heat loss which could lead to
hypothermiahypothermia
 Also provides gentle stimulation to infantAlso provides gentle stimulation to infant
www.indiandentalacademy.comwww.indiandentalacademy.com
Learning Points (70.1999)Learning Points (70.1999)
 When suctioning, mouth or nose first and why?When suctioning, mouth or nose first and why?
– Mouth to ensure nothing in oropharynx that could beMouth to ensure nothing in oropharynx that could be
aspirated as we are dealing with obligate noseaspirated as we are dealing with obligate nose
breathersbreathers
 Suctioning also provides tactile stimulationSuctioning also provides tactile stimulation
 Score of 7 or more indicates…Score of 7 or more indicates…
– WELL NEWBORNWELL NEWBORN  ROUTINE CARE ANDROUTINE CARE AND
OBSERVATIONOBSERVATION
 Score of 4-6 indicates…Score of 4-6 indicates…
– MILD TO MODERATE DEPRESSION, NEED MOREMILD TO MODERATE DEPRESSION, NEED MORE
INTERVENTION (Blowby, stimulation etc.)INTERVENTION (Blowby, stimulation etc.)
 Score 3 or less…Score 3 or less…
– SEVERE DEPRESSION, NEED TO INTUBATE, CVSEVERE DEPRESSION, NEED TO INTUBATE, CV
SUPPORT, BAG-MASK VENTILATIONSUPPORT, BAG-MASK VENTILATIONwww.indiandentalacademy.comwww.indiandentalacademy.com
Learning Points (70.1999)Learning Points (70.1999)
 When do you keep on taking ApgarWhen do you keep on taking Apgar
scores?scores?
– When 5 minute is 6 or lessWhen 5 minute is 6 or less
 Additional scores should be assignedAdditional scores should be assigned
every 5 minutes for up to 20 minutes orevery 5 minutes for up to 20 minutes or
until two consecutive scores of 7+ areuntil two consecutive scores of 7+ are
obtainedobtained
www.indiandentalacademy.comwww.indiandentalacademy.com
Question 71Question 71
A 1-year-old boy has been treated with a low-phenylalanine diet forA 1-year-old boy has been treated with a low-phenylalanine diet for
the past year after having been identified in infancy as havingthe past year after having been identified in infancy as having
phenylketonuria. Despite appropriate dietary restriction ofphenylketonuria. Despite appropriate dietary restriction of
phenylalanine, he has developed neurologic symptoms.phenylalanine, he has developed neurologic symptoms.
Of the following, this child is MOST likely to be deficient inOf the following, this child is MOST likely to be deficient in
A.A.BiotinBiotin
B.B.CobalaminCobalamin
C.C.CarnitineCarnitine
D.D.TegrahydrobiopterinTegrahydrobiopterin
E.E.ThiamineThiamine
Answer: DAnswer: D
www.indiandentalacademy.comwww.indiandentalacademy.com
Learning Points (71.1999)Learning Points (71.1999)
 PKU diagnosed by hyperphenylalaninemia inPKU diagnosed by hyperphenylalaninemia in
blood sample taken at 48 hours of ageblood sample taken at 48 hours of age
 Two types:Two types:
– Classical PKU has deficiency inClassical PKU has deficiency in phenylalaninephenylalanine
hydroxylasehydroxylase
– Deficiency of enzyme cofactor tetrahydrobiopterinDeficiency of enzyme cofactor tetrahydrobiopterin
(1-3% patients with hyperphenylalaninemia)(1-3% patients with hyperphenylalaninemia)
 Involved in hydroxylation reactions for tryptophan andInvolved in hydroxylation reactions for tryptophan and
tyrosinetyrosine  OH-tryptophan and L-DOPAOH-tryptophan and L-DOPA
(Neurotransmitters)(Neurotransmitters)
 Thus, phenylalanine restriction by itself in presence ofThus, phenylalanine restriction by itself in presence of
cofactor deficiency WILL NOT prevent neurologicalcofactor deficiency WILL NOT prevent neurological
damagedamage
www.indiandentalacademy.comwww.indiandentalacademy.com
Learning Points (71.1999)Learning Points (71.1999)
 How do you diagnose tetrahydrobiopterinHow do you diagnose tetrahydrobiopterin
deficiency?deficiency?
– Caused by recycling or synthesis defectsCaused by recycling or synthesis defects
– Measure pterin metabolites found in urineMeasure pterin metabolites found in urine
– Can also diagnose by a reduction inCan also diagnose by a reduction in
phenylalanine after IV or oral load ofphenylalanine after IV or oral load of
tetrahydrobiopterintetrahydrobiopterin
www.indiandentalacademy.comwww.indiandentalacademy.com
Learning Points (71.1999)Learning Points (71.1999)
 How do you treat tetrahydrobiopterinHow do you treat tetrahydrobiopterin
deficiency?deficiency?
– Administer tetrahydrobiopterinAdminister tetrahydrobiopterin
– Replace dopamine and serotonin asReplace dopamine and serotonin as
tetrahydrobiopterin poorly penetrates braintetrahydrobiopterin poorly penetrates brain
where neurotransmitters are formedwhere neurotransmitters are formed
 Also remember there are a smallAlso remember there are a small
percentage of infants born with transientpercentage of infants born with transient
hyperphenylalaninemia which has nohyperphenylalaninemia which has no
clinical consequenceclinical consequence
www.indiandentalacademy.comwww.indiandentalacademy.com
Learning Points (71.1999)Learning Points (71.1999)
 Cobalamin is coenzyme foir methmalonyl CoACobalamin is coenzyme foir methmalonyl CoA
mutase whose deficiency results in…mutase whose deficiency results in…
– Methylmalonic aciduriaMethylmalonic aciduria
 Carnitine used for transport of medium and longCarnitine used for transport of medium and long
chain fatty acids across mitochondriachain fatty acids across mitochondria
– Carnitine deficient patients have muscle weaknessCarnitine deficient patients have muscle weakness
 Thiamine deficiency results in…Thiamine deficiency results in…
– Beri beriBeri beri
 Biotin is cofactor for carboxylases thatBiotin is cofactor for carboxylases that
catabolize branched chain amino acids involvedcatabolize branched chain amino acids involved
in fatty acid synthesisin fatty acid synthesis
www.indiandentalacademy.comwww.indiandentalacademy.com
Question 72Question 72
A 22-month-old girls is nonverbal. She sat alone at 7 months andA 22-month-old girls is nonverbal. She sat alone at 7 months and
walked by 13 months, but now exhibits a wide-based stance, nowalked by 13 months, but now exhibits a wide-based stance, no
longer ambulates, and will not pick up or manipulate toys. Findingslonger ambulates, and will not pick up or manipulate toys. Findings
include: height and weight at the 50include: height and weight at the 50thth
percentile; headpercentile; head
circumference below the 5circumference below the 5thth
percentile, with no increase over thepercentile, with no increase over the
past 8 months; normal fundi, and no organomegaly.past 8 months; normal fundi, and no organomegaly.
Of the following, the MOST likely diagnosis isOf the following, the MOST likely diagnosis is
A.A.AdrenoleukodystrophyAdrenoleukodystrophy
B.B.Cerebral palsyCerebral palsy
C.C.GMGM22 gangliosidosis (Tay-Sachs disease)gangliosidosis (Tay-Sachs disease)
D.D.HypothroidismHypothroidism
E.E.Rett syndromeRett syndrome
Answer: EAnswer: E
www.indiandentalacademy.comwww.indiandentalacademy.com
Learning Points (72.1999)Learning Points (72.1999)
 Developmental DelayDevelopmental Delay
– RegressionRegression
– PlateauPlateau
– ProgressionProgression
 Good somatic growth but no head growthGood somatic growth but no head growth
in 8 months known as…in 8 months known as…
– Acquired microcephalyAcquired microcephaly
 Lost purposeful hand use when picking upLost purposeful hand use when picking up
objects known as…objects known as…
– DyspraxiaDyspraxia
www.indiandentalacademy.comwww.indiandentalacademy.com
Learning Points (72.1999)Learning Points (72.1999)
 Rett syndrome (SSSH I don’t have Rett syndrome)Rett syndrome (SSSH I don’t have Rett syndrome)
– Neurodegenerative disorderNeurodegenerative disorder
– Females only (male fetuses die in utero)Females only (male fetuses die in utero)
– 1:10,0001:10,000
– X-linked dominantX-linked dominant
– SeizuresSeizures
– SpasticitySpasticity
– ScoliosisScoliosis
– Hand-wringingHand-wringing
– Become severe spastic quadriplegicsBecome severe spastic quadriplegics
www.indiandentalacademy.comwww.indiandentalacademy.com
Learning Points (72.1999)Learning Points (72.1999)
 AdrenoleukodystrophyAdrenoleukodystrophy
– Proxisomal disorder only found in malesProxisomal disorder only found in males
– Defect in acyl CoA synthetase in peroxisomesDefect in acyl CoA synthetase in peroxisomes
– Cerebral form (50%),Cerebral form (50%),
adrenomyeloneuropathy (25%), isolatedadrenomyeloneuropathy (25%), isolated
Addison disease or symptomatic (25%)Addison disease or symptomatic (25%)
– Progressive demyelination of brainProgressive demyelination of brain 
 Dementia, Blindness, Deafness, Decorticate stateDementia, Blindness, Deafness, Decorticate state
– Adrenal insufficiencyAdrenal insufficiency
www.indiandentalacademy.comwww.indiandentalacademy.com
Learning Points (72.1999)Learning Points (72.1999)
 Tay-SachsTay-Sachs
– Rapidly progressive neurodegenerative disorder due toRapidly progressive neurodegenerative disorder due to
lysosomal storage disorder (GM2, gangliosidosis)lysosomal storage disorder (GM2, gangliosidosis)
caused by deficiency in …caused by deficiency in …
 Beta-hexosaminidase A enzymeBeta-hexosaminidase A enzyme
– Onset before 9 monthsOnset before 9 months
– Hypotonic, never walk, increased startle responseHypotonic, never walk, increased startle response
– Fundoscopic exam reveals…Fundoscopic exam reveals…
 Cherry red spotCherry red spot
– What populations has this been found more frequentlyWhat populations has this been found more frequently
in?in?
 Ashkenazi JewsAshkenazi Jews
 French CanadiansFrench Canadians
www.indiandentalacademy.comwww.indiandentalacademy.com
Learning Points (72.1999)Learning Points (72.1999)
 Congenital hypothyroidismCongenital hypothyroidism
– High likelihood for normal development ifHigh likelihood for normal development if
treatment started before 3 months of agetreatment started before 3 months of age
– Can make up some developmental milestonesCan make up some developmental milestones
if treatment begun before 6 months of ageif treatment begun before 6 months of age
– Can see polyneurpathies and muscleCan see polyneurpathies and muscle
weaknessweakness
www.indiandentalacademy.comwww.indiandentalacademy.com
Learning Points (72.1999)Learning Points (72.1999)
 Cerebral Palsy: What are 4 types?Cerebral Palsy: What are 4 types?
– SpasticSpastic
– DyskineticDyskinetic
– AtaxicAtaxic
– HypotonicHypotonic
 Must have impaired motor control to makeMust have impaired motor control to make
this diagnosisthis diagnosis
www.indiandentalacademy.comwww.indiandentalacademy.com
Question 73Question 73
A 10-year-old boy who has hereditary spherocytosis has developedA 10-year-old boy who has hereditary spherocytosis has developed
increasing fatigue and pallor over the past 5 days, and hisincreasing fatigue and pallor over the past 5 days, and his
hemoglobin concentration, which usually ranges between 10 andhemoglobin concentration, which usually ranges between 10 and
11g/dL, now is 5.9g/dL.11g/dL, now is 5.9g/dL.
Of the following, the MOST likely etiology of this decrease inOf the following, the MOST likely etiology of this decrease in
hemoglobin ishemoglobin is
A.A.Aplastic crisisAplastic crisis
B.B.Folic acid deficiencyFolic acid deficiency
C.C.Hyperhemolytic crisisHyperhemolytic crisis
D.D.Iron-deficiency anemiaIron-deficiency anemia
E.E.Splenic sequestrationSplenic sequestration
Answer: AAnswer: A
www.indiandentalacademy.comwww.indiandentalacademy.com
Learning Points (73.1999)Learning Points (73.1999)
 Aplastic crisis is the most common cause of severeAplastic crisis is the most common cause of severe
anemia in children with hereditary spherocytosisanemia in children with hereditary spherocytosis
(HS)(HS)
 What is the most common cause of such a crisis?What is the most common cause of such a crisis?
– Parvovirus B19Parvovirus B19 erythema infectiosum (fever, chills,erythema infectiosum (fever, chills,
lethargy, nausea, vomiting, abdominal pain, respiratorylethargy, nausea, vomiting, abdominal pain, respiratory
symptoms, myalgias, arthralgias, “slapped cheek” rash)symptoms, myalgias, arthralgias, “slapped cheek” rash)
– Parvovirus selectively attacks erythroid precursor cellsParvovirus selectively attacks erythroid precursor cells
inhibiting their growthinhibiting their growth  decreased reticulocytes anddecreased reticulocytes and
bilirubin as number of RBCs destroyed decreases dailybilirubin as number of RBCs destroyed decreases daily
www.indiandentalacademy.comwww.indiandentalacademy.com
Learning Points (73.1999)Learning Points (73.1999)
 How long does the crisis typically last?How long does the crisis typically last?
– 10-14 days10-14 days
 Are WBCs and platelets affected as well?Are WBCs and platelets affected as well?
– Yes they can but it is usually much milderYes they can but it is usually much milder
 When do you know someone is recovering fromWhen do you know someone is recovering from
a lab standpoint?a lab standpoint?
– Presence of a reticulocytosisPresence of a reticulocytosis
 * Parvovirus infection in first half of pregnancy in* Parvovirus infection in first half of pregnancy in
patient with HS may cause fetal loss due topatient with HS may cause fetal loss due to
severe anemiasevere anemia
www.indiandentalacademy.comwww.indiandentalacademy.com
Learning Points (73.1999)Learning Points (73.1999)
 What percentage of kids with HS haveWhat percentage of kids with HS have
mild or no anemia?mild or no anemia?
– 90%90%
 How long is the life span of a RBC in aHow long is the life span of a RBC in a
patient with HS?patient with HS?
– 30 days30 days
 With this short of a lifespan hw does theWith this short of a lifespan hw does the
body keep up?body keep up?
– Enhanced erythropoiesisEnhanced erythropoiesis
www.indiandentalacademy.comwww.indiandentalacademy.com
Learning Points (73.1999)Learning Points (73.1999)
 Folic acid and iron deficiency may exacerbateFolic acid and iron deficiency may exacerbate
the anemia but this is a gradual decline inthe anemia but this is a gradual decline in
hemoglobinhemoglobin
 Hyperhemolytic crisis without aplasia occurs inHyperhemolytic crisis without aplasia occurs in
HS in children less than 6 years who have viralHS in children less than 6 years who have viral
syndromessyndromes
– Mild and transient increases in bilirubin,Mild and transient increases in bilirubin,
splenomegaly, anemia, reticulocytosissplenomegaly, anemia, reticulocytosis
 Splenic sequestration is typical of sickle cellSplenic sequestration is typical of sickle cell
disease and is a minor component of hemolyticdisease and is a minor component of hemolytic
crisis associated with spherocytosiscrisis associated with spherocytosis
www.indiandentalacademy.comwww.indiandentalacademy.com
Question 74Question 74
An 18-day-old term infant develops fever, lethargy, and focalAn 18-day-old term infant develops fever, lethargy, and focal
seizures. Findings include: an ill-appearing infant withoutseizures. Findings include: an ill-appearing infant without
exanthem, hepatomegaly, or jaundice. Analysis of cerebrospinalexanthem, hepatomegaly, or jaundice. Analysis of cerebrospinal
fluid reveals white blood cells, 115/mmfluid reveals white blood cells, 115/mm33
; 45% neutrophils; 55%; 45% neutrophils; 55%
lymphocytes; red blood cells, 40/mmlymphocytes; red blood cells, 40/mm33
; glucose, 45 mg/dL; protien; glucose, 45 mg/dL; protien
200 mg/dL; and negative Gram stain.200 mg/dL; and negative Gram stain.
In addition to ampicillin and cefotaxime, the MOST appropriateIn addition to ampicillin and cefotaxime, the MOST appropriate
treatment to begin at this time istreatment to begin at this time is
A.A.AcyclovirAcyclovir
B.B.Amphotericin BAmphotericin B
C.C.DexamethasoneDexamethasone
D.D.MetronidazoleMetronidazole
E.E.vancomycinvancomycin
Answer: AAnswer: Awww.indiandentalacademy.comwww.indiandentalacademy.com
Learning Points (74.1999)Learning Points (74.1999)
 HSV is transmitted during delivery via contact withHSV is transmitted during delivery via contact with
maternal secretionsmaternal secretions
 Only 5% of adults in US have a history of genital herpesOnly 5% of adults in US have a history of genital herpes
 20-25% are actively infected with HSV-220-25% are actively infected with HSV-2
 What is the risk of an HSV infection in an infant bornWhat is the risk of an HSV infection in an infant born
vaginally to a mother who has a 1vaginally to a mother who has a 1stst
or primary genitalor primary genital
infection?infection?
– 33-50%33-50%
 What is the risk in a mom with recurrent HSV?What is the risk in a mom with recurrent HSV?
– 3-5%3-5%
 MOST HSV-INFECTED INFANTS ARE BORN TOMOST HSV-INFECTED INFANTS ARE BORN TO
WOMEN WITH NO HISTORY OF GENITAL HERPESWOMEN WITH NO HISTORY OF GENITAL HERPES
AND NO SIGNS OF INFECTION DURINGAND NO SIGNS OF INFECTION DURING
PREGNANCY OR AT DELIVERYPREGNANCY OR AT DELIVERY
www.indiandentalacademy.comwww.indiandentalacademy.com
Learning Points (74.1999)Learning Points (74.1999)
 HSV in newbornHSV in newborn
– Generalized systemic infection involving liverGeneralized systemic infection involving liver
and CNS in 25% casesand CNS in 25% cases
– Localized CNS disease 35%Localized CNS disease 35%
– Localized to skin, eyes, mouth 40%Localized to skin, eyes, mouth 40%
– UP TO 33% OF INFANTS WITHUP TO 33% OF INFANTS WITH
DISSEMINATED OR LOCALIZED CNSDISSEMINATED OR LOCALIZED CNS
DISEASE WILL HAVEDISEASE WILL HAVE NONO SKIN, EYES ORSKIN, EYES OR
MOUTH INVOLVEMENTMOUTH INVOLVEMENT
www.indiandentalacademy.comwww.indiandentalacademy.com
Learning Points (74.1999)Learning Points (74.1999)
 Disseminated HSV diseaseDisseminated HSV disease
– 11stst
week of lifeweek of life
– Liver and adrenals primarily involvedLiver and adrenals primarily involved
– Irritability, seizures, respiratory distress,Irritability, seizures, respiratory distress,
jaundice, vesicular rash, shockjaundice, vesicular rash, shock
– What is the mortality rate even with treatment?What is the mortality rate even with treatment?
 50-60%50-60%
– What are the two most common causes ofWhat are the two most common causes of
death in disseminated HSV?death in disseminated HSV?
 HSV pneumonitisHSV pneumonitis
 Disseminated intravascular coagulopathyDisseminated intravascular coagulopathy
www.indiandentalacademy.comwww.indiandentalacademy.com
Question 75Question 75
A 10-year-old girl has complained of intermittent left lowerA 10-year-old girl has complained of intermittent left lower
abdominal pain for 2 days. Previous evaluations, including aabdominal pain for 2 days. Previous evaluations, including a
thorough physical examination, urinalysis, and complete bloodthorough physical examination, urinalysis, and complete blood
count, have not revealed the cause. Tonight she is complaining ofcount, have not revealed the cause. Tonight she is complaining of
pain in the lower left abdomen that radiates into her left leg. Therepain in the lower left abdomen that radiates into her left leg. There
is no history of fever, vomiting, or diarrhea.is no history of fever, vomiting, or diarrhea.
The MOST likely cause of this girl’s pain isThe MOST likely cause of this girl’s pain is
A.A.AppendicitisAppendicitis
B.B.IntussusceptionIntussusception
C.C.Malrotation of the intestineMalrotation of the intestine
D.D.NephrolithiasisNephrolithiasis
E.E.Ovarian torsionOvarian torsion
Answer: EAnswer: E
www.indiandentalacademy.comwww.indiandentalacademy.com
Teaching Points 75.1999Teaching Points 75.1999
 Ovarian torsion pain: sharp lower abdominal pain radiatingOvarian torsion pain: sharp lower abdominal pain radiating
to ipsilateral extremity, occasional vomitingto ipsilateral extremity, occasional vomiting
 US can assess ovariesUS can assess ovaries
 Surgical exploration to confirm diagnosis and preventSurgical exploration to confirm diagnosis and prevent
ovarian necrosis (and R/O appendicitis if not already done)ovarian necrosis (and R/O appendicitis if not already done)
 Left sided pain with appendicitis rare but seenLeft sided pain with appendicitis rare but seen
 What is pain on right with palpation of the left called?What is pain on right with palpation of the left called?
– Rovsing’s SignRovsing’s Sign
 What is the obturator sign?What is the obturator sign?
– Pain on internal rotation of the right hip, seen with pelvic appendixPain on internal rotation of the right hip, seen with pelvic appendix
 What is the iliopsoas sign?What is the iliopsoas sign?
– Pain on right hip extension, often seen with retrocecal appendixPain on right hip extension, often seen with retrocecal appendix
www.indiandentalacademy.comwww.indiandentalacademy.com
Teaching Points 75.1999Teaching Points 75.1999
 Wrong age for intussusception (around 2 yearsWrong age for intussusception (around 2 years
of age)of age)
 Irritability, colicky abdominal pain, emesisIrritability, colicky abdominal pain, emesis
 Impaired venous returnImpaired venous return  bowel edemabowel edema 
ischemiaischemia  necrosisnecrosis  perforationperforation
 Rectal bleeding seen in what percentage ofRectal bleeding seen in what percentage of
kids?kids?
– 80%; in “currant jelly” form (blood plus mucous)80%; in “currant jelly” form (blood plus mucous)
 Most common location?Most common location?
– IleocolicIleocolic
 Tubular mass palpable in what percentage ofTubular mass palpable in what percentage of
kids?kids?
– 80%80%
www.indiandentalacademy.comwww.indiandentalacademy.com
Teaching Points 75.1999Teaching Points 75.1999
 Lead point should be sought out in neonates and childrenLead point should be sought out in neonates and children
older than 5. Name lead points.older than 5. Name lead points.
– Meckel’s diverticulumMeckel’s diverticulum
– Intestinal polypIntestinal polyp
– LymphomaLymphoma
– Foreign bodyForeign body
 KUB: Paucity gas in RLQ or evidence of obstruction with airKUB: Paucity gas in RLQ or evidence of obstruction with air
fluid levelsfluid levels
 Gastrograffin enema shows coiled-spring appearance toGastrograffin enema shows coiled-spring appearance to
bowel which is diagnostic, and may treat as well 75% casesbowel which is diagnostic, and may treat as well 75% cases
 IVFs with NSIVFs with NS
 Laporotomy with direct reduction if enema unsuccessful orLaporotomy with direct reduction if enema unsuccessful or
peritoneal signs present and enema contraindicated due toperitoneal signs present and enema contraindicated due to
concern of perforationconcern of perforation
 Immediate recurrence rate 15%Immediate recurrence rate 15%
www.indiandentalacademy.comwww.indiandentalacademy.com
Question 76Question 76
A 13-year-old boy has a congested, itchy, and runny noseA 13-year-old boy has a congested, itchy, and runny nose
accompanied by itchy eyes. These symptoms usually occur in theaccompanied by itchy eyes. These symptoms usually occur in the
fall, are unresponsive to over-the-counter decongestants, and hisfall, are unresponsive to over-the-counter decongestants, and his
father has the same problems. Physical examination reveals pale,father has the same problems. Physical examination reveals pale,
boggy nasal turbinates; clear nasal discharge; and dark circlesboggy nasal turbinates; clear nasal discharge; and dark circles
under his eyes.under his eyes.
These findings are MOST consistent withThese findings are MOST consistent with
A.A.Allergic rhinitisAllergic rhinitis
B.B.Infectious rhinitisInfectious rhinitis
C.C.Nonallergic rhinitis with eosinophiliaNonallergic rhinitis with eosinophilia
D.D.Rhinitis medicamentosaRhinitis medicamentosa
E.E.Vasomotor rhinitisVasomotor rhinitis
Answer: AAnswer: A
www.indiandentalacademy.comwww.indiandentalacademy.com
Teaching Points 76.1999Teaching Points 76.1999
 Perennial rhinitis (year round)Perennial rhinitis (year round)
– Pets, dust mites, molds, cockroachesPets, dust mites, molds, cockroaches
 Vasomotor rhinitisVasomotor rhinitis
– Diagnosis of exclusion, chronic blockage orDiagnosis of exclusion, chronic blockage or
hypersecretion but negative skin tests and normalhypersecretion but negative skin tests and normal
paranasal sinus imagingparanasal sinus imaging
 Infectious rhinitisInfectious rhinitis
– Mucopurulent discharge, sneezing, limited pruritusMucopurulent discharge, sneezing, limited pruritus
 Rhinitis medicamentosaRhinitis medicamentosa
– Overuse topical decongestantsOveruse topical decongestants
 Nonallergic rhinitis with eosinophiliaNonallergic rhinitis with eosinophilia
– Diagnosis of exclusion, negative skin tests, nasalDiagnosis of exclusion, negative skin tests, nasal
smear positive for eosinophilssmear positive for eosinophils
www.indiandentalacademy.comwww.indiandentalacademy.com
Pale turbinatesPale turbinates
http://www.entusa.com/Nasal%20Photos/Chronic_Allergic_Rhinitis_15.jpghttp://www.entusa.com/Nasal%20Photos/Chronic_Allergic_Rhinitis_15.jpg
www.indiandentalacademy.comwww.indiandentalacademy.com
Question 77Question 77
A 5-month-old child is brought to the emergency department by herA 5-month-old child is brought to the emergency department by her
mother because she has been “crying a lot” for the past 24 hours.mother because she has been “crying a lot” for the past 24 hours.
The mother denies any history of fever, trauma, or illness. PhysicalThe mother denies any history of fever, trauma, or illness. Physical
examination reveals a lethargic toddler who is very irritable whenexamination reveals a lethargic toddler who is very irritable when
examined and who has mild tachycardia, scattered bruises over theexamined and who has mild tachycardia, scattered bruises over the
chest, and ecchymosis behind the left ear.chest, and ecchymosis behind the left ear.
The MOST appropriate management is toThe MOST appropriate management is to
A.A.Administer intravenous naloxoneAdminister intravenous naloxone
B.B.Administer a 20 mL/kg bolus of intravenous normal salineAdminister a 20 mL/kg bolus of intravenous normal saline
C.C.Obtain an abdominal radiographObtain an abdominal radiograph
D.D.Obtain a complete blood count and blood culture and administerObtain a complete blood count and blood culture and administer
intravenous ceftriaxoneintravenous ceftriaxone
E.E.Obtain a computed tomographic scan of the head and a skeletalObtain a computed tomographic scan of the head and a skeletal
survey.survey.
Answer: EAnswer: E
www.indiandentalacademy.comwww.indiandentalacademy.com
Teaching Points 77.1999Teaching Points 77.1999
 Child abuse red flagsChild abuse red flags
– Inconsistent historyInconsistent history
– History not compatible with injuryHistory not compatible with injury
 Ecchymosis behind left ear is called…Ecchymosis behind left ear is called…
– Battle signBattle sign
 What does this indicate?What does this indicate?
– Basilar skull fractureBasilar skull fracture
 CSF leakage from nose or ear, periorbitalCSF leakage from nose or ear, periorbital
ecchymosis (Raccoon Sign), blood behind theecchymosis (Raccoon Sign), blood behind the
eat drum (hemotympanum) are also signs ofeat drum (hemotympanum) are also signs of
basal skull fracturebasal skull fracture
www.indiandentalacademy.comwww.indiandentalacademy.com
Battle SignBattle Sign
www.aic.cuhk.edu.hk/web8/Battlewww.aic.cuhk.edu.hk/web8/Battle
www.indiandentalacademy.comwww.indiandentalacademy.com
Periorbital EcchymosisPeriorbital Ecchymosis
(Raccoon Eyes)(Raccoon Eyes)
www.indianpediatrics.net/sep2005/sep-949.HTMwww.indianpediatrics.net/sep2005/sep-949.HTM
 15 month old with stage IV metastatic15 month old with stage IV metastatic
neuroblastoma and involvement ofneuroblastoma and involvement of
the periorbital tissues and resultantthe periorbital tissues and resultant
proptosis and orbital ecchymosisproptosis and orbital ecchymosis
 Orbital metastases found in up toOrbital metastases found in up to
20% of children with stage IV20% of children with stage IV
neuroblastoma.neuroblastoma.
 Raccoon eyes appearanceRaccoon eyes appearance
associated with neuroblastoma isassociated with neuroblastoma is
probably related to obstruction of theprobably related to obstruction of the
palpebral vessels by tumor tissue inpalpebral vessels by tumor tissue in
and around the orbit.and around the orbit.
 Differential diagnoses for periorbitalDifferential diagnoses for periorbital
edema and ecchymosisedema and ecchymosis
– Child abuse or traumaChild abuse or trauma
– Infection of the soft tissues associatedInfection of the soft tissues associated
with a spreading dental infectionwith a spreading dental infection
– Allergic reactionAllergic reaction
– MyxoedemaMyxoedema
– LymphomaLymphoma
– HaemophiliaHaemophilia
www.indiandentalacademy.comwww.indiandentalacademy.com
HemotympanumHemotympanum
me.hawkelibrary.com/hemotympanum/26_Mme.hawkelibrary.com/hemotympanum/26_M
www.indiandentalacademy.comwww.indiandentalacademy.com
Question 78Question 78
An important component of the diagnostic assessment forAn important component of the diagnostic assessment for
any child suspected of having attention deficit hyperactivityany child suspected of having attention deficit hyperactivity
disorder (ADHD) is the use of standardized behavior ratingdisorder (ADHD) is the use of standardized behavior rating
scales. Of the following, the BEST reason to usescales. Of the following, the BEST reason to use
standardized behavior rating scales is that theystandardized behavior rating scales is that they
A.A.Are specific for diagnosing ADHDAre specific for diagnosing ADHD
B.B.Assist in monitoring treatmentAssist in monitoring treatment
C.C.Can be completed after a single classroom observationCan be completed after a single classroom observation
D.D.Determine initial medication dosageDetermine initial medication dosage
E.E.Predict response to medicationPredict response to medication
Answer: BAnswer: B
www.indiandentalacademy.comwww.indiandentalacademy.com
Teaching Points 78.1999Teaching Points 78.1999
www.indiandentalacademy.comwww.indiandentalacademy.com
Teaching Points 78.1999Teaching Points 78.1999
www.indiandentalacademy.comwww.indiandentalacademy.com
Question 79Question 79
A 14-year-old boy who has allergic rhinitis reports that heA 14-year-old boy who has allergic rhinitis reports that he
frequently develops coughing and wheezing after aboutfrequently develops coughing and wheezing after about
minutes of playing soccer. These symptoms improve afterminutes of playing soccer. These symptoms improve after
resting for 30 minutes. Of the following, the drug that will giveresting for 30 minutes. Of the following, the drug that will give
the BEST response in this patient if administered just prior tothe BEST response in this patient if administered just prior to
exercise isexercise is
A.A.Inhaled betaInhaled beta22-agonist-agonist
B.B.Inhaled corticosteroidInhaled corticosteroid
C.C.Oral betaOral beta22-agonist-agonist
D.D.Oral corticosteroidOral corticosteroid
E.E.Oral theophyllineOral theophylline
Answer: AAnswer: A
www.indiandentalacademy.comwww.indiandentalacademy.com
Teaching Points 79.1999Teaching Points 79.1999
 Exercise Induced AsthmaExercise Induced Asthma
– Bronchoconstriction during exerciseBronchoconstriction during exercise
– Typically within 15 minutesTypically within 15 minutes
– Can occur during cool down as a lat-phaseCan occur during cool down as a lat-phase
response up to 4-12 hours laterresponse up to 4-12 hours later
– Cough, wheeze, SOB, dizzy, stomach painCough, wheeze, SOB, dizzy, stomach pain
– Occurs 80% patients with asthmaOccurs 80% patients with asthma
– Occurs 50% patients with allergic rhinitisOccurs 50% patients with allergic rhinitis
www.indiandentalacademy.comwww.indiandentalacademy.com
Pediatric board review/certified fixed orthodontic courses by Indian dental academy
Pediatric board review/certified fixed orthodontic courses by Indian dental academy
Pediatric board review/certified fixed orthodontic courses by Indian dental academy
Pediatric board review/certified fixed orthodontic courses by Indian dental academy
Pediatric board review/certified fixed orthodontic courses by Indian dental academy
Pediatric board review/certified fixed orthodontic courses by Indian dental academy
Pediatric board review/certified fixed orthodontic courses by Indian dental academy
Pediatric board review/certified fixed orthodontic courses by Indian dental academy
Pediatric board review/certified fixed orthodontic courses by Indian dental academy
Pediatric board review/certified fixed orthodontic courses by Indian dental academy
Pediatric board review/certified fixed orthodontic courses by Indian dental academy
Pediatric board review/certified fixed orthodontic courses by Indian dental academy
Pediatric board review/certified fixed orthodontic courses by Indian dental academy

More Related Content

Similar to Pediatric board review/certified fixed orthodontic courses by Indian dental academy

opioids for general anaesthesia for c section
opioids for general anaesthesia for c sectionopioids for general anaesthesia for c section
opioids for general anaesthesia for c sectionRoshan Fernando
 
16672475 23 Mock Codes 2
16672475 23  Mock  Codes 216672475 23  Mock  Codes 2
16672475 23 Mock Codes 2Surgery Center
 
Identification ,management & referral of a sick.ppt [autosaved]
Identification ,management & referral of a sick.ppt [autosaved]Identification ,management & referral of a sick.ppt [autosaved]
Identification ,management & referral of a sick.ppt [autosaved]Mahesh Hiranandani
 
Cardio Pulmonary Resuscitation
Cardio Pulmonary ResuscitationCardio Pulmonary Resuscitation
Cardio Pulmonary ResuscitationEneutron
 
Pediatrics CME 2006
Pediatrics CME 2006Pediatrics CME 2006
Pediatrics CME 2006brownEMS
 
Creating Health and Balance in Today's Classroom
Creating Health and Balance in Today's ClassroomCreating Health and Balance in Today's Classroom
Creating Health and Balance in Today's ClassroomRegional Training Center
 
Medical-Mnemonic-Booklet.pdf jdnd
Medical-Mnemonic-Booklet.pdf jdndMedical-Mnemonic-Booklet.pdf jdnd
Medical-Mnemonic-Booklet.pdf jdndNancyJhariya
 
Sleep disorders dentistry
Sleep disorders dentistrySleep disorders dentistry
Sleep disorders dentistryJohn Viviano
 
Pain &amp; pain/certified fixed orthodontic courses by Indian dental academy
Pain &amp; pain/certified fixed orthodontic courses by Indian dental academyPain &amp; pain/certified fixed orthodontic courses by Indian dental academy
Pain &amp; pain/certified fixed orthodontic courses by Indian dental academyIndian dental academy
 
Santosh seminar/ dental implant courses
Santosh seminar/ dental implant coursesSantosh seminar/ dental implant courses
Santosh seminar/ dental implant coursesIndian dental academy
 
Bell's palsy with oral submucous fibrosis /prosthodontic courses
Bell's palsy with oral submucous fibrosis /prosthodontic coursesBell's palsy with oral submucous fibrosis /prosthodontic courses
Bell's palsy with oral submucous fibrosis /prosthodontic coursesIndian dental academy
 
penyakit infeksi pada anak
penyakit infeksi pada anakpenyakit infeksi pada anak
penyakit infeksi pada anakKindal
 

Similar to Pediatric board review/certified fixed orthodontic courses by Indian dental academy (20)

opioids for general anaesthesia for c section
opioids for general anaesthesia for c sectionopioids for general anaesthesia for c section
opioids for general anaesthesia for c section
 
16672475 23 Mock Codes 2
16672475 23  Mock  Codes 216672475 23  Mock  Codes 2
16672475 23 Mock Codes 2
 
Identification ,management & referral of a sick.ppt [autosaved]
Identification ,management & referral of a sick.ppt [autosaved]Identification ,management & referral of a sick.ppt [autosaved]
Identification ,management & referral of a sick.ppt [autosaved]
 
O'Rourke spreecast 11 9-15
O'Rourke spreecast 11 9-15O'Rourke spreecast 11 9-15
O'Rourke spreecast 11 9-15
 
2012 asthma
2012 asthma2012 asthma
2012 asthma
 
Cardio Pulmonary Resuscitation
Cardio Pulmonary ResuscitationCardio Pulmonary Resuscitation
Cardio Pulmonary Resuscitation
 
Foreign body airway
Foreign body airwayForeign body airway
Foreign body airway
 
Pediatrics CME 2006
Pediatrics CME 2006Pediatrics CME 2006
Pediatrics CME 2006
 
Creating Health and Balance in Today's Classroom
Creating Health and Balance in Today's ClassroomCreating Health and Balance in Today's Classroom
Creating Health and Balance in Today's Classroom
 
Bacic cpr
Bacic cprBacic cpr
Bacic cpr
 
Medical-Mnemonic-Booklet.pdf jdnd
Medical-Mnemonic-Booklet.pdf jdndMedical-Mnemonic-Booklet.pdf jdnd
Medical-Mnemonic-Booklet.pdf jdnd
 
ALS and BTLS
ALS and BTLSALS and BTLS
ALS and BTLS
 
SPHMMC.pptx
SPHMMC.pptxSPHMMC.pptx
SPHMMC.pptx
 
Sleep disorders dentistry
Sleep disorders dentistrySleep disorders dentistry
Sleep disorders dentistry
 
Intestinal Obstruction 2
Intestinal Obstruction 2Intestinal Obstruction 2
Intestinal Obstruction 2
 
Pain &amp; pain/certified fixed orthodontic courses by Indian dental academy
Pain &amp; pain/certified fixed orthodontic courses by Indian dental academyPain &amp; pain/certified fixed orthodontic courses by Indian dental academy
Pain &amp; pain/certified fixed orthodontic courses by Indian dental academy
 
Santosh seminar/ dental implant courses
Santosh seminar/ dental implant coursesSantosh seminar/ dental implant courses
Santosh seminar/ dental implant courses
 
Whats Up With The Flu
Whats Up With The FluWhats Up With The Flu
Whats Up With The Flu
 
Bell's palsy with oral submucous fibrosis /prosthodontic courses
Bell's palsy with oral submucous fibrosis /prosthodontic coursesBell's palsy with oral submucous fibrosis /prosthodontic courses
Bell's palsy with oral submucous fibrosis /prosthodontic courses
 
penyakit infeksi pada anak
penyakit infeksi pada anakpenyakit infeksi pada anak
penyakit infeksi pada anak
 

More from Indian dental academy

Indian Dentist - relocate to united kingdom
Indian Dentist - relocate to united kingdomIndian Dentist - relocate to united kingdom
Indian Dentist - relocate to united kingdomIndian dental academy
 
1ST, 2ND AND 3RD ORDER BENDS IN STANDARD EDGEWISE APPLIANCE SYSTEM /Fixed ort...
1ST, 2ND AND 3RD ORDER BENDS IN STANDARD EDGEWISE APPLIANCE SYSTEM /Fixed ort...1ST, 2ND AND 3RD ORDER BENDS IN STANDARD EDGEWISE APPLIANCE SYSTEM /Fixed ort...
1ST, 2ND AND 3RD ORDER BENDS IN STANDARD EDGEWISE APPLIANCE SYSTEM /Fixed ort...Indian dental academy
 
Invisalign -invisible aligners course in india
Invisalign -invisible aligners course in india Invisalign -invisible aligners course in india
Invisalign -invisible aligners course in india Indian dental academy
 
Invisible aligners for your orthodontics pratice
Invisible aligners for your orthodontics praticeInvisible aligners for your orthodontics pratice
Invisible aligners for your orthodontics praticeIndian dental academy
 
Development of muscles of mastication / dental implant courses
Development of muscles of mastication / dental implant coursesDevelopment of muscles of mastication / dental implant courses
Development of muscles of mastication / dental implant coursesIndian dental academy
 
Corticosteriods uses in dentistry/ oral surgery courses  
Corticosteriods uses in dentistry/ oral surgery courses  Corticosteriods uses in dentistry/ oral surgery courses  
Corticosteriods uses in dentistry/ oral surgery courses  Indian dental academy
 
Cytotoxicity of silicone materials used in maxillofacial prosthesis / dental ...
Cytotoxicity of silicone materials used in maxillofacial prosthesis / dental ...Cytotoxicity of silicone materials used in maxillofacial prosthesis / dental ...
Cytotoxicity of silicone materials used in maxillofacial prosthesis / dental ...Indian dental academy
 
Diagnosis and treatment planning in completely endntulous arches/dental courses
Diagnosis and treatment planning in completely endntulous arches/dental coursesDiagnosis and treatment planning in completely endntulous arches/dental courses
Diagnosis and treatment planning in completely endntulous arches/dental coursesIndian dental academy
 
Properties of Denture base materials /rotary endodontic courses
Properties of Denture base materials /rotary endodontic coursesProperties of Denture base materials /rotary endodontic courses
Properties of Denture base materials /rotary endodontic coursesIndian dental academy
 
Use of modified tooth forms in complete denture occlusion / dental implant...
Use of modified  tooth forms  in  complete denture occlusion / dental implant...Use of modified  tooth forms  in  complete denture occlusion / dental implant...
Use of modified tooth forms in complete denture occlusion / dental implant...Indian dental academy
 
Dental luting cements / oral surgery courses  
Dental   luting cements / oral surgery courses  Dental   luting cements / oral surgery courses  
Dental luting cements / oral surgery courses  Indian dental academy
 
Dental casting alloys/ oral surgery courses  
Dental casting alloys/ oral surgery courses  Dental casting alloys/ oral surgery courses  
Dental casting alloys/ oral surgery courses  Indian dental academy
 
Dental casting investment materials/endodontic courses
Dental casting investment materials/endodontic coursesDental casting investment materials/endodontic courses
Dental casting investment materials/endodontic coursesIndian dental academy
 
Dental casting waxes/ oral surgery courses  
Dental casting waxes/ oral surgery courses  Dental casting waxes/ oral surgery courses  
Dental casting waxes/ oral surgery courses  Indian dental academy
 
Dental ceramics/prosthodontic courses
Dental ceramics/prosthodontic coursesDental ceramics/prosthodontic courses
Dental ceramics/prosthodontic coursesIndian dental academy
 
Dental implant/ oral surgery courses  
Dental implant/ oral surgery courses  Dental implant/ oral surgery courses  
Dental implant/ oral surgery courses  Indian dental academy
 
Dental perspective/cosmetic dentistry courses
Dental perspective/cosmetic dentistry coursesDental perspective/cosmetic dentistry courses
Dental perspective/cosmetic dentistry coursesIndian dental academy
 
Dental tissues and their replacements/ oral surgery courses  
Dental tissues and their replacements/ oral surgery courses  Dental tissues and their replacements/ oral surgery courses  
Dental tissues and their replacements/ oral surgery courses  Indian dental academy
 

More from Indian dental academy (20)

Indian Dentist - relocate to united kingdom
Indian Dentist - relocate to united kingdomIndian Dentist - relocate to united kingdom
Indian Dentist - relocate to united kingdom
 
1ST, 2ND AND 3RD ORDER BENDS IN STANDARD EDGEWISE APPLIANCE SYSTEM /Fixed ort...
1ST, 2ND AND 3RD ORDER BENDS IN STANDARD EDGEWISE APPLIANCE SYSTEM /Fixed ort...1ST, 2ND AND 3RD ORDER BENDS IN STANDARD EDGEWISE APPLIANCE SYSTEM /Fixed ort...
1ST, 2ND AND 3RD ORDER BENDS IN STANDARD EDGEWISE APPLIANCE SYSTEM /Fixed ort...
 
Invisalign -invisible aligners course in india
Invisalign -invisible aligners course in india Invisalign -invisible aligners course in india
Invisalign -invisible aligners course in india
 
Invisible aligners for your orthodontics pratice
Invisible aligners for your orthodontics praticeInvisible aligners for your orthodontics pratice
Invisible aligners for your orthodontics pratice
 
online fixed orthodontics course
online fixed orthodontics courseonline fixed orthodontics course
online fixed orthodontics course
 
online orthodontics course
online orthodontics courseonline orthodontics course
online orthodontics course
 
Development of muscles of mastication / dental implant courses
Development of muscles of mastication / dental implant coursesDevelopment of muscles of mastication / dental implant courses
Development of muscles of mastication / dental implant courses
 
Corticosteriods uses in dentistry/ oral surgery courses  
Corticosteriods uses in dentistry/ oral surgery courses  Corticosteriods uses in dentistry/ oral surgery courses  
Corticosteriods uses in dentistry/ oral surgery courses  
 
Cytotoxicity of silicone materials used in maxillofacial prosthesis / dental ...
Cytotoxicity of silicone materials used in maxillofacial prosthesis / dental ...Cytotoxicity of silicone materials used in maxillofacial prosthesis / dental ...
Cytotoxicity of silicone materials used in maxillofacial prosthesis / dental ...
 
Diagnosis and treatment planning in completely endntulous arches/dental courses
Diagnosis and treatment planning in completely endntulous arches/dental coursesDiagnosis and treatment planning in completely endntulous arches/dental courses
Diagnosis and treatment planning in completely endntulous arches/dental courses
 
Properties of Denture base materials /rotary endodontic courses
Properties of Denture base materials /rotary endodontic coursesProperties of Denture base materials /rotary endodontic courses
Properties of Denture base materials /rotary endodontic courses
 
Use of modified tooth forms in complete denture occlusion / dental implant...
Use of modified  tooth forms  in  complete denture occlusion / dental implant...Use of modified  tooth forms  in  complete denture occlusion / dental implant...
Use of modified tooth forms in complete denture occlusion / dental implant...
 
Dental luting cements / oral surgery courses  
Dental   luting cements / oral surgery courses  Dental   luting cements / oral surgery courses  
Dental luting cements / oral surgery courses  
 
Dental casting alloys/ oral surgery courses  
Dental casting alloys/ oral surgery courses  Dental casting alloys/ oral surgery courses  
Dental casting alloys/ oral surgery courses  
 
Dental casting investment materials/endodontic courses
Dental casting investment materials/endodontic coursesDental casting investment materials/endodontic courses
Dental casting investment materials/endodontic courses
 
Dental casting waxes/ oral surgery courses  
Dental casting waxes/ oral surgery courses  Dental casting waxes/ oral surgery courses  
Dental casting waxes/ oral surgery courses  
 
Dental ceramics/prosthodontic courses
Dental ceramics/prosthodontic coursesDental ceramics/prosthodontic courses
Dental ceramics/prosthodontic courses
 
Dental implant/ oral surgery courses  
Dental implant/ oral surgery courses  Dental implant/ oral surgery courses  
Dental implant/ oral surgery courses  
 
Dental perspective/cosmetic dentistry courses
Dental perspective/cosmetic dentistry coursesDental perspective/cosmetic dentistry courses
Dental perspective/cosmetic dentistry courses
 
Dental tissues and their replacements/ oral surgery courses  
Dental tissues and their replacements/ oral surgery courses  Dental tissues and their replacements/ oral surgery courses  
Dental tissues and their replacements/ oral surgery courses  
 

Recently uploaded

DATA STRUCTURE AND ALGORITHM for beginners
DATA STRUCTURE AND ALGORITHM for beginnersDATA STRUCTURE AND ALGORITHM for beginners
DATA STRUCTURE AND ALGORITHM for beginnersSabitha Banu
 
Biting mechanism of poisonous snakes.pdf
Biting mechanism of poisonous snakes.pdfBiting mechanism of poisonous snakes.pdf
Biting mechanism of poisonous snakes.pdfadityarao40181
 
Roles & Responsibilities in Pharmacovigilance
Roles & Responsibilities in PharmacovigilanceRoles & Responsibilities in Pharmacovigilance
Roles & Responsibilities in PharmacovigilanceSamikshaHamane
 
History Class XII Ch. 3 Kinship, Caste and Class (1).pptx
History Class XII Ch. 3 Kinship, Caste and Class (1).pptxHistory Class XII Ch. 3 Kinship, Caste and Class (1).pptx
History Class XII Ch. 3 Kinship, Caste and Class (1).pptxsocialsciencegdgrohi
 
How to Make a Pirate ship Primary Education.pptx
How to Make a Pirate ship Primary Education.pptxHow to Make a Pirate ship Primary Education.pptx
How to Make a Pirate ship Primary Education.pptxmanuelaromero2013
 
Full Stack Web Development Course for Beginners
Full Stack Web Development Course  for BeginnersFull Stack Web Development Course  for Beginners
Full Stack Web Development Course for BeginnersSabitha Banu
 
ECONOMIC CONTEXT - PAPER 1 Q3: NEWSPAPERS.pptx
ECONOMIC CONTEXT - PAPER 1 Q3: NEWSPAPERS.pptxECONOMIC CONTEXT - PAPER 1 Q3: NEWSPAPERS.pptx
ECONOMIC CONTEXT - PAPER 1 Q3: NEWSPAPERS.pptxiammrhaywood
 
Crayon Activity Handout For the Crayon A
Crayon Activity Handout For the Crayon ACrayon Activity Handout For the Crayon A
Crayon Activity Handout For the Crayon AUnboundStockton
 
Painted Grey Ware.pptx, PGW Culture of India
Painted Grey Ware.pptx, PGW Culture of IndiaPainted Grey Ware.pptx, PGW Culture of India
Painted Grey Ware.pptx, PGW Culture of IndiaVirag Sontakke
 
भारत-रोम व्यापार.pptx, Indo-Roman Trade,
भारत-रोम व्यापार.pptx, Indo-Roman Trade,भारत-रोम व्यापार.pptx, Indo-Roman Trade,
भारत-रोम व्यापार.pptx, Indo-Roman Trade,Virag Sontakke
 
KSHARA STURA .pptx---KSHARA KARMA THERAPY (CAUSTIC THERAPY)————IMP.OF KSHARA ...
KSHARA STURA .pptx---KSHARA KARMA THERAPY (CAUSTIC THERAPY)————IMP.OF KSHARA ...KSHARA STURA .pptx---KSHARA KARMA THERAPY (CAUSTIC THERAPY)————IMP.OF KSHARA ...
KSHARA STURA .pptx---KSHARA KARMA THERAPY (CAUSTIC THERAPY)————IMP.OF KSHARA ...M56BOOKSTORE PRODUCT/SERVICE
 
Earth Day Presentation wow hello nice great
Earth Day Presentation wow hello nice greatEarth Day Presentation wow hello nice great
Earth Day Presentation wow hello nice greatYousafMalik24
 
Final demo Grade 9 for demo Plan dessert.pptx
Final demo Grade 9 for demo Plan dessert.pptxFinal demo Grade 9 for demo Plan dessert.pptx
Final demo Grade 9 for demo Plan dessert.pptxAvyJaneVismanos
 
Types of Journalistic Writing Grade 8.pptx
Types of Journalistic Writing Grade 8.pptxTypes of Journalistic Writing Grade 8.pptx
Types of Journalistic Writing Grade 8.pptxEyham Joco
 
Presiding Officer Training module 2024 lok sabha elections
Presiding Officer Training module 2024 lok sabha electionsPresiding Officer Training module 2024 lok sabha elections
Presiding Officer Training module 2024 lok sabha electionsanshu789521
 
18-04-UA_REPORT_MEDIALITERAСY_INDEX-DM_23-1-final-eng.pdf
18-04-UA_REPORT_MEDIALITERAСY_INDEX-DM_23-1-final-eng.pdf18-04-UA_REPORT_MEDIALITERAСY_INDEX-DM_23-1-final-eng.pdf
18-04-UA_REPORT_MEDIALITERAСY_INDEX-DM_23-1-final-eng.pdfssuser54595a
 
Capitol Tech U Doctoral Presentation - April 2024.pptx
Capitol Tech U Doctoral Presentation - April 2024.pptxCapitol Tech U Doctoral Presentation - April 2024.pptx
Capitol Tech U Doctoral Presentation - April 2024.pptxCapitolTechU
 

Recently uploaded (20)

DATA STRUCTURE AND ALGORITHM for beginners
DATA STRUCTURE AND ALGORITHM for beginnersDATA STRUCTURE AND ALGORITHM for beginners
DATA STRUCTURE AND ALGORITHM for beginners
 
Biting mechanism of poisonous snakes.pdf
Biting mechanism of poisonous snakes.pdfBiting mechanism of poisonous snakes.pdf
Biting mechanism of poisonous snakes.pdf
 
Model Call Girl in Tilak Nagar Delhi reach out to us at 🔝9953056974🔝
Model Call Girl in Tilak Nagar Delhi reach out to us at 🔝9953056974🔝Model Call Girl in Tilak Nagar Delhi reach out to us at 🔝9953056974🔝
Model Call Girl in Tilak Nagar Delhi reach out to us at 🔝9953056974🔝
 
Roles & Responsibilities in Pharmacovigilance
Roles & Responsibilities in PharmacovigilanceRoles & Responsibilities in Pharmacovigilance
Roles & Responsibilities in Pharmacovigilance
 
History Class XII Ch. 3 Kinship, Caste and Class (1).pptx
History Class XII Ch. 3 Kinship, Caste and Class (1).pptxHistory Class XII Ch. 3 Kinship, Caste and Class (1).pptx
History Class XII Ch. 3 Kinship, Caste and Class (1).pptx
 
9953330565 Low Rate Call Girls In Rohini Delhi NCR
9953330565 Low Rate Call Girls In Rohini  Delhi NCR9953330565 Low Rate Call Girls In Rohini  Delhi NCR
9953330565 Low Rate Call Girls In Rohini Delhi NCR
 
How to Make a Pirate ship Primary Education.pptx
How to Make a Pirate ship Primary Education.pptxHow to Make a Pirate ship Primary Education.pptx
How to Make a Pirate ship Primary Education.pptx
 
Full Stack Web Development Course for Beginners
Full Stack Web Development Course  for BeginnersFull Stack Web Development Course  for Beginners
Full Stack Web Development Course for Beginners
 
ECONOMIC CONTEXT - PAPER 1 Q3: NEWSPAPERS.pptx
ECONOMIC CONTEXT - PAPER 1 Q3: NEWSPAPERS.pptxECONOMIC CONTEXT - PAPER 1 Q3: NEWSPAPERS.pptx
ECONOMIC CONTEXT - PAPER 1 Q3: NEWSPAPERS.pptx
 
Crayon Activity Handout For the Crayon A
Crayon Activity Handout For the Crayon ACrayon Activity Handout For the Crayon A
Crayon Activity Handout For the Crayon A
 
Painted Grey Ware.pptx, PGW Culture of India
Painted Grey Ware.pptx, PGW Culture of IndiaPainted Grey Ware.pptx, PGW Culture of India
Painted Grey Ware.pptx, PGW Culture of India
 
भारत-रोम व्यापार.pptx, Indo-Roman Trade,
भारत-रोम व्यापार.pptx, Indo-Roman Trade,भारत-रोम व्यापार.pptx, Indo-Roman Trade,
भारत-रोम व्यापार.pptx, Indo-Roman Trade,
 
KSHARA STURA .pptx---KSHARA KARMA THERAPY (CAUSTIC THERAPY)————IMP.OF KSHARA ...
KSHARA STURA .pptx---KSHARA KARMA THERAPY (CAUSTIC THERAPY)————IMP.OF KSHARA ...KSHARA STURA .pptx---KSHARA KARMA THERAPY (CAUSTIC THERAPY)————IMP.OF KSHARA ...
KSHARA STURA .pptx---KSHARA KARMA THERAPY (CAUSTIC THERAPY)————IMP.OF KSHARA ...
 
TataKelola dan KamSiber Kecerdasan Buatan v022.pdf
TataKelola dan KamSiber Kecerdasan Buatan v022.pdfTataKelola dan KamSiber Kecerdasan Buatan v022.pdf
TataKelola dan KamSiber Kecerdasan Buatan v022.pdf
 
Earth Day Presentation wow hello nice great
Earth Day Presentation wow hello nice greatEarth Day Presentation wow hello nice great
Earth Day Presentation wow hello nice great
 
Final demo Grade 9 for demo Plan dessert.pptx
Final demo Grade 9 for demo Plan dessert.pptxFinal demo Grade 9 for demo Plan dessert.pptx
Final demo Grade 9 for demo Plan dessert.pptx
 
Types of Journalistic Writing Grade 8.pptx
Types of Journalistic Writing Grade 8.pptxTypes of Journalistic Writing Grade 8.pptx
Types of Journalistic Writing Grade 8.pptx
 
Presiding Officer Training module 2024 lok sabha elections
Presiding Officer Training module 2024 lok sabha electionsPresiding Officer Training module 2024 lok sabha elections
Presiding Officer Training module 2024 lok sabha elections
 
18-04-UA_REPORT_MEDIALITERAСY_INDEX-DM_23-1-final-eng.pdf
18-04-UA_REPORT_MEDIALITERAСY_INDEX-DM_23-1-final-eng.pdf18-04-UA_REPORT_MEDIALITERAСY_INDEX-DM_23-1-final-eng.pdf
18-04-UA_REPORT_MEDIALITERAСY_INDEX-DM_23-1-final-eng.pdf
 
Capitol Tech U Doctoral Presentation - April 2024.pptx
Capitol Tech U Doctoral Presentation - April 2024.pptxCapitol Tech U Doctoral Presentation - April 2024.pptx
Capitol Tech U Doctoral Presentation - April 2024.pptx
 

Pediatric board review/certified fixed orthodontic courses by Indian dental academy

  • 1. Pediatric Board ReviewPediatric Board Review 2006-20072006-2007 Session One: July 24, 2006Session One: July 24, 2006 Session Two: August 24, 2006Session Two: August 24, 2006 Session Three: September 26, 2006Session Three: September 26, 2006 www.indiandentalacademy.comwww.indiandentalacademy.com INDIAN DENTAL ACADEMY Leader in continuing dental education www.indiandentalacademy.com
  • 2. BibliographyBibliography  Blueprints in Pediatrics 1998Blueprints in Pediatrics 1998  Nelson’s Pediatrics 17Nelson’s Pediatrics 17thth EditionEdition  PREP Questions 1999-2006PREP Questions 1999-2006  Up To DateUp To Date www.indiandentalacademy.comwww.indiandentalacademy.com
  • 3. Scrabble QuizScrabble Quiz AEINRSTAEINRST  This is the most common seven letterThis is the most common seven letter combination you can have on your rack. Therecombination you can have on your rack. There are 8 different seven letter words “bingos” thatare 8 different seven letter words “bingos” that can be spelled from this. Can you name one?can be spelled from this. Can you name one? – Anestri – Nastier – Ratines – Retains – Retinas – Retsina – Stainer – Stearin www.indiandentalacademy.comwww.indiandentalacademy.com
  • 4. Scrabble QuizScrabble Quiz  There are 101 acceptable two letter wordThere are 101 acceptable two letter word plays in Scrabble, 16 of which start withplays in Scrabble, 16 of which start with what letter?what letter?  AA  AA, AB, AD, AE, AG, AH, AI, AL, AM, AN,AA, AB, AD, AE, AG, AH, AI, AL, AM, AN, AR, AS, AT, AW, AX, AYAR, AS, AT, AW, AX, AY www.indiandentalacademy.comwww.indiandentalacademy.com
  • 5. Scrabble QuizScrabble Quiz  The word ROOST has 4 anagrams, nameThe word ROOST has 4 anagrams, name them.them. – ROOTSROOTS – ROTOSROTOS – TOROSTOROS – TORSOTORSO www.indiandentalacademy.comwww.indiandentalacademy.com
  • 6. Question 1.2000Question 1.2000  A TRUE statement about the epidemiology ofA TRUE statement about the epidemiology of measles is thatmeasles is that – A. In countries with no immunization programs, theA. In countries with no immunization programs, the peak age of infection is infancypeak age of infection is infancy – B. In US immunization programs have reduced theB. In US immunization programs have reduced the incidence of infection by 80%incidence of infection by 80% – C. Usually spread by direct contact with infectiousC. Usually spread by direct contact with infectious dropletsdroplets – D. Patients become contagious when the rashD. Patients become contagious when the rash appearsappears – E. Incubation period is 4-5 days from exposure toE. Incubation period is 4-5 days from exposure to onset of symptomsonset of symptoms  Answer CAnswer C www.indiandentalacademy.comwww.indiandentalacademy.com
  • 7. Teaching Points 1.2000Teaching Points 1.2000  MeaslesMeasles – Cough, coryza (inflammation of nasalCough, coryza (inflammation of nasal mucosal membranes), fever, conjunctivitis,mucosal membranes), fever, conjunctivitis, exanthem of red macules and papules andexanthem of red macules and papules and Koplik spotsKoplik spots – Young children have OM, pneumonia, croupYoung children have OM, pneumonia, croup and diarrheaand diarrhea – Acute encephalitis (1:1000) -->PermanentAcute encephalitis (1:1000) -->Permanent brain injurybrain injury – In US death 1-3/1000 due to respiratory ofIn US death 1-3/1000 due to respiratory of neurologic complicationsneurologic complications www.indiandentalacademy.comwww.indiandentalacademy.com
  • 8. Teaching Points 1.2000Teaching Points 1.2000  Transmitted by direct contact with infectiousTransmitted by direct contact with infectious droplets ordroplets or LESS COMMONLYLESS COMMONLY airborne spreadairborne spread  Since vaccine use in 1963 there has been aSince vaccine use in 1963 there has been a 99% reduction99% reduction in incidence in USin incidence in US  Children are contagiousChildren are contagious 4-5 days BEFORE rash4-5 days BEFORE rash appears to 4 days after appearance of rashappears to 4 days after appearance of rash  Incubation period is 8-12 days from exposure toIncubation period is 8-12 days from exposure to onset of symptomsonset of symptoms www.indiandentalacademy.comwww.indiandentalacademy.com
  • 10. Koplik SpotsKoplik Spots Henry Koplik (U.S. Physician, 1858 to 1927)Henry Koplik (U.S. Physician, 1858 to 1927) www.indiandentalacademy.comwww.indiandentalacademy.com
  • 11. Question 2.2000Question 2.2000  A previously healthy 2 year old is brought to EDA previously healthy 2 year old is brought to ED because her mom is unable to awaken her forbecause her mom is unable to awaken her for 45 minutes. She has not been ill. PE reveals an45 minutes. She has not been ill. PE reveals an afebrile, hypotonic child who withdraws her handafebrile, hypotonic child who withdraws her hand from painful stimuli but does not spontaneouslyfrom painful stimuli but does not spontaneously open her eyes. RR is 36/min, BP is 92/64. Whatopen her eyes. RR is 36/min, BP is 92/64. What is the next best thing to obtain?is the next best thing to obtain? – A. BUNA. BUN – B. CXRB. CXR – C. EKGC. EKG – D. EEGD. EEG – E. Toxicology screenE. Toxicology screen  Answer EAnswer E www.indiandentalacademy.comwww.indiandentalacademy.com
  • 12. Teaching Points 2.2000Teaching Points 2.2000  Pneumonic for Causes of AlteredPneumonic for Causes of Altered Consciousness: “Tips from the Vowels”Consciousness: “Tips from the Vowels”  T-I-P-S-A-E-I-O-UT-I-P-S-A-E-I-O-U – Trauma/TumorTrauma/Tumor – Infection/Inborn Errors/Insulin/hypoglycemiaInfection/Inborn Errors/Insulin/hypoglycemia – PoisonsPoisons – ShockShock – Alcohol/AbuseAlcohol/Abuse – Epilepsy/EncephalopathyEpilepsy/Encephalopathy – IntussusceptionIntussusception – OpiatesOpiates – UremiaUremia www.indiandentalacademy.comwww.indiandentalacademy.com
  • 13. Question 3.2000Question 3.2000  You have intubated an 8 month old with sepsis and apneaYou have intubated an 8 month old with sepsis and apnea with a 4.0 endotracheal tube. Proper placement is confirmedwith a 4.0 endotracheal tube. Proper placement is confirmed by observing chest rise and auscultating symmetric breathby observing chest rise and auscultating symmetric breath sounds after bag and mask ventilation. Perfusion is <3sounds after bag and mask ventilation. Perfusion is <3 seconds, and heart rate is 120/minute. Five minutes laterseconds, and heart rate is 120/minute. Five minutes later the RT tells you the oxygen saturation is 83% and the bloodthe RT tells you the oxygen saturation is 83% and the blood pressure and pulse are dropping. Breath sounds are absentpressure and pulse are dropping. Breath sounds are absent on the right and the right chest is hypertympanitic. What ison the right and the right chest is hypertympanitic. What is the most likely diagnosis?the most likely diagnosis? – A. Esophageal intubationA. Esophageal intubation – B. Incorrect ET sizeB. Incorrect ET size – C. Obstructed ET tubeC. Obstructed ET tube – D. Right main stem bronchus intubationD. Right main stem bronchus intubation – E. Right pneumothoraxE. Right pneumothorax  Answer EAnswer E www.indiandentalacademy.comwww.indiandentalacademy.com
  • 14. Teaching Points 3.2000Teaching Points 3.2000  ET Tube Size (mm) = …ET Tube Size (mm) = … – [16 + Age(yr)]/4[16 + Age(yr)]/4 – Preterm 2.5-3.0Preterm 2.5-3.0 – Term 3.0-3.5Term 3.0-3.5 – Infant 3.5-4.0Infant 3.5-4.0 – 1 year 4.0-4.51 year 4.0-4.5 – 3 year 4.5-5.03 year 4.5-5.0 – *An uncuffed tube should be used in patients…*An uncuffed tube should be used in patients…  <8 years of age<8 years of age  Approximate distance of insertion measured at teethApproximate distance of insertion measured at teeth or lips in cm = …or lips in cm = … – Internal Diameter X 3Internal Diameter X 3  Straight blades preferred for neonates and infantsStraight blades preferred for neonates and infants www.indiandentalacademy.comwww.indiandentalacademy.com
  • 15. Teaching Points 3.2000Teaching Points 3.2000  Drugs that can be given by ETT after dilution withDrugs that can be given by ETT after dilution with normal saline to 3-5 ml followed by positivenormal saline to 3-5 ml followed by positive pressure ventilation…pressure ventilation…  LANELANE – LidocaineLidocaine – AtropineAtropine  0.02mg/kg IV,IO; 0.02-0.06mg/kg ET0.02mg/kg IV,IO; 0.02-0.06mg/kg ET  Minimum dose is 1ml (0.1 mg) as it comes 0.1 mg/mlMinimum dose is 1ml (0.1 mg) as it comes 0.1 mg/ml – Naloxone (Narcan)Naloxone (Narcan)  0.1 mg/kg/dose IM/ET/IV/IO to maximum 2mg/dose. May repeat0.1 mg/kg/dose IM/ET/IV/IO to maximum 2mg/dose. May repeat every 2-3 minutesevery 2-3 minutes – EpinephrineEpinephrine  0.1ml/kg of 1:1000 (0.1mg/kg) IV, IO, ET in non-neonates0.1ml/kg of 1:1000 (0.1mg/kg) IV, IO, ET in non-neonates  Use 1:10,000 (0.1-0.3ml/kg for all doses and routes) epinephrineUse 1:10,000 (0.1-0.3ml/kg for all doses and routes) epinephrine via ETT for neonates only. Use high dose 1:1,000 epinephrine forvia ETT for neonates only. Use high dose 1:1,000 epinephrine for ETT beyond neonatal periodETT beyond neonatal period www.indiandentalacademy.comwww.indiandentalacademy.com
  • 18. Teaching Points 3.2000Teaching Points 3.2000  Esophageal intubation essentially ruled outEsophageal intubation essentially ruled out because of previous presence of rise and fall ofbecause of previous presence of rise and fall of chest and presence of breath sounds unilaterallychest and presence of breath sounds unilaterally  However, ET tube could have moved if theyHowever, ET tube could have moved if they repositioned patient; check depth marker atrepositioned patient; check depth marker at teeth/lipsteeth/lips  If ETT tube is too small, air leaking around tubeIf ETT tube is too small, air leaking around tube can impair efforts to ventilate but an audible aircan impair efforts to ventilate but an audible air leak generally is heard with each positiveleak generally is heard with each positive pressure breathpressure breath www.indiandentalacademy.comwww.indiandentalacademy.com
  • 19. Teaching Points 3.2000Teaching Points 3.2000  Treatment for tension pneumothoraxTreatment for tension pneumothorax – Immediate needle decompressionImmediate needle decompression – Patient supine with head of bed at 30 degreePatient supine with head of bed at 30 degree – 18-20 gauge over the needle catheter (angiocatheter)18-20 gauge over the needle catheter (angiocatheter) inserted into the…inserted into the…  22ndnd intercostal space just “over the top” (Hey dude, this procedureintercostal space just “over the top” (Hey dude, this procedure is over the top , man!) of the 3is over the top , man!) of the 3rdrd rib at the midclavicular linerib at the midclavicular line – 5-10 ml syringe attached to angiocatheter and aspirated5-10 ml syringe attached to angiocatheter and aspirated gently as needle is advancedgently as needle is advanced – Loss of resistance or rush of air apparent as soon asLoss of resistance or rush of air apparent as soon as pleural space is enteredpleural space is entered – If pneumothorax is confirmed a one-way drainage deviceIf pneumothorax is confirmed a one-way drainage device should be attachedshould be attached – A chest tube should be placed after successful needleA chest tube should be placed after successful needle decompressiondecompression www.indiandentalacademy.comwww.indiandentalacademy.com
  • 21. Question 4.2000Question 4.2000  A 6 year old boy with severe factor VIII deficiencyA 6 year old boy with severe factor VIII deficiency hemophilia develops increased swelling of the righthemophilia develops increased swelling of the right distal forearm. There is not history of trauma. Thedistal forearm. There is not history of trauma. The peripheral circulation is normal and there is no jointperipheral circulation is normal and there is no joint involvement. The MOST important complication ofinvolvement. The MOST important complication of bleeding in this location is:bleeding in this location is: – A. Blood lossA. Blood loss – B. Muscular DamageB. Muscular Damage – C. Neurologic ImpairmentC. Neurologic Impairment – D. Tendon ShorteningD. Tendon Shortening – E. Vascular DamageE. Vascular Damage  Answer CAnswer C www.indiandentalacademy.comwww.indiandentalacademy.com
  • 22. Teaching Points 4.2000Teaching Points 4.2000  Children with hemophilia can develop severeChildren with hemophilia can develop severe peripheral neurologic deficits when hematomasperipheral neurologic deficits when hematomas compress nerves via compartment syndromecompress nerves via compartment syndrome  In older children increasing pain out of proportionIn older children increasing pain out of proportion to size of hematoma, numbness and paresis areto size of hematoma, numbness and paresis are critical signscritical signs  Often there is no history of trauma inOften there is no history of trauma in hemophiliacshemophiliacs  Significant blood loss seen in hematomas of…Significant blood loss seen in hematomas of… – Thigh or retroperitoneumThigh or retroperitoneum www.indiandentalacademy.comwww.indiandentalacademy.com
  • 23. Teaching Points 4.2000Teaching Points 4.2000  Routine venipuncture is safe as long as it isRoutine venipuncture is safe as long as it is followed by…followed by… – 5 minutes of firm finger pressure5 minutes of firm finger pressure  Femoral or jugular venipuncture or arterialFemoral or jugular venipuncture or arterial puncture should not be undertaken WITHOUT…puncture should not be undertaken WITHOUT… – PRIOR FACTOR REPLACEMENTPRIOR FACTOR REPLACEMENT  What about IM immunizations…What about IM immunizations… – Ok as long as followed by 5 minutes of firm fingerOk as long as followed by 5 minutes of firm finger pressurepressure – Large IM injections should be avoided (Decadron,Large IM injections should be avoided (Decadron, Rocephin IM etc)Rocephin IM etc) www.indiandentalacademy.comwww.indiandentalacademy.com
  • 24. Teaching Points 4.2000Teaching Points 4.2000  Tendon shortening only occurs with…Tendon shortening only occurs with… – Chronic, sever hemarthrosesChronic, sever hemarthroses – Limited mobilityLimited mobility – Nerve damageNerve damage  Muscular damage is not frequent amongMuscular damage is not frequent among children with hemophiliachildren with hemophilia www.indiandentalacademy.comwww.indiandentalacademy.com
  • 26. Hematoma after IM injectionHematoma after IM injection www.indiandentalacademy.comwww.indiandentalacademy.com
  • 27. Question 5.2000Question 5.2000  An ambulance team brings a 6 year old girl in anAn ambulance team brings a 6 year old girl in an MVA to the ED. She had been unrestrained in theMVA to the ED. She had been unrestrained in the back seat. The girl is unresponsive on arrival and isback seat. The girl is unresponsive on arrival and is bleeding profusely from a scalp wound. Herbleeding profusely from a scalp wound. Her Glasgow Coma Scale is 3. What is the BEST initialGlasgow Coma Scale is 3. What is the BEST initial step in evaluation and management?step in evaluation and management? – A. Control profuse scalp bleedingA. Control profuse scalp bleeding – B. Establish IV accessB. Establish IV access – C. Order portable cervical spine radiographsC. Order portable cervical spine radiographs – D. Remove all clothingD. Remove all clothing – E. Secure an adequate airwayE. Secure an adequate airway  Answer E.Answer E. www.indiandentalacademy.comwww.indiandentalacademy.com
  • 28. Teaching Points 5.2000Teaching Points 5.2000  Priority during primary surveyPriority during primary survey – AirwayAirway  Failure to provideFailure to provide oxygenated blood to theoxygenated blood to the brain and other vital organsbrain and other vital organs is the quickest killer of ais the quickest killer of a trauma victimtrauma victim – BreathingBreathing – CirculationCirculation – DisabilityDisability – ExposureExposure  Any victim of head trauma with aAny victim of head trauma with a GCS of ___ or less should beGCS of ___ or less should be intubated immediatelyintubated immediately – 88  X-rays for cervical fractures…X-rays for cervical fractures… – AP, Lateral and odontoid viewsAP, Lateral and odontoid views Dens of Axis C-2Atlas C-1 Odontoid Viewwww.indiandentalacademy.comwww.indiandentalacademy.com
  • 29. Teaching Points 5.2000Teaching Points 5.2000 Glasgow Coma ScalesGlasgow Coma Scales INFANTS & TODDLERS CHILDREN’S & ADULTS EYE OPENING Spontaneous 4 EYE OPENING Spontaneous 4 To Voice 3 To Voice 3 To Pain 2 To Pain 2 None 1 None 1 BEST VERBAL RESPONSE Smiles, Interacts 5 BEST VERBAL RESPONSE Oriented 5 Consolable 4 Confused 4 Cries to Pain 3 Inappropriate Words 3 Moans to Pain 2 Incomprehensible Words 2 None 1 None 1 BEST MOTOR RESPONSE Normal spontaneous movement 6 BEST MOTOR RESPONSE Obeys Commands 6 Localizes Pain 5 Localizes Pain 5 Withdraws to Pain 4 Withdraws (Pain) 4 Abnormal Flexion 3 Flexion (Pain) 3 Abnormal Extension 2 Extension (Pain) 2 None 1 None 1www.indiandentalacademy.comwww.indiandentalacademy.com
  • 30. Question 6.2000Question 6.2000  A 2 week old presents with tachypnea, poorA 2 week old presents with tachypnea, poor perfusion, gallop rhythm, diminished pulses, andperfusion, gallop rhythm, diminished pulses, and hepatomegaly. ABG shows metabolic acidosis.hepatomegaly. ABG shows metabolic acidosis. Echocardiography reveals critical AorticEchocardiography reveals critical Aortic Stenosis. What intervention is most likely toStenosis. What intervention is most likely to stabilize the infant’s condition?stabilize the infant’s condition? – A. DobutamineA. Dobutamine – B. EpinephrineB. Epinephrine – C. Nitric OxideC. Nitric Oxide – D. 100% oxygenD. 100% oxygen – E. Prostaglandin E-1E. Prostaglandin E-1  Answer E.Answer E. www.indiandentalacademy.comwww.indiandentalacademy.com
  • 31. Teaching Points 6.2000Teaching Points 6.2000  Severe CHF and cardiogenic shock inSevere CHF and cardiogenic shock in neonate, think LEFT SIDED LESIONSneonate, think LEFT SIDED LESIONS – Hypoplastic left heartHypoplastic left heart – Critical congenital aortic stenosisCritical congenital aortic stenosis – Critical neonatal coarctation of aortaCritical neonatal coarctation of aorta  As ductus arteriosus closes, earlyAs ductus arteriosus closes, early compensatory RV contribution tocompensatory RV contribution to systemic blood flow and perfusion tosystemic blood flow and perfusion to kidneys and other organs is lostkidneys and other organs is lost rapidly developing severe metabolicrapidly developing severe metabolic acidosisacidosis  myocardial and organmyocardial and organ dysfunctiondysfunction  Therefore, need to keep duct OPENTherefore, need to keep duct OPEN with PGEwith PGE11 (alprostadil) EVEN TO AN(alprostadil) EVEN TO AN INFANT 1-2INFANT 1-2 MONTHSMONTHS OF AGEOF AGE www.indiandentalacademy.comwww.indiandentalacademy.com
  • 32. Teaching Points 6.2000Teaching Points 6.2000  Inotropic agents such as dobutamine orInotropic agents such as dobutamine or epinephrine may provide supportive therapy butepinephrine may provide supportive therapy but will not be lifesavingwill not be lifesaving  Nitric oxide is a pulmonary dilator used in PPHNitric oxide is a pulmonary dilator used in PPH of newborn but is contraindicated in obstructiveof newborn but is contraindicated in obstructive left sided heart lesionsleft sided heart lesions  Serious hypoxia is NOT a problem for infantsSerious hypoxia is NOT a problem for infants with obstructive left sided lesionswith obstructive left sided lesions  Arterial pO2 may be normal initially in a sickArterial pO2 may be normal initially in a sick neonateneonate  As PDA is opened the O2 will drop due to rightAs PDA is opened the O2 will drop due to right ventricular flow to the bodyventricular flow to the body www.indiandentalacademy.comwww.indiandentalacademy.com
  • 33. Teaching Points 6.2000Teaching Points 6.2000  Attempts to keep O2 HIGH ARE DETRIMENTAL asAttempts to keep O2 HIGH ARE DETRIMENTAL as high inspired oxygen levels causes powerfulhigh inspired oxygen levels causes powerful PULMONARY ARTERIAL DILATION which “steals”PULMONARY ARTERIAL DILATION which “steals” RV output to the pulmonary arteries and away fromRV output to the pulmonary arteries and away from the bodythe body  This “steal” factor NOT o2 induced ductal closure isThis “steal” factor NOT o2 induced ductal closure is the primary risk associated with use ofthe primary risk associated with use of supplemental O2 when PGE1 is used to keep ductsupplemental O2 when PGE1 is used to keep duct openopen  Some degree of pulmonary hypertension (i.e. small,Some degree of pulmonary hypertension (i.e. small, tight, non-dilated arteries) is essential to promotetight, non-dilated arteries) is essential to promote systemic flow to the peripheral circulation via thesystemic flow to the peripheral circulation via the ductusductus www.indiandentalacademy.comwww.indiandentalacademy.com
  • 34. Teaching Points 6.2000Teaching Points 6.2000  Hypoplastic Left HeartHypoplastic Left Heart – 22ndnd most common congenitalmost common congenital cardiac lesion presenting in 1cardiac lesion presenting in 1stst week of lifeweek of life – Most common cause of deathMost common cause of death from CHD in 1from CHD in 1stst month of lifemonth of life  What are characteristics?What are characteristics? – Hypoplasia of left ventricleHypoplasia of left ventricle – Hypoplasia of aortic rootHypoplasia of aortic root – Aortic valve atresiaAortic valve atresia – Critical mitral valve stenosis or atresiaCritical mitral valve stenosis or atresia  Result is a reduction or eliminationResult is a reduction or elimination of blood flow through left side ofof blood flow through left side of heartheart www.indiandentalacademy.comwww.indiandentalacademy.com
  • 35. Hypoplastic Left HeartHypoplastic Left Heart  Ineffective left side of heartIneffective left side of heart  obligatory left to right shunt where?obligatory left to right shunt where? – Atrial levelAtrial level  And right to left shunting where?And right to left shunting where? – Ductus arteriosusDuctus arteriosus  Systemic flow isSystemic flow is completely ductalcompletely ductal dependentdependent  How do coronary arteries get fed?How do coronary arteries get fed? – Retrograde coronary perfusionRetrograde coronary perfusion  As PDA closes neonates becomeAs PDA closes neonates become critically illcritically ill  CHF, cyanotic,CHF, cyanotic, tachycardic, tachypneic, ralestachycardic, tachypneic, rales (crackles) from pulmonary edema,(crackles) from pulmonary edema, and hepatomegaly; poor peripheraland hepatomegaly; poor peripheral pulses with vasoconstrictedpulses with vasoconstricted extremities.extremities. www.indiandentalacademy.comwww.indiandentalacademy.com
  • 36. Hypoplastic Left HeartHypoplastic Left Heart  Cardiac ExamCardiac Exam – Loud S2 (Marks beginningLoud S2 (Marks beginning of diastole and is due toof diastole and is due to closure of semilunarclosure of semilunar valves; primarily pulmonicvalves; primarily pulmonic valve slamming againstvalve slamming against increased blood volumeincreased blood volume and pressure)and pressure) – S3 (Early diastoleS3 (Early diastole corresponds with end ofcorresponds with end of first phase of rapidfirst phase of rapid ventricular filling from atriaventricular filling from atria and can be normal inand can be normal in children and young adults)children and young adults) www.indiandentalacademy.comwww.indiandentalacademy.com
  • 37. Hypoplastic Left HeartHypoplastic Left Heart  Treatment:Treatment: – PGEPGE11 (alprostadil) immediately to maintain ductal(alprostadil) immediately to maintain ductal dependent systemic blood flowdependent systemic blood flow – Cardiac transplantation in the newborn period isCardiac transplantation in the newborn period is primary treatment for HLHSprimary treatment for HLHS  Scarcity of newborn organs available for transplantationScarcity of newborn organs available for transplantation  Life-long need for anti-rejection therapyLife-long need for anti-rejection therapy  Average life span of the transplanted heart is limitedAverage life span of the transplanted heart is limited (currently less than 15 years).(currently less than 15 years). – Most common treatment for HLHS is palliativeMost common treatment for HLHS is palliative "staged reconstruction""staged reconstruction" www.indiandentalacademy.comwww.indiandentalacademy.com
  • 38. Hypoplastic Left HeartHypoplastic Left Heart – 1. Norwood operation: performed 11. Norwood operation: performed 1stst weekweek  RV becomes the systemic or main ventricleRV becomes the systemic or main ventricle pumping to the body.pumping to the body.  ““Neoaorta” is made from part of the pulmonaryNeoaorta” is made from part of the pulmonary artery and the original, tiny aorta, is enlarged toartery and the original, tiny aorta, is enlarged to provide blood flow to the body.provide blood flow to the body.  To provide blood flow to lungs, a small tube graft isTo provide blood flow to lungs, a small tube graft is placed either from the left subclavian or leftplaced either from the left subclavian or left innominate artery to the lung vessels (modifiedinnominate artery to the lung vessels (modified Blalock-Taussig shunt: or from the RV to the lungBlalock-Taussig shunt: or from the RV to the lung vessels (Sano modification).vessels (Sano modification). www.indiandentalacademy.comwww.indiandentalacademy.com
  • 39. Hypoplastic Left HeartHypoplastic Left Heart – 2. Bi-directional Glenn procedure2. Bi-directional Glenn procedure  3 to 6 months of age3 to 6 months of age  SVC is taken off the heart and sewn directly to theSVC is taken off the heart and sewn directly to the pulmonary arterypulmonary artery – 3. Fontan operation3. Fontan operation  2 or 3 years of age2 or 3 years of age  IVC is connected directly the blood from theIVC is connected directly the blood from the pulmonary arteriespulmonary arteries www.indiandentalacademy.comwww.indiandentalacademy.com
  • 40. Question 7.2000Question 7.2000  During a routine visit, a 14 year old overweightDuring a routine visit, a 14 year old overweight patient tells you he watches more than 25 hours ofpatient tells you he watches more than 25 hours of TV per week. What is the most appropriate advice?TV per week. What is the most appropriate advice? – A. Parents should limit TV to no more than 2 hours/dayA. Parents should limit TV to no more than 2 hours/day – B. TV commercials have little impact on selection of toysB. TV commercials have little impact on selection of toys and foodand food – C. Average adolescent spends 15 hours/week watchingC. Average adolescent spends 15 hours/week watching TVTV – D. Unclear relationship between TV viewing and risk ofD. Unclear relationship between TV viewing and risk of obesityobesity – E. No relationship between TV viewing and snackingE. No relationship between TV viewing and snacking  ANSWER AANSWER A www.indiandentalacademy.comwww.indiandentalacademy.com
  • 41. Teaching Points 7.2000Teaching Points 7.2000  Children 2-11 22 h/weekChildren 2-11 22 h/week  Adolescents 12-17 20 h/weekAdolescents 12-17 20 h/week  Age 18, adolescent will have seen 200,000 violentAge 18, adolescent will have seen 200,000 violent acts and 18,000 murders on TVacts and 18,000 murders on TV  Encourage parents to “co-view” programs withEncourage parents to “co-view” programs with childrenchildren  Saturday morning commercials; 50% food, 33%Saturday morning commercials; 50% food, 33% toystoys  20 best selling toys are from TV shows based on20 best selling toys are from TV shows based on the toysthe toys  Increased TV viewing significant factor leading toIncreased TV viewing significant factor leading to obesityobesity  Increased TV viewing = increased snackingIncreased TV viewing = increased snacking www.indiandentalacademy.comwww.indiandentalacademy.com
  • 42. Deep Thoughts from RyanDeep Thoughts from Ryan  ““Dad, how long doesDad, how long does it take a booger toit take a booger to form?”form?”  Booger: Dried nasalBooger: Dried nasal mucus; bogie inmucus; bogie in EnglandEngland  Medical Term:Medical Term: – RhinolithsRhinoliths www.indiandentalacademy.comwww.indiandentalacademy.com
  • 43. Question 8.2000Question 8.2000  A male infant born at 36 weeks gestation had a leftA male infant born at 36 weeks gestation had a left testicle palpable in the inguinal canal. At 12 monthstesticle palpable in the inguinal canal. At 12 months of age, the left testicle has failed to descend into theof age, the left testicle has failed to descend into the scrotum. What is most appropriate care for thisscrotum. What is most appropriate care for this infant?infant? – A. Observation until 2 years of ageA. Observation until 2 years of age – B. OrchiopexyB. Orchiopexy – C. Radionuclide scan of left testicleC. Radionuclide scan of left testicle – D. Treatment with human chorionic gonadotropinD. Treatment with human chorionic gonadotropin – E. Treatment with testosteroneE. Treatment with testosterone  ANSWER BANSWER B www.indiandentalacademy.comwww.indiandentalacademy.com
  • 44. Teaching Points 8.2000Teaching Points 8.2000  What percent of preterm male infants have anWhat percent of preterm male infants have an undescended testisundescended testis – 30%30%  Spontaneous descent into scrotum occurs…Spontaneous descent into scrotum occurs… – over next 3-6 monthsover next 3-6 months  What is the most reliable method to localize theWhat is the most reliable method to localize the undescended testis?undescended testis? – LaparoscopyLaparoscopy  Orchiopexy, a surgery which places the testis intoOrchiopexy, a surgery which places the testis into the scrotum is indicated for a testis that fails tothe scrotum is indicated for a testis that fails to descend by…descend by… – 6 months of age and is performed at 9-12 months of age6 months of age and is performed at 9-12 months of age www.indiandentalacademy.comwww.indiandentalacademy.com
  • 45. Teaching Points 8.2000Teaching Points 8.2000  At 6-12 months of age histological changesAt 6-12 months of age histological changes representing degeneration of the seminiferousrepresenting degeneration of the seminiferous tubules occurstubules occurs decrease in quality and quantitydecrease in quality and quantity of spermatogenesisof spermatogenesis which is progressivewhich is progressive  The longer the testis remains in its improperThe longer the testis remains in its improper location the greater the fertility impairmentlocation the greater the fertility impairment  What percent of all testicular tumors occur inWhat percent of all testicular tumors occur in patients with an undescended testiclepatients with an undescended testicle – 10%10%  Orchiopexy improves fertility but DOES NOTOrchiopexy improves fertility but DOES NOT change malignancy riskchange malignancy risk www.indiandentalacademy.comwww.indiandentalacademy.com
  • 46. Teaching Points 8.2000Teaching Points 8.2000  Relocated testis has aRelocated testis has a 35-48 times35-48 times greatergreater risk of malignancy than normal testisrisk of malignancy than normal testis  Does the normal testis also have anDoes the normal testis also have an increased risk?increased risk? – YES, just not as highYES, just not as high  Counsel parents to:Counsel parents to: – Seek immediate attention for acute testicularSeek immediate attention for acute testicular pain due to risk of torsionpain due to risk of torsion – Perform monthly examination of both testesPerform monthly examination of both testes www.indiandentalacademy.comwww.indiandentalacademy.com
  • 47. Teaching Points 8.2000Teaching Points 8.2000  Retractile testiclesRetractile testicles – Bilateral mostlyBilateral mostly – Found in children >1 year of age, 5-6 yearFound in children >1 year of age, 5-6 year olds strong reflexolds strong reflex – Due to strong cremasteric reflexDue to strong cremasteric reflex – ““Milk them” into scrotumMilk them” into scrotum – Warm room, frog leg position can makeWarm room, frog leg position can make examination easierexamination easier www.indiandentalacademy.comwww.indiandentalacademy.com
  • 48. Question 9.2000Question 9.2000  A newborn female has an open neural tubeA newborn female has an open neural tube defect, low set ears, VSD, and rib anddefect, low set ears, VSD, and rib and vertebral column malformations. Which ofvertebral column malformations. Which of the following MATERNAL conditions wasthe following MATERNAL conditions was most likely present during pregnancy?most likely present during pregnancy?  A. AlcoholismA. Alcoholism  B. Diabetes mellitusB. Diabetes mellitus  C. HypothyroidismC. Hypothyroidism  D. Iodine deficiencyD. Iodine deficiency  E. SyphilisE. Syphilis – ANSWER: BANSWER: Bwww.indiandentalacademy.comwww.indiandentalacademy.com
  • 49. Teaching Points 9.2000Teaching Points 9.2000  Early prenatal deficits: malformations (3 foldEarly prenatal deficits: malformations (3 fold increase), growth deficiency, stillbirthincrease), growth deficiency, stillbirth  The worse the diabetic control the moreThe worse the diabetic control the more severe the defectssevere the defects  Cardiac: VSD, TGA, dextrocardiaCardiac: VSD, TGA, dextrocardia  CNS: Anencephaly,CNS: Anencephaly, holoprosencephalyholoprosencephaly ,, spina bifida, hydrocephalus,spina bifida, hydrocephalus, caudalcaudal regression syndromeregression syndrome  Rib defectsRib defects www.indiandentalacademy.comwww.indiandentalacademy.com
  • 50. Teaching Points 9.2000Teaching Points 9.2000  Infants present with macrosomia due toInfants present with macrosomia due to hyperinsulinemia and excessive glucosehyperinsulinemia and excessive glucose availabilityavailability  Both linear growth and weight are affectedBoth linear growth and weight are affected  If mom has severe vascular disease, canIf mom has severe vascular disease, can result in IUGRresult in IUGR  Additional complications: Hyperbilirubinemia,Additional complications: Hyperbilirubinemia, hypoglycemia, vascular thromboses,hypoglycemia, vascular thromboses, respiratory distress, birth injury due torespiratory distress, birth injury due to macrosomiamacrosomia www.indiandentalacademy.comwww.indiandentalacademy.com
  • 51. HoloprosencephalyHoloprosencephaly  Caused by a failure of theCaused by a failure of the embryo's forebrain to divide toembryo's forebrain to divide to form bilateral cerebralform bilateral cerebral hemispheres causing defects inhemispheres causing defects in face development face and inface development face and in brain structure and function.brain structure and function.  The "alobar" form ofThe "alobar" form of holoprosencephaly is shown hereholoprosencephaly is shown here in which there is a single largein which there is a single large ventricle, because there is noventricle, because there is no attempt to form separate cerebralattempt to form separate cerebral hemispheres.hemispheres.  May be associated with trisomy 13May be associated with trisomy 13 and rarely in association withand rarely in association with maternal diabetes mellitus.maternal diabetes mellitus. www.indiandentalacademy.comwww.indiandentalacademy.com
  • 54. Teaching Points 9.2000Teaching Points 9.2000  Fetal alcohol syndromeFetal alcohol syndrome – What is the most common teratogen to which fetusesWhat is the most common teratogen to which fetuses are exposed?are exposed?  AlcoholAlcohol – Which U.S. population has the highest incidence ofWhich U.S. population has the highest incidence of children with FAS?children with FAS?  Native AmericansNative Americans – What is the incidence in U.S. (excluding NativeWhat is the incidence in U.S. (excluding Native Americans)Americans)  1:10001:1000 – Affects 40% of children in women who drink moreAffects 40% of children in women who drink more than 4-6 drinks per daythan 4-6 drinks per day www.indiandentalacademy.comwww.indiandentalacademy.com
  • 56. Fetal Alcohol SyndromeFetal Alcohol Syndrome www.indiandentalacademy.comwww.indiandentalacademy.com
  • 57. Fetal Alcohol SyndromeFetal Alcohol Syndrome www.indiandentalacademy.comwww.indiandentalacademy.com
  • 58. Fetal Alcohol SyndromeFetal Alcohol Syndrome www.indiandentalacademy.comwww.indiandentalacademy.com
  • 60. Teaching Points 9.2000Teaching Points 9.2000  MaternalMaternal hypothyroidism hashypothyroidism has little effect on fetuslittle effect on fetus which produces itswhich produces its own thyroid hormoneown thyroid hormone  Women withWomen with untreateduntreated hypothyroidism alsohypothyroidism also give birth togive birth to NORMAL babiesNORMAL babies www.indiandentalacademy.comwww.indiandentalacademy.com
  • 61. Teaching Points 9.2000Teaching Points 9.2000  Maternal iodine deficiency (rare in developedMaternal iodine deficiency (rare in developed countries)countries)  fetal deficiencyfetal deficiency – GoiterGoiter – Mental retardationMental retardation – Slightly increased head size due to…Slightly increased head size due to…  Myxedema [“hard” edema due to increased mucinsMyxedema [“hard” edema due to increased mucins (proteoglycans) in the fluid] of subcutaneous tissues of(proteoglycans) in the fluid] of subcutaneous tissues of the brain}the brain} – Hyperbilirubinemia due to…Hyperbilirubinemia due to…  Delayed maturation of glucuronide conjugationDelayed maturation of glucuronide conjugation – Feeding difficulties (choking spells, lack ofFeeding difficulties (choking spells, lack of interest, somnolence, sluggishness)interest, somnolence, sluggishness)www.indiandentalacademy.comwww.indiandentalacademy.com
  • 62. Teaching Points 9.2000Teaching Points 9.2000 – Respiratory difficulties due to…Respiratory difficulties due to…  Large tongue, apneic episodes, noisy respirations, nasalLarge tongue, apneic episodes, noisy respirations, nasal obstructionobstruction – Retarded bone growthRetarded bone growth – ConstipationConstipation – Umbilical hernia with large abdomenUmbilical hernia with large abdomen – Hypothermia and cold and mottled skinHypothermia and cold and mottled skin – Slow pulseSlow pulse – Genital and extremity edemaGenital and extremity edema – Pericardial effusion, murmur, cardiomegalyPericardial effusion, murmur, cardiomegaly  Prompt treatment with iodine necessary toPrompt treatment with iodine necessary to prevent mental retardationprevent mental retardation www.indiandentalacademy.comwww.indiandentalacademy.com
  • 63. Teaching Points 9.2000Teaching Points 9.2000  Maternal syphilisMaternal syphilis – Fetus affected by…Fetus affected by…  Transplacental transmissionTransplacental transmission Treponema pallidumTreponema pallidum – Syphilis in untreated women can be transmitted toSyphilis in untreated women can be transmitted to fetus at any time, fetal transfer most common duringfetus at any time, fetal transfer most common during 11stst year of maternal infectionyear of maternal infection – 2/3 of live-born neonates with congenital syphilis are2/3 of live-born neonates with congenital syphilis are asymptomatic at birth.asymptomatic at birth. – Overt infection can manifest in the fetus, the newborn,Overt infection can manifest in the fetus, the newborn, or later in childhood.or later in childhood. – Clinical manifestations after birth are dividedClinical manifestations after birth are divided arbitrarily into early ( 2 years of age) and late (>2arbitrarily into early ( 2 years of age) and late (>2 years of age)years of age) www.indiandentalacademy.comwww.indiandentalacademy.com
  • 64. Teaching Points 9.2000Teaching Points 9.2000  Fetal manifestations of SyphilisFetal manifestations of Syphilis – StillbirthStillbirth – Neonatal deathNeonatal death – Overt infection at birth, such as hydropsOvert infection at birth, such as hydrops fetalis (abnormal accumulation serous fluid infetalis (abnormal accumulation serous fluid in fetal tissues)fetal tissues) – Intrauterine death in 25% of affected infants,Intrauterine death in 25% of affected infants, with perinatal mortality in an additional 25-with perinatal mortality in an additional 25- 30%, if untreated30%, if untreated www.indiandentalacademy.comwww.indiandentalacademy.com
  • 65. Teaching Points 9.2000Teaching Points 9.2000  Early congenital manifestations ofEarly congenital manifestations of SyphilisSyphilis – Quite variable, appear within the first 5 weeks of lifeQuite variable, appear within the first 5 weeks of life – Cutaneous lesions frequently occur on the palms andCutaneous lesions frequently occur on the palms and soles; if ulcerative in nature, they are highlysoles; if ulcerative in nature, they are highly contagiouscontagious – HepatosplenomegalyHepatosplenomegaly – JaundiceJaundice – AnemiaAnemia – Snuffles (obstructed nasal respiration in newborn)Snuffles (obstructed nasal respiration in newborn) – Metaphyseal dystrophy and periostitis often are notedMetaphyseal dystrophy and periostitis often are noted on radiographs at birthon radiographs at birth www.indiandentalacademy.comwww.indiandentalacademy.com
  • 66. Teaching Points 9.2000Teaching Points 9.2000  Late congenital manifestationsLate congenital manifestations – Develop from scarring related to earlyDevelop from scarring related to early infection but can be prevented by treatment ofinfection but can be prevented by treatment of the infant within the first three months of birththe infant within the first three months of birth – Late findings include frontal bossing, shortLate findings include frontal bossing, short maxilla, high palatal arch, Hutchinson triadmaxilla, high palatal arch, Hutchinson triad (Hutchinson teeth [blunted upper incisors],(Hutchinson teeth [blunted upper incisors], interstitial keratitis, and eighth nerveinterstitial keratitis, and eighth nerve deafness), saddle nose, and perioral fissuresdeafness), saddle nose, and perioral fissures (rhagades)(rhagades) www.indiandentalacademy.comwww.indiandentalacademy.com
  • 67. Saddle Nose deformity in congenital syphilis www.indiandentalacademy.comwww.indiandentalacademy.com
  • 69. Question 10.2000Question 10.2000  A 5 year old male is hospitalized in January withA 5 year old male is hospitalized in January with fever and seizures. LP reveals clear CSF with 47fever and seizures. LP reveals clear CSF with 47 WBCs/mmWBCs/mm33 all of which are lymphocytes. On PEall of which are lymphocytes. On PE he appears obtunded but arouses with painfulhe appears obtunded but arouses with painful stimuli. Neurologic exam reveals no focal findings.stimuli. Neurologic exam reveals no focal findings. Which diagnostic test is most likely to reveal thisWhich diagnostic test is most likely to reveal this child’s illness?child’s illness? – A. Bacterial culture of CSFA. Bacterial culture of CSF – B. PCR test of CSF for HSVB. PCR test of CSF for HSV – C.C. Streptococcus pneumoniaeStreptococcus pneumoniae bacterial antigen test ofbacterial antigen test of CSFCSF – D. Viral culture of CSFD. Viral culture of CSF – E. Viral culture of nasopharyngeal and rectal swabsE. Viral culture of nasopharyngeal and rectal swabs  ANSWER BANSWER B www.indiandentalacademy.comwww.indiandentalacademy.com
  • 70. Learning Points 10.2000Learning Points 10.2000  Exam findings are consistent with encephalitisExam findings are consistent with encephalitis  CSF findings are consistent with a viral etiologyCSF findings are consistent with a viral etiology  Most likely cause is a sporadic case of herpesMost likely cause is a sporadic case of herpes simplex virussimplex virus  Viral cultures of CSF for HSV are RARELYViral cultures of CSF for HSV are RARELY positive beyond neonatal periodpositive beyond neonatal period  HSV is not found in cultures of sites outside theHSV is not found in cultures of sites outside the CNSCNS www.indiandentalacademy.comwww.indiandentalacademy.com
  • 71. Learning Points 10.2000Learning Points 10.2000  What is the treatment of choice for herpesWhat is the treatment of choice for herpes encephalitis in a child?encephalitis in a child? – IV Acyclovir 10 mg/kg/dose q8 for 14-21 daysIV Acyclovir 10 mg/kg/dose q8 for 14-21 days  What is the treatment of choice forWhat is the treatment of choice for neonatal herpes encephalitis?neonatal herpes encephalitis? – IV Acyclovir 20 mg/kg/dose q8 for 14-21 daysIV Acyclovir 20 mg/kg/dose q8 for 14-21 days www.indiandentalacademy.comwww.indiandentalacademy.com
  • 72. Question 11.2000Question 11.2000  An 18 year old girl presents for a RV. ExamAn 18 year old girl presents for a RV. Exam reveals tanner stage 5 breasts with 2x2 cmreveals tanner stage 5 breasts with 2x2 cm nontender, smooth, mobile mass in left breast. Itnontender, smooth, mobile mass in left breast. It is located at the upper outer quadrant. Theis located at the upper outer quadrant. The overlying skin is normal and there is no historyoverlying skin is normal and there is no history of nipple discharge. Family history is negative forof nipple discharge. Family history is negative for breast cancer. What is the most appropriate nextbreast cancer. What is the most appropriate next step in evaluation of this breast mass?step in evaluation of this breast mass? – A. Excisional biopsyA. Excisional biopsy – B. Fine-needle biopsyB. Fine-needle biopsy – C. MammographyC. Mammography – D. Reassurance and reevaluation in 4-6 monthsD. Reassurance and reevaluation in 4-6 months – E. Referral for surgical consultationE. Referral for surgical consultation  ANSWER DANSWER D www.indiandentalacademy.comwww.indiandentalacademy.com
  • 73. Teaching Points 11.2000Teaching Points 11.2000  What is the diagnosis?What is the diagnosis? – FibroadenomaFibroadenoma  Fibroadenomas are most common breastFibroadenomas are most common breast lesion in an adolescent femalelesion in an adolescent female  Differential diagnosis breast masses:Differential diagnosis breast masses: – Fibrocystic changesFibrocystic changes – CystsCysts – AbscessesAbscesses – Rarely, malignancyRarely, malignancy www.indiandentalacademy.comwww.indiandentalacademy.com
  • 74. Teaching Points 11.2000Teaching Points 11.2000  Without family history of breast cancer in a 1Without family history of breast cancer in a 1stst or 2or 2ndnd degreedegree relative, primary breast cancer is extremely rare inrelative, primary breast cancer is extremely rare in adolescentsadolescents  Fewer than 1/1,000 of adolescent breast masses areFewer than 1/1,000 of adolescent breast masses are malignantmalignant  Metastatic lesions from…Metastatic lesions from… – Rhabdomyosarcoma, neuroblastoma, and lymphoma can occur, butRhabdomyosarcoma, neuroblastoma, and lymphoma can occur, but rare.rare.  If anxiety high can begin with what test…If anxiety high can begin with what test… – Ultrasound to differentiate between cystic and solid massesUltrasound to differentiate between cystic and solid masses  FNA or biopsy can be done in adolescent with unusually largeFNA or biopsy can be done in adolescent with unusually large or atypical mass or who is highly anxious about malignantor atypical mass or who is highly anxious about malignant potentialpotential  Mammography never is indicated fir evaluation of a breastMammography never is indicated fir evaluation of a breast mass in teenagers because the dense breast tissue makesmass in teenagers because the dense breast tissue makes study difficult to interpretstudy difficult to interpret www.indiandentalacademy.comwww.indiandentalacademy.com
  • 75. Question 12.2000Question 12.2000  A 15 year old girl complains of dysuria andA 15 year old girl complains of dysuria and abdominal pain for 2 days. She denies nausea,abdominal pain for 2 days. She denies nausea, vomiting, flank pain and vaginal discharge.vomiting, flank pain and vaginal discharge. Menarche occurred to years ago and mensesMenarche occurred to years ago and menses have been irregular. PE reveals SMR stage 4have been irregular. PE reveals SMR stage 4 genitalia, mild suprapubic tenderness, andgenitalia, mild suprapubic tenderness, and otherwise normal findings. What is the mostotherwise normal findings. What is the most likely diagnosis?likely diagnosis? – A. Bacterial vaginosisA. Bacterial vaginosis – B. Candidal vulvovaginitisB. Candidal vulvovaginitis – C.C. ChlamydiaChlamydia urethritisurethritis – D. Pelvic inflammatory diseaseD. Pelvic inflammatory disease – E. Urinary tract infectionE. Urinary tract infection  ANSWER E.ANSWER E. www.indiandentalacademy.comwww.indiandentalacademy.com
  • 76. Teaching Points 12.2000Teaching Points 12.2000  Bacterial vaginosisBacterial vaginosis – Grayish discharge with pH >4.5Grayish discharge with pH >4.5 – Saline wet mount reveals >20% clue cells and an absenceSaline wet mount reveals >20% clue cells and an absence of lactobacilliof lactobacilli – An isolatedAn isolated Gardenerella vaginalisGardenerella vaginalis infection does notinfection does not cause dysuria and lower abdominal paincause dysuria and lower abdominal pain  Candidal vaginosisCandidal vaginosis – Whitish dischargeWhitish discharge  Chlamydia trachomatisChlamydia trachomatis – Causes dysuria if urethra involvedCauses dysuria if urethra involved – No abdominal pain UNLESS PID presentNo abdominal pain UNLESS PID present – PID symptoms: lower abdominal pain, cervical motionPID symptoms: lower abdominal pain, cervical motion tenderness, adnexal tenderness; fever, vomiting, diarrhea,tenderness, adnexal tenderness; fever, vomiting, diarrhea, irregular vaginal bleeding, increased vaginal dischargeirregular vaginal bleeding, increased vaginal discharge www.indiandentalacademy.comwww.indiandentalacademy.com
  • 77. Clue CellsClue Cells  Vaginal epithelial cells that appear fuzzy without distinct edges under a microscope due to being coated with G. vaginalis  Clue cells flake off of the walls of the vagina and are found in vaginal smear.  Chemicals released by the bacteria that cause bacterial vaginosis (BV) may damage vaginal wall cells, causing them to flake off in greater numbers than usual.  When bacterial vaginosis is present, more than 20% of the sample vaginal epithelial cells are clue cells. www.indiandentalacademy.comwww.indiandentalacademy.com
  • 79. Question 13.2000Question 13.2000  A 3 year old girl comes to the ER withA 3 year old girl comes to the ER with temperature of 103.1temperature of 103.1oo F and acute onsetF and acute onset diarrhea. Stool is guaiac positive withdiarrhea. Stool is guaiac positive with leukocytes. There is no history of foreign travelleukocytes. There is no history of foreign travel and the child has not received antibioticsand the child has not received antibiotics recently. What is the most likely organism?recently. What is the most likely organism? – A.A. Clostridium difficileClostridium difficile – B.B. Giardia lambliaGiardia lamblia – C. RotavirusC. Rotavirus – D.D. Salmonella enteritidisSalmonella enteritidis – E.E. Vibrio choleraeVibrio cholerae  ANSWER D.ANSWER D. www.indiandentalacademy.comwww.indiandentalacademy.com
  • 80. Teaching Points 13.2000Teaching Points 13.2000  Viral diarrheaViral diarrhea – Low-grade fever, vomiting, large, loose wateryLow-grade fever, vomiting, large, loose watery stoolsstools – Most common cause…Most common cause…  RotavirusRotavirus – Season predominance…Season predominance…  Winter in United StatesWinter in United States www.indiandentalacademy.comwww.indiandentalacademy.com
  • 81. Teaching Points 13.2000Teaching Points 13.2000  Bacterial DiarrheaBacterial Diarrhea – High fevers, small frequent stools with mucous orHigh fevers, small frequent stools with mucous or bloodblood – What can happen if you treatWhat can happen if you treat SalmonellaSalmonella infectioninfection with antibiotics?with antibiotics?  Prolong the carrier stateProlong the carrier state – Which organism is seen after antibiotics?Which organism is seen after antibiotics?  Clostridium difficileClostridium difficile – What organism is seen after ingestion of seafood orWhat organism is seen after ingestion of seafood or water?water?  VibrioVibrio cholerae – Giardia lambliaGiardia lamblia results in chronic diarrhea withresults in chronic diarrhea with malabsorptionmalabsorption www.indiandentalacademy.comwww.indiandentalacademy.com
  • 82. Question 14.1000Question 14.1000  You are evaluating a 4 week old boy for tearing ofYou are evaluating a 4 week old boy for tearing of the right eye that has worsened over the past week.the right eye that has worsened over the past week. Physical exam reveals slight tearing but noPhysical exam reveals slight tearing but no evidence of purulent exudate or conjunctivalevidence of purulent exudate or conjunctival erythema. All other findings are normal. The MOSTerythema. All other findings are normal. The MOST appropriate initial management is…appropriate initial management is… – A. Administration of amoxicillinA. Administration of amoxicillin – B. Endoscopic dacrocystorhinostomyB. Endoscopic dacrocystorhinostomy – C. Instillation of silver nitrate in the eyesC. Instillation of silver nitrate in the eyes – D. Observation with intermittent massage of the ductD. Observation with intermittent massage of the duct – E. Surgical dilation of the nasolacrimal ductE. Surgical dilation of the nasolacrimal duct  ANSWER DANSWER D www.indiandentalacademy.comwww.indiandentalacademy.com
  • 83. Teaching Points 14.2000Teaching Points 14.2000  Congenital nasolacrimal duct obstruction is mostCongenital nasolacrimal duct obstruction is most common abnormality of infant lacrimal system, 5%common abnormality of infant lacrimal system, 5% affectedaffected  What percent of those affected have bilateralWhat percent of those affected have bilateral obstruction?obstruction? – 30%30%  Obstruction is usually found where?Obstruction is usually found where? – Distal endDistal end  Pertinent negatives on examPertinent negatives on exam – Conjunctival inflammation, photophobia, blepharospasm,Conjunctival inflammation, photophobia, blepharospasm, corneal cloudingcorneal clouding www.indiandentalacademy.comwww.indiandentalacademy.com
  • 84. Question 68.1999Question 68.1999 A previously health 1-year-old infant who weighs 10 kg presentsA previously health 1-year-old infant who weighs 10 kg presents to your office with a fever of 39° C (102.2° F). Her mother isto your office with a fever of 39° C (102.2° F). Her mother is very concerned about the child’s intake and asks for guidancevery concerned about the child’s intake and asks for guidance regarding caloric requirements during this illness.regarding caloric requirements during this illness. Of the following, the best estimate of the child’s caloricOf the following, the best estimate of the child’s caloric requirements at this time isrequirements at this time is A.A.500 kcal/d plus 500 kcal due to the fever500 kcal/d plus 500 kcal due to the fever B.B.1,000 kcal/d plus 250 kcal due to the fever1,000 kcal/d plus 250 kcal due to the fever C.C.1,500 kcal/d1,500 kcal/d D.D.1,500 kcal/d plus 250 kcal due to the fever1,500 kcal/d plus 250 kcal due to the fever E.E.2,000 kcal/d2,000 kcal/d Answer: BAnswer: Bwww.indiandentalacademy.comwww.indiandentalacademy.com
  • 85. Question (68.1999)Question (68.1999)  What is the most effective indicator of whether aWhat is the most effective indicator of whether a child is getting enough calories?child is getting enough calories? – Growth ChartGrowth Chart  Name 5 factors that affect a child’s energy (calorie)Name 5 factors that affect a child’s energy (calorie) requirements.requirements. – Basal metabolism calories (Maintenance at rest andBasal metabolism calories (Maintenance at rest and fasting)fasting) – Growing caloriesGrowing calories – Exercise caloriesExercise calories – Eating caloriesEating calories – Hypermetabolic states et al (See Table)Hypermetabolic states et al (See Table) www.indiandentalacademy.comwww.indiandentalacademy.com
  • 86. Calculation of CaloricCalculation of Caloric Requirements Based on BodyRequirements Based on Body Weight (68.1999)Weight (68.1999) Body WeightBody Weight Caloric RequirementCaloric Requirement ≤≤ 10 kg10 kg 100 kcal/kg100 kcal/kg 11 to 20 kg11 to 20 kg 1,000 kcal + 501,000 kcal + 50 kcal/kg for each kgkcal/kg for each kg above 10 kgabove 10 kg ≥≥ 20 kg20 kg 1,500 kcal + 201,500 kcal + 20 kcal/kg for each kgkcal/kg for each kg above 20 kgabove 20 kgwww.indiandentalacademy.comwww.indiandentalacademy.com
  • 87. Calculating Energy Lost in FeverCalculating Energy Lost in Fever (68.1999)(68.1999)  FormulaFormula – For each degree above 37For each degree above 37oo C, multiply by 12%C, multiply by 12% of maintenance requirements per degree. Thenof maintenance requirements per degree. Then add to original maintenance requirements foradd to original maintenance requirements for total energy needs.total energy needs. – Example: 10 kg infant with 39Example: 10 kg infant with 39oo C temperature.C temperature. Calculate total caloric requirements.Calculate total caloric requirements.  10 kg X 100kcal/kg= 1,000 kcal maintenance needs10 kg X 100kcal/kg= 1,000 kcal maintenance needs  22oo X 0.12(1,000kcal) = 240 kcalX 0.12(1,000kcal) = 240 kcal  Total Needs = 1,240 kcalTotal Needs = 1,240 kcal www.indiandentalacademy.comwww.indiandentalacademy.com
  • 88. Question 69Question 69 A 5-year-old boy is brought to the emergency department afterA 5-year-old boy is brought to the emergency department after having been struck by an automobile. Physical examination revealshaving been struck by an automobile. Physical examination reveals facial abrasions, abdominal tenderness, and gross blood at thefacial abrasions, abdominal tenderness, and gross blood at the urethral meatus. Pelvic radiography reveals a left-sided fracture ofurethral meatus. Pelvic radiography reveals a left-sided fracture of the superior pubic rami.the superior pubic rami. Of the following, the best procedure for INITIAL evaluation of theOf the following, the best procedure for INITIAL evaluation of the urinary tract in this patient isurinary tract in this patient is A.A.Bladder catheterization via the urethraBladder catheterization via the urethra B.B.Computed tomography of the abdomenComputed tomography of the abdomen C.C.Intravenous pyelographyIntravenous pyelography D.D.Renal untrasonographyRenal untrasonography E.E.Retrograde urethrographyRetrograde urethrography Answer: EAnswer: Ewww.indiandentalacademy.comwww.indiandentalacademy.com
  • 89. Additional Caloric Requirements in SelectedAdditional Caloric Requirements in Selected Medical and Surgical Conditions (68.1999)Medical and Surgical Conditions (68.1999) ConditionCondition Maximum Amount ofMaximum Amount of Additional CaloriesAdditional Calories RequiredRequired Hypermetabolic stateHypermetabolic state due to recent surgerydue to recent surgery 20%20% Multiple traumaMultiple trauma 25%25% Severe infectionSevere infection 50%50% Third-degree burnsThird-degree burns (<20% of body)(<20% of body) 100%100% UncomplicatedUncomplicated starvationstarvation 50%50% www.indiandentalacademy.comwww.indiandentalacademy.com
  • 90. Question (69.1999)Question (69.1999)  In children who sustain multiple injuries in aIn children who sustain multiple injuries in a vehicular crash what are the top two “systems”vehicular crash what are the top two “systems” that are involved?that are involved? – Central Nervous System #1Central Nervous System #1 – Genitourinary System #2Genitourinary System #2  Blood at tip of penis suggests urethral injury.Blood at tip of penis suggests urethral injury.  Injury to the prostatomembranous portion ofInjury to the prostatomembranous portion of urethra associated with pelvic fracture is mosturethra associated with pelvic fracture is most common.common.  Isolated urethral injury in female isIsolated urethral injury in female is UNCOMMON.UNCOMMON. www.indiandentalacademy.comwww.indiandentalacademy.com
  • 91. Answer Explanations (69.1999)Answer Explanations (69.1999)  Retrograde urethrography (E.) -CatheterRetrograde urethrography (E.) -Catheter placed just inside urethral meatus and dyeplaced just inside urethral meatus and dye insertedinserted  Routine catheterization (A.) isRoutine catheterization (A.) is contraindicated with blood at urethralcontraindicated with blood at urethral meatus because…meatus because… – Procedure might convert a PARTIAL TEAR ofProcedure might convert a PARTIAL TEAR of meatus into a COMPLETE TRANSECTIONmeatus into a COMPLETE TRANSECTION www.indiandentalacademy.comwww.indiandentalacademy.com
  • 92. Answer Explanations (69.1999)Answer Explanations (69.1999)  Abdominal CT would be helpful for…Abdominal CT would be helpful for… – Evaluating intrabdominal injuriesEvaluating intrabdominal injuries  Splenic injurySplenic injury  Liver injuryLiver injury  Renal injuryRenal injury  Intravenous pyelography and Renal USIntravenous pyelography and Renal US will evaluate renal pathology but notwill evaluate renal pathology but not evaluate suspected urethral injuriesevaluate suspected urethral injuries www.indiandentalacademy.comwww.indiandentalacademy.com
  • 93. Apgar ScoreApgar Score (Virginia Apgar 1953)(Virginia Apgar 1953) Component ofComponent of Apgar ScoreApgar Score Score (Check at 1 and 5 minutes)Score (Check at 1 and 5 minutes) 00 11 22 Heart RateHeart Rate NoneNone <100<100 beats/minbeats/min >100>100 beats/minbeats/min RespiratoryRespiratory EffortEffort NoneNone Weak cryWeak cry Lusty cryLusty cry Muscle ToneMuscle Tone LimpLimp Some flexionSome flexion Well flexedWell flexed ReflexReflex IrratibilityIrratibility No responseNo response Some motionSome motion VigorousVigorous responseresponse ColorColor Uniformly BlueUniformly Blue AcrocyanosisAcrocyanosis CompletelyCompletely pinkpink www.indiandentalacademy.comwww.indiandentalacademy.com
  • 94. Question 70Question 70 You are attending the emergency delivery by cesarean section of aYou are attending the emergency delivery by cesarean section of a primiparous woman. The gestation was complicated by pregnancy-primiparous woman. The gestation was complicated by pregnancy- induced hypertension. Deep variable fetal heart rate decelerationsinduced hypertension. Deep variable fetal heart rate decelerations were noted during labor. At delivery, the infant is acrocyanotic withwere noted during labor. At delivery, the infant is acrocyanotic with poor tone; spontaneous movement and minimal respiratory effort arepoor tone; spontaneous movement and minimal respiratory effort are present.present. Of the following, your INITIAL management is toOf the following, your INITIAL management is to A.A.Ascertain the heart rate and assign a 1-minute Apgar scoreAscertain the heart rate and assign a 1-minute Apgar score B.B.Begin tactile stimulation and provide blow-blow oxygenBegin tactile stimulation and provide blow-blow oxygen supplementationsupplementation C.C.Dry all skin surfaces and clear the oropharynxDry all skin surfaces and clear the oropharynx D.D.Initiate bag-mask ventilationInitiate bag-mask ventilation E.E.Insert an umbilical catheter and administer naloxoneInsert an umbilical catheter and administer naloxone Answer: CAnswer: Cwww.indiandentalacademy.comwww.indiandentalacademy.com
  • 95. Learning Points (70.1999)Learning Points (70.1999)  The body and head of an infant areThe body and head of an infant are immediately dried with a prewarmed towelimmediately dried with a prewarmed towel to remove…to remove… – Amniotic fluidAmniotic fluid  And to prevent…And to prevent… – Evaporative heat loss which could lead toEvaporative heat loss which could lead to hypothermiahypothermia  Also provides gentle stimulation to infantAlso provides gentle stimulation to infant www.indiandentalacademy.comwww.indiandentalacademy.com
  • 96. Learning Points (70.1999)Learning Points (70.1999)  When suctioning, mouth or nose first and why?When suctioning, mouth or nose first and why? – Mouth to ensure nothing in oropharynx that could beMouth to ensure nothing in oropharynx that could be aspirated as we are dealing with obligate noseaspirated as we are dealing with obligate nose breathersbreathers  Suctioning also provides tactile stimulationSuctioning also provides tactile stimulation  Score of 7 or more indicates…Score of 7 or more indicates… – WELL NEWBORNWELL NEWBORN  ROUTINE CARE ANDROUTINE CARE AND OBSERVATIONOBSERVATION  Score of 4-6 indicates…Score of 4-6 indicates… – MILD TO MODERATE DEPRESSION, NEED MOREMILD TO MODERATE DEPRESSION, NEED MORE INTERVENTION (Blowby, stimulation etc.)INTERVENTION (Blowby, stimulation etc.)  Score 3 or less…Score 3 or less… – SEVERE DEPRESSION, NEED TO INTUBATE, CVSEVERE DEPRESSION, NEED TO INTUBATE, CV SUPPORT, BAG-MASK VENTILATIONSUPPORT, BAG-MASK VENTILATIONwww.indiandentalacademy.comwww.indiandentalacademy.com
  • 97. Learning Points (70.1999)Learning Points (70.1999)  When do you keep on taking ApgarWhen do you keep on taking Apgar scores?scores? – When 5 minute is 6 or lessWhen 5 minute is 6 or less  Additional scores should be assignedAdditional scores should be assigned every 5 minutes for up to 20 minutes orevery 5 minutes for up to 20 minutes or until two consecutive scores of 7+ areuntil two consecutive scores of 7+ are obtainedobtained www.indiandentalacademy.comwww.indiandentalacademy.com
  • 98. Question 71Question 71 A 1-year-old boy has been treated with a low-phenylalanine diet forA 1-year-old boy has been treated with a low-phenylalanine diet for the past year after having been identified in infancy as havingthe past year after having been identified in infancy as having phenylketonuria. Despite appropriate dietary restriction ofphenylketonuria. Despite appropriate dietary restriction of phenylalanine, he has developed neurologic symptoms.phenylalanine, he has developed neurologic symptoms. Of the following, this child is MOST likely to be deficient inOf the following, this child is MOST likely to be deficient in A.A.BiotinBiotin B.B.CobalaminCobalamin C.C.CarnitineCarnitine D.D.TegrahydrobiopterinTegrahydrobiopterin E.E.ThiamineThiamine Answer: DAnswer: D www.indiandentalacademy.comwww.indiandentalacademy.com
  • 99. Learning Points (71.1999)Learning Points (71.1999)  PKU diagnosed by hyperphenylalaninemia inPKU diagnosed by hyperphenylalaninemia in blood sample taken at 48 hours of ageblood sample taken at 48 hours of age  Two types:Two types: – Classical PKU has deficiency inClassical PKU has deficiency in phenylalaninephenylalanine hydroxylasehydroxylase – Deficiency of enzyme cofactor tetrahydrobiopterinDeficiency of enzyme cofactor tetrahydrobiopterin (1-3% patients with hyperphenylalaninemia)(1-3% patients with hyperphenylalaninemia)  Involved in hydroxylation reactions for tryptophan andInvolved in hydroxylation reactions for tryptophan and tyrosinetyrosine  OH-tryptophan and L-DOPAOH-tryptophan and L-DOPA (Neurotransmitters)(Neurotransmitters)  Thus, phenylalanine restriction by itself in presence ofThus, phenylalanine restriction by itself in presence of cofactor deficiency WILL NOT prevent neurologicalcofactor deficiency WILL NOT prevent neurological damagedamage www.indiandentalacademy.comwww.indiandentalacademy.com
  • 100. Learning Points (71.1999)Learning Points (71.1999)  How do you diagnose tetrahydrobiopterinHow do you diagnose tetrahydrobiopterin deficiency?deficiency? – Caused by recycling or synthesis defectsCaused by recycling or synthesis defects – Measure pterin metabolites found in urineMeasure pterin metabolites found in urine – Can also diagnose by a reduction inCan also diagnose by a reduction in phenylalanine after IV or oral load ofphenylalanine after IV or oral load of tetrahydrobiopterintetrahydrobiopterin www.indiandentalacademy.comwww.indiandentalacademy.com
  • 101. Learning Points (71.1999)Learning Points (71.1999)  How do you treat tetrahydrobiopterinHow do you treat tetrahydrobiopterin deficiency?deficiency? – Administer tetrahydrobiopterinAdminister tetrahydrobiopterin – Replace dopamine and serotonin asReplace dopamine and serotonin as tetrahydrobiopterin poorly penetrates braintetrahydrobiopterin poorly penetrates brain where neurotransmitters are formedwhere neurotransmitters are formed  Also remember there are a smallAlso remember there are a small percentage of infants born with transientpercentage of infants born with transient hyperphenylalaninemia which has nohyperphenylalaninemia which has no clinical consequenceclinical consequence www.indiandentalacademy.comwww.indiandentalacademy.com
  • 102. Learning Points (71.1999)Learning Points (71.1999)  Cobalamin is coenzyme foir methmalonyl CoACobalamin is coenzyme foir methmalonyl CoA mutase whose deficiency results in…mutase whose deficiency results in… – Methylmalonic aciduriaMethylmalonic aciduria  Carnitine used for transport of medium and longCarnitine used for transport of medium and long chain fatty acids across mitochondriachain fatty acids across mitochondria – Carnitine deficient patients have muscle weaknessCarnitine deficient patients have muscle weakness  Thiamine deficiency results in…Thiamine deficiency results in… – Beri beriBeri beri  Biotin is cofactor for carboxylases thatBiotin is cofactor for carboxylases that catabolize branched chain amino acids involvedcatabolize branched chain amino acids involved in fatty acid synthesisin fatty acid synthesis www.indiandentalacademy.comwww.indiandentalacademy.com
  • 103. Question 72Question 72 A 22-month-old girls is nonverbal. She sat alone at 7 months andA 22-month-old girls is nonverbal. She sat alone at 7 months and walked by 13 months, but now exhibits a wide-based stance, nowalked by 13 months, but now exhibits a wide-based stance, no longer ambulates, and will not pick up or manipulate toys. Findingslonger ambulates, and will not pick up or manipulate toys. Findings include: height and weight at the 50include: height and weight at the 50thth percentile; headpercentile; head circumference below the 5circumference below the 5thth percentile, with no increase over thepercentile, with no increase over the past 8 months; normal fundi, and no organomegaly.past 8 months; normal fundi, and no organomegaly. Of the following, the MOST likely diagnosis isOf the following, the MOST likely diagnosis is A.A.AdrenoleukodystrophyAdrenoleukodystrophy B.B.Cerebral palsyCerebral palsy C.C.GMGM22 gangliosidosis (Tay-Sachs disease)gangliosidosis (Tay-Sachs disease) D.D.HypothroidismHypothroidism E.E.Rett syndromeRett syndrome Answer: EAnswer: E www.indiandentalacademy.comwww.indiandentalacademy.com
  • 104. Learning Points (72.1999)Learning Points (72.1999)  Developmental DelayDevelopmental Delay – RegressionRegression – PlateauPlateau – ProgressionProgression  Good somatic growth but no head growthGood somatic growth but no head growth in 8 months known as…in 8 months known as… – Acquired microcephalyAcquired microcephaly  Lost purposeful hand use when picking upLost purposeful hand use when picking up objects known as…objects known as… – DyspraxiaDyspraxia www.indiandentalacademy.comwww.indiandentalacademy.com
  • 105. Learning Points (72.1999)Learning Points (72.1999)  Rett syndrome (SSSH I don’t have Rett syndrome)Rett syndrome (SSSH I don’t have Rett syndrome) – Neurodegenerative disorderNeurodegenerative disorder – Females only (male fetuses die in utero)Females only (male fetuses die in utero) – 1:10,0001:10,000 – X-linked dominantX-linked dominant – SeizuresSeizures – SpasticitySpasticity – ScoliosisScoliosis – Hand-wringingHand-wringing – Become severe spastic quadriplegicsBecome severe spastic quadriplegics www.indiandentalacademy.comwww.indiandentalacademy.com
  • 106. Learning Points (72.1999)Learning Points (72.1999)  AdrenoleukodystrophyAdrenoleukodystrophy – Proxisomal disorder only found in malesProxisomal disorder only found in males – Defect in acyl CoA synthetase in peroxisomesDefect in acyl CoA synthetase in peroxisomes – Cerebral form (50%),Cerebral form (50%), adrenomyeloneuropathy (25%), isolatedadrenomyeloneuropathy (25%), isolated Addison disease or symptomatic (25%)Addison disease or symptomatic (25%) – Progressive demyelination of brainProgressive demyelination of brain   Dementia, Blindness, Deafness, Decorticate stateDementia, Blindness, Deafness, Decorticate state – Adrenal insufficiencyAdrenal insufficiency www.indiandentalacademy.comwww.indiandentalacademy.com
  • 107. Learning Points (72.1999)Learning Points (72.1999)  Tay-SachsTay-Sachs – Rapidly progressive neurodegenerative disorder due toRapidly progressive neurodegenerative disorder due to lysosomal storage disorder (GM2, gangliosidosis)lysosomal storage disorder (GM2, gangliosidosis) caused by deficiency in …caused by deficiency in …  Beta-hexosaminidase A enzymeBeta-hexosaminidase A enzyme – Onset before 9 monthsOnset before 9 months – Hypotonic, never walk, increased startle responseHypotonic, never walk, increased startle response – Fundoscopic exam reveals…Fundoscopic exam reveals…  Cherry red spotCherry red spot – What populations has this been found more frequentlyWhat populations has this been found more frequently in?in?  Ashkenazi JewsAshkenazi Jews  French CanadiansFrench Canadians www.indiandentalacademy.comwww.indiandentalacademy.com
  • 108. Learning Points (72.1999)Learning Points (72.1999)  Congenital hypothyroidismCongenital hypothyroidism – High likelihood for normal development ifHigh likelihood for normal development if treatment started before 3 months of agetreatment started before 3 months of age – Can make up some developmental milestonesCan make up some developmental milestones if treatment begun before 6 months of ageif treatment begun before 6 months of age – Can see polyneurpathies and muscleCan see polyneurpathies and muscle weaknessweakness www.indiandentalacademy.comwww.indiandentalacademy.com
  • 109. Learning Points (72.1999)Learning Points (72.1999)  Cerebral Palsy: What are 4 types?Cerebral Palsy: What are 4 types? – SpasticSpastic – DyskineticDyskinetic – AtaxicAtaxic – HypotonicHypotonic  Must have impaired motor control to makeMust have impaired motor control to make this diagnosisthis diagnosis www.indiandentalacademy.comwww.indiandentalacademy.com
  • 110. Question 73Question 73 A 10-year-old boy who has hereditary spherocytosis has developedA 10-year-old boy who has hereditary spherocytosis has developed increasing fatigue and pallor over the past 5 days, and hisincreasing fatigue and pallor over the past 5 days, and his hemoglobin concentration, which usually ranges between 10 andhemoglobin concentration, which usually ranges between 10 and 11g/dL, now is 5.9g/dL.11g/dL, now is 5.9g/dL. Of the following, the MOST likely etiology of this decrease inOf the following, the MOST likely etiology of this decrease in hemoglobin ishemoglobin is A.A.Aplastic crisisAplastic crisis B.B.Folic acid deficiencyFolic acid deficiency C.C.Hyperhemolytic crisisHyperhemolytic crisis D.D.Iron-deficiency anemiaIron-deficiency anemia E.E.Splenic sequestrationSplenic sequestration Answer: AAnswer: A www.indiandentalacademy.comwww.indiandentalacademy.com
  • 111. Learning Points (73.1999)Learning Points (73.1999)  Aplastic crisis is the most common cause of severeAplastic crisis is the most common cause of severe anemia in children with hereditary spherocytosisanemia in children with hereditary spherocytosis (HS)(HS)  What is the most common cause of such a crisis?What is the most common cause of such a crisis? – Parvovirus B19Parvovirus B19 erythema infectiosum (fever, chills,erythema infectiosum (fever, chills, lethargy, nausea, vomiting, abdominal pain, respiratorylethargy, nausea, vomiting, abdominal pain, respiratory symptoms, myalgias, arthralgias, “slapped cheek” rash)symptoms, myalgias, arthralgias, “slapped cheek” rash) – Parvovirus selectively attacks erythroid precursor cellsParvovirus selectively attacks erythroid precursor cells inhibiting their growthinhibiting their growth  decreased reticulocytes anddecreased reticulocytes and bilirubin as number of RBCs destroyed decreases dailybilirubin as number of RBCs destroyed decreases daily www.indiandentalacademy.comwww.indiandentalacademy.com
  • 112. Learning Points (73.1999)Learning Points (73.1999)  How long does the crisis typically last?How long does the crisis typically last? – 10-14 days10-14 days  Are WBCs and platelets affected as well?Are WBCs and platelets affected as well? – Yes they can but it is usually much milderYes they can but it is usually much milder  When do you know someone is recovering fromWhen do you know someone is recovering from a lab standpoint?a lab standpoint? – Presence of a reticulocytosisPresence of a reticulocytosis  * Parvovirus infection in first half of pregnancy in* Parvovirus infection in first half of pregnancy in patient with HS may cause fetal loss due topatient with HS may cause fetal loss due to severe anemiasevere anemia www.indiandentalacademy.comwww.indiandentalacademy.com
  • 113. Learning Points (73.1999)Learning Points (73.1999)  What percentage of kids with HS haveWhat percentage of kids with HS have mild or no anemia?mild or no anemia? – 90%90%  How long is the life span of a RBC in aHow long is the life span of a RBC in a patient with HS?patient with HS? – 30 days30 days  With this short of a lifespan hw does theWith this short of a lifespan hw does the body keep up?body keep up? – Enhanced erythropoiesisEnhanced erythropoiesis www.indiandentalacademy.comwww.indiandentalacademy.com
  • 114. Learning Points (73.1999)Learning Points (73.1999)  Folic acid and iron deficiency may exacerbateFolic acid and iron deficiency may exacerbate the anemia but this is a gradual decline inthe anemia but this is a gradual decline in hemoglobinhemoglobin  Hyperhemolytic crisis without aplasia occurs inHyperhemolytic crisis without aplasia occurs in HS in children less than 6 years who have viralHS in children less than 6 years who have viral syndromessyndromes – Mild and transient increases in bilirubin,Mild and transient increases in bilirubin, splenomegaly, anemia, reticulocytosissplenomegaly, anemia, reticulocytosis  Splenic sequestration is typical of sickle cellSplenic sequestration is typical of sickle cell disease and is a minor component of hemolyticdisease and is a minor component of hemolytic crisis associated with spherocytosiscrisis associated with spherocytosis www.indiandentalacademy.comwww.indiandentalacademy.com
  • 115. Question 74Question 74 An 18-day-old term infant develops fever, lethargy, and focalAn 18-day-old term infant develops fever, lethargy, and focal seizures. Findings include: an ill-appearing infant withoutseizures. Findings include: an ill-appearing infant without exanthem, hepatomegaly, or jaundice. Analysis of cerebrospinalexanthem, hepatomegaly, or jaundice. Analysis of cerebrospinal fluid reveals white blood cells, 115/mmfluid reveals white blood cells, 115/mm33 ; 45% neutrophils; 55%; 45% neutrophils; 55% lymphocytes; red blood cells, 40/mmlymphocytes; red blood cells, 40/mm33 ; glucose, 45 mg/dL; protien; glucose, 45 mg/dL; protien 200 mg/dL; and negative Gram stain.200 mg/dL; and negative Gram stain. In addition to ampicillin and cefotaxime, the MOST appropriateIn addition to ampicillin and cefotaxime, the MOST appropriate treatment to begin at this time istreatment to begin at this time is A.A.AcyclovirAcyclovir B.B.Amphotericin BAmphotericin B C.C.DexamethasoneDexamethasone D.D.MetronidazoleMetronidazole E.E.vancomycinvancomycin Answer: AAnswer: Awww.indiandentalacademy.comwww.indiandentalacademy.com
  • 116. Learning Points (74.1999)Learning Points (74.1999)  HSV is transmitted during delivery via contact withHSV is transmitted during delivery via contact with maternal secretionsmaternal secretions  Only 5% of adults in US have a history of genital herpesOnly 5% of adults in US have a history of genital herpes  20-25% are actively infected with HSV-220-25% are actively infected with HSV-2  What is the risk of an HSV infection in an infant bornWhat is the risk of an HSV infection in an infant born vaginally to a mother who has a 1vaginally to a mother who has a 1stst or primary genitalor primary genital infection?infection? – 33-50%33-50%  What is the risk in a mom with recurrent HSV?What is the risk in a mom with recurrent HSV? – 3-5%3-5%  MOST HSV-INFECTED INFANTS ARE BORN TOMOST HSV-INFECTED INFANTS ARE BORN TO WOMEN WITH NO HISTORY OF GENITAL HERPESWOMEN WITH NO HISTORY OF GENITAL HERPES AND NO SIGNS OF INFECTION DURINGAND NO SIGNS OF INFECTION DURING PREGNANCY OR AT DELIVERYPREGNANCY OR AT DELIVERY www.indiandentalacademy.comwww.indiandentalacademy.com
  • 117. Learning Points (74.1999)Learning Points (74.1999)  HSV in newbornHSV in newborn – Generalized systemic infection involving liverGeneralized systemic infection involving liver and CNS in 25% casesand CNS in 25% cases – Localized CNS disease 35%Localized CNS disease 35% – Localized to skin, eyes, mouth 40%Localized to skin, eyes, mouth 40% – UP TO 33% OF INFANTS WITHUP TO 33% OF INFANTS WITH DISSEMINATED OR LOCALIZED CNSDISSEMINATED OR LOCALIZED CNS DISEASE WILL HAVEDISEASE WILL HAVE NONO SKIN, EYES ORSKIN, EYES OR MOUTH INVOLVEMENTMOUTH INVOLVEMENT www.indiandentalacademy.comwww.indiandentalacademy.com
  • 118. Learning Points (74.1999)Learning Points (74.1999)  Disseminated HSV diseaseDisseminated HSV disease – 11stst week of lifeweek of life – Liver and adrenals primarily involvedLiver and adrenals primarily involved – Irritability, seizures, respiratory distress,Irritability, seizures, respiratory distress, jaundice, vesicular rash, shockjaundice, vesicular rash, shock – What is the mortality rate even with treatment?What is the mortality rate even with treatment?  50-60%50-60% – What are the two most common causes ofWhat are the two most common causes of death in disseminated HSV?death in disseminated HSV?  HSV pneumonitisHSV pneumonitis  Disseminated intravascular coagulopathyDisseminated intravascular coagulopathy www.indiandentalacademy.comwww.indiandentalacademy.com
  • 119. Question 75Question 75 A 10-year-old girl has complained of intermittent left lowerA 10-year-old girl has complained of intermittent left lower abdominal pain for 2 days. Previous evaluations, including aabdominal pain for 2 days. Previous evaluations, including a thorough physical examination, urinalysis, and complete bloodthorough physical examination, urinalysis, and complete blood count, have not revealed the cause. Tonight she is complaining ofcount, have not revealed the cause. Tonight she is complaining of pain in the lower left abdomen that radiates into her left leg. Therepain in the lower left abdomen that radiates into her left leg. There is no history of fever, vomiting, or diarrhea.is no history of fever, vomiting, or diarrhea. The MOST likely cause of this girl’s pain isThe MOST likely cause of this girl’s pain is A.A.AppendicitisAppendicitis B.B.IntussusceptionIntussusception C.C.Malrotation of the intestineMalrotation of the intestine D.D.NephrolithiasisNephrolithiasis E.E.Ovarian torsionOvarian torsion Answer: EAnswer: E www.indiandentalacademy.comwww.indiandentalacademy.com
  • 120. Teaching Points 75.1999Teaching Points 75.1999  Ovarian torsion pain: sharp lower abdominal pain radiatingOvarian torsion pain: sharp lower abdominal pain radiating to ipsilateral extremity, occasional vomitingto ipsilateral extremity, occasional vomiting  US can assess ovariesUS can assess ovaries  Surgical exploration to confirm diagnosis and preventSurgical exploration to confirm diagnosis and prevent ovarian necrosis (and R/O appendicitis if not already done)ovarian necrosis (and R/O appendicitis if not already done)  Left sided pain with appendicitis rare but seenLeft sided pain with appendicitis rare but seen  What is pain on right with palpation of the left called?What is pain on right with palpation of the left called? – Rovsing’s SignRovsing’s Sign  What is the obturator sign?What is the obturator sign? – Pain on internal rotation of the right hip, seen with pelvic appendixPain on internal rotation of the right hip, seen with pelvic appendix  What is the iliopsoas sign?What is the iliopsoas sign? – Pain on right hip extension, often seen with retrocecal appendixPain on right hip extension, often seen with retrocecal appendix www.indiandentalacademy.comwww.indiandentalacademy.com
  • 121. Teaching Points 75.1999Teaching Points 75.1999  Wrong age for intussusception (around 2 yearsWrong age for intussusception (around 2 years of age)of age)  Irritability, colicky abdominal pain, emesisIrritability, colicky abdominal pain, emesis  Impaired venous returnImpaired venous return  bowel edemabowel edema  ischemiaischemia  necrosisnecrosis  perforationperforation  Rectal bleeding seen in what percentage ofRectal bleeding seen in what percentage of kids?kids? – 80%; in “currant jelly” form (blood plus mucous)80%; in “currant jelly” form (blood plus mucous)  Most common location?Most common location? – IleocolicIleocolic  Tubular mass palpable in what percentage ofTubular mass palpable in what percentage of kids?kids? – 80%80% www.indiandentalacademy.comwww.indiandentalacademy.com
  • 122. Teaching Points 75.1999Teaching Points 75.1999  Lead point should be sought out in neonates and childrenLead point should be sought out in neonates and children older than 5. Name lead points.older than 5. Name lead points. – Meckel’s diverticulumMeckel’s diverticulum – Intestinal polypIntestinal polyp – LymphomaLymphoma – Foreign bodyForeign body  KUB: Paucity gas in RLQ or evidence of obstruction with airKUB: Paucity gas in RLQ or evidence of obstruction with air fluid levelsfluid levels  Gastrograffin enema shows coiled-spring appearance toGastrograffin enema shows coiled-spring appearance to bowel which is diagnostic, and may treat as well 75% casesbowel which is diagnostic, and may treat as well 75% cases  IVFs with NSIVFs with NS  Laporotomy with direct reduction if enema unsuccessful orLaporotomy with direct reduction if enema unsuccessful or peritoneal signs present and enema contraindicated due toperitoneal signs present and enema contraindicated due to concern of perforationconcern of perforation  Immediate recurrence rate 15%Immediate recurrence rate 15% www.indiandentalacademy.comwww.indiandentalacademy.com
  • 123. Question 76Question 76 A 13-year-old boy has a congested, itchy, and runny noseA 13-year-old boy has a congested, itchy, and runny nose accompanied by itchy eyes. These symptoms usually occur in theaccompanied by itchy eyes. These symptoms usually occur in the fall, are unresponsive to over-the-counter decongestants, and hisfall, are unresponsive to over-the-counter decongestants, and his father has the same problems. Physical examination reveals pale,father has the same problems. Physical examination reveals pale, boggy nasal turbinates; clear nasal discharge; and dark circlesboggy nasal turbinates; clear nasal discharge; and dark circles under his eyes.under his eyes. These findings are MOST consistent withThese findings are MOST consistent with A.A.Allergic rhinitisAllergic rhinitis B.B.Infectious rhinitisInfectious rhinitis C.C.Nonallergic rhinitis with eosinophiliaNonallergic rhinitis with eosinophilia D.D.Rhinitis medicamentosaRhinitis medicamentosa E.E.Vasomotor rhinitisVasomotor rhinitis Answer: AAnswer: A www.indiandentalacademy.comwww.indiandentalacademy.com
  • 124. Teaching Points 76.1999Teaching Points 76.1999  Perennial rhinitis (year round)Perennial rhinitis (year round) – Pets, dust mites, molds, cockroachesPets, dust mites, molds, cockroaches  Vasomotor rhinitisVasomotor rhinitis – Diagnosis of exclusion, chronic blockage orDiagnosis of exclusion, chronic blockage or hypersecretion but negative skin tests and normalhypersecretion but negative skin tests and normal paranasal sinus imagingparanasal sinus imaging  Infectious rhinitisInfectious rhinitis – Mucopurulent discharge, sneezing, limited pruritusMucopurulent discharge, sneezing, limited pruritus  Rhinitis medicamentosaRhinitis medicamentosa – Overuse topical decongestantsOveruse topical decongestants  Nonallergic rhinitis with eosinophiliaNonallergic rhinitis with eosinophilia – Diagnosis of exclusion, negative skin tests, nasalDiagnosis of exclusion, negative skin tests, nasal smear positive for eosinophilssmear positive for eosinophils www.indiandentalacademy.comwww.indiandentalacademy.com
  • 126. Question 77Question 77 A 5-month-old child is brought to the emergency department by herA 5-month-old child is brought to the emergency department by her mother because she has been “crying a lot” for the past 24 hours.mother because she has been “crying a lot” for the past 24 hours. The mother denies any history of fever, trauma, or illness. PhysicalThe mother denies any history of fever, trauma, or illness. Physical examination reveals a lethargic toddler who is very irritable whenexamination reveals a lethargic toddler who is very irritable when examined and who has mild tachycardia, scattered bruises over theexamined and who has mild tachycardia, scattered bruises over the chest, and ecchymosis behind the left ear.chest, and ecchymosis behind the left ear. The MOST appropriate management is toThe MOST appropriate management is to A.A.Administer intravenous naloxoneAdminister intravenous naloxone B.B.Administer a 20 mL/kg bolus of intravenous normal salineAdminister a 20 mL/kg bolus of intravenous normal saline C.C.Obtain an abdominal radiographObtain an abdominal radiograph D.D.Obtain a complete blood count and blood culture and administerObtain a complete blood count and blood culture and administer intravenous ceftriaxoneintravenous ceftriaxone E.E.Obtain a computed tomographic scan of the head and a skeletalObtain a computed tomographic scan of the head and a skeletal survey.survey. Answer: EAnswer: E www.indiandentalacademy.comwww.indiandentalacademy.com
  • 127. Teaching Points 77.1999Teaching Points 77.1999  Child abuse red flagsChild abuse red flags – Inconsistent historyInconsistent history – History not compatible with injuryHistory not compatible with injury  Ecchymosis behind left ear is called…Ecchymosis behind left ear is called… – Battle signBattle sign  What does this indicate?What does this indicate? – Basilar skull fractureBasilar skull fracture  CSF leakage from nose or ear, periorbitalCSF leakage from nose or ear, periorbital ecchymosis (Raccoon Sign), blood behind theecchymosis (Raccoon Sign), blood behind the eat drum (hemotympanum) are also signs ofeat drum (hemotympanum) are also signs of basal skull fracturebasal skull fracture www.indiandentalacademy.comwww.indiandentalacademy.com
  • 129. Periorbital EcchymosisPeriorbital Ecchymosis (Raccoon Eyes)(Raccoon Eyes) www.indianpediatrics.net/sep2005/sep-949.HTMwww.indianpediatrics.net/sep2005/sep-949.HTM  15 month old with stage IV metastatic15 month old with stage IV metastatic neuroblastoma and involvement ofneuroblastoma and involvement of the periorbital tissues and resultantthe periorbital tissues and resultant proptosis and orbital ecchymosisproptosis and orbital ecchymosis  Orbital metastases found in up toOrbital metastases found in up to 20% of children with stage IV20% of children with stage IV neuroblastoma.neuroblastoma.  Raccoon eyes appearanceRaccoon eyes appearance associated with neuroblastoma isassociated with neuroblastoma is probably related to obstruction of theprobably related to obstruction of the palpebral vessels by tumor tissue inpalpebral vessels by tumor tissue in and around the orbit.and around the orbit.  Differential diagnoses for periorbitalDifferential diagnoses for periorbital edema and ecchymosisedema and ecchymosis – Child abuse or traumaChild abuse or trauma – Infection of the soft tissues associatedInfection of the soft tissues associated with a spreading dental infectionwith a spreading dental infection – Allergic reactionAllergic reaction – MyxoedemaMyxoedema – LymphomaLymphoma – HaemophiliaHaemophilia www.indiandentalacademy.comwww.indiandentalacademy.com
  • 131. Question 78Question 78 An important component of the diagnostic assessment forAn important component of the diagnostic assessment for any child suspected of having attention deficit hyperactivityany child suspected of having attention deficit hyperactivity disorder (ADHD) is the use of standardized behavior ratingdisorder (ADHD) is the use of standardized behavior rating scales. Of the following, the BEST reason to usescales. Of the following, the BEST reason to use standardized behavior rating scales is that theystandardized behavior rating scales is that they A.A.Are specific for diagnosing ADHDAre specific for diagnosing ADHD B.B.Assist in monitoring treatmentAssist in monitoring treatment C.C.Can be completed after a single classroom observationCan be completed after a single classroom observation D.D.Determine initial medication dosageDetermine initial medication dosage E.E.Predict response to medicationPredict response to medication Answer: BAnswer: B www.indiandentalacademy.comwww.indiandentalacademy.com
  • 132. Teaching Points 78.1999Teaching Points 78.1999 www.indiandentalacademy.comwww.indiandentalacademy.com
  • 133. Teaching Points 78.1999Teaching Points 78.1999 www.indiandentalacademy.comwww.indiandentalacademy.com
  • 134. Question 79Question 79 A 14-year-old boy who has allergic rhinitis reports that heA 14-year-old boy who has allergic rhinitis reports that he frequently develops coughing and wheezing after aboutfrequently develops coughing and wheezing after about minutes of playing soccer. These symptoms improve afterminutes of playing soccer. These symptoms improve after resting for 30 minutes. Of the following, the drug that will giveresting for 30 minutes. Of the following, the drug that will give the BEST response in this patient if administered just prior tothe BEST response in this patient if administered just prior to exercise isexercise is A.A.Inhaled betaInhaled beta22-agonist-agonist B.B.Inhaled corticosteroidInhaled corticosteroid C.C.Oral betaOral beta22-agonist-agonist D.D.Oral corticosteroidOral corticosteroid E.E.Oral theophyllineOral theophylline Answer: AAnswer: A www.indiandentalacademy.comwww.indiandentalacademy.com
  • 135. Teaching Points 79.1999Teaching Points 79.1999  Exercise Induced AsthmaExercise Induced Asthma – Bronchoconstriction during exerciseBronchoconstriction during exercise – Typically within 15 minutesTypically within 15 minutes – Can occur during cool down as a lat-phaseCan occur during cool down as a lat-phase response up to 4-12 hours laterresponse up to 4-12 hours later – Cough, wheeze, SOB, dizzy, stomach painCough, wheeze, SOB, dizzy, stomach pain – Occurs 80% patients with asthmaOccurs 80% patients with asthma – Occurs 50% patients with allergic rhinitisOccurs 50% patients with allergic rhinitis www.indiandentalacademy.comwww.indiandentalacademy.com